Zufällige Graphen

Transcrição

Zufällige Graphen
Zufällige Graphen
SS 2009
Inhaltsverzeichnis
1 Einleitung und grundlegende Definitionen
1
2 Die Gradfolge
4
3 Schwellenfunktionen und Teilgraphen
15
4 Asymptotische Verteilungen
29
5 Verzweigungsprozesse
54
6 Der Poisson-Verzweigungsprozess
65
7 Der Phasenübergang im Erdös-Rényi-Graph
75
8 Der kritische Erdös-Rényi-Graph
95
9 Der Zentrale Grenzwertsatz für die riesige
Komponente
100
10 Inhomogene Zufallsgraphen
106
2
1
Einleitung und grundlegende Definitionen
Diese Vorlesung beschäftigt sich mit der Theorie zufälliger Graphen. Darunter wollen wir
die Mengen aller Graphen auf einer Vertexmenge, versehen mit einem Zufallsmechanismus,
verstehen. Es bietet sich an, zunächst zu definieren, was ein Graph ist.
Definition 1.1 Ein Graph G ist ein Tupel (V, E). Dabei ist V eine Menge von Punkten,
die man Knoten oder Vertices nennt. E ist entweder eine Teilmenge der Menge
{(i, j) : i, j ∈ V },
in welchen Fall man von eiem gerichteten Graphen spricht, oder
{{i, j} : i, j ∈ V };
dies ist der Fall eines ungerichteten Graphen.
Wir werden in dieser Vorlesung ausschließlich mit dem ungerichteten Fall befasst sein.
Beim Zufallsgraphen unterscheidet man nun zwischen verschiedenen Modellen. Dabei ist
ein Modell durch die Wahrscheinlichkeit bestimmt, mit der bestimmte Teilgraphen ausgewählt werden. Da wir V für gewöhnlich endlich wählen, müssen (und werden) wir uns
um die Wahl einer σ-Algebra keine Bedanken machen: Wir nehmen einfach die Potenzmenge der Menge aller möglichen Kanten.
Definition 1.2 Es sei V eine höchstens abzählbare Menge von Vertices. Ein Zufallsgraphenmodell ist ein Wahrscheinlichkeitsmaß P auf der Menge
V
:= {{i, j} : i, j ∈ V }.
2
Beispiel 1.3
a) Das wohl bekannteste Modell ist der sogenannte Erdös-Rényi-Graph
G(n, p). Hierbei ist
V = {1, . . . , n}.
Das Maß P wählt alle Kanten i.i.d. mit Wahrscheinlichkeit p, also hat jeder Graph
n
G = (V, E) mit |E| = k, 0 ≤ k ≤ n2 die Wahrscheinlichkeit P(G) = pk (1 − p)( 2 )−k
aufzutreten.
b) Eng verwandt mit dem Erdös-Rényi-Graphen ist das Modell G(n, M). Wieder wählen
wir als V = {1, . . . , n}. Über V wählt man jede Eckenmenge E mit |E| = M mit
gleicher Wahrscheinlichkeit, also
1
1l
.
(n2 ) {|E|=M }
P(G) =
M
1
c) (Perkolation) Eine geometrisch geordnete Version des Erdös-Renyi-Graphen ist das
Perkolationsmodell. Hierbei nimmt man ein zumeist regelmäßiges Gitter, oft den
Zd , d ≥ 1, löscht Kanten daraus mit Wahrscheinlichkeit 1 − p und behält sie mit
Wahrscheinlichkeit p bei – dies unabhängig für alle Kanten.
Man kann sich natürlich fragen, wozu man Zufallsgraphen betrachtet. Wir werden zumeist
den Standpunkt vertreten, dass es sich bei Zufallsgraphen um interessante mathematische
Objekte handelt, die bei unterschiedlicher Parameterwahl einen Wechsel im Verhalten
zeigen, einen sogenannten Phasenübergang. Darüber hinaus sind einzige Zufallsgraphen
gute und handhabbare Modelle für nicht-mathematische Phänomene, etwa:
• poröse Medien
• soziale Strukturen
• Internet
uvm. Die erstmalige Einführung von Zufallsgraphen geht allerding auf Paul Erdös zurück,
der sie als eine neue Beweistechnik, die probabilistische Methode, in die Graphentheorie
einführte:
Beispiel 1.4 Ein Km , m ≥ 2, ist ein vollständiger Graph auf m Kanten. In der Graphentheorie sind nun die Ramsey-Zahlen bekannt geworden. Wir sagen, dass die (k, l)’te
Ramseyzahl R(k, l) = n ist, wenn jede Färbug der Kanten des kn mit blau und rot immer
entweder einen blauen Kl oder einen roten Kl liefert. Beispielsweise ist R(2, 2) = 2, weil
man die eine Kante zwischen den beiden Punkten entweder rot oder blau färben muss.
Man sieht auch sofort, dass R(m, 2) = R(2, m) = m gilt, denn entweder ist eine Kante
rot (bzw. blau) gefärbt oder man hat einen blauen (bzw. roten) Km .
Die Frage nach anderen Ramsey-Zahlen R(k, l) ist aber schwierig, insbesondere ist für die
Diagonalzahlen R(k, k) nur wenig bekannt: Man weiß, dass R(3, 3) = 6 und R(4, 4) = 18
gilt. Für R(5, 5) ist bekannt, dass
43 ≤ R(5, 5) ≤ 49
gilt und ähnlich
102 ≤ R(6, 6) ≤ 165.
Von Erdös ist die Anekdote überliefert, dass wir, falls uns eine weit überlegene außerirdische Macht angriffe und drohte, die Welt zu zerstören, falls wir nicht R(5, 5) angeben
können, alles daran setzen sollten, alle Computer und Wissenschaftler, die wir besitzen,
um R(5, 5) zu berechnen. Sollten die Aliens allerdings R(6, 6) willen wollen, so sollten wir
versuchen, sie zu zerstören.
Die probabilistische Methode gibt nun eine Möglichkeit, eine untere Schranke für R(k, k)
zu berechnen (dass R(l, l) für alle k, l endlich ist, zeigte Ramsey 1929). Es gilt
R(k, k) ≥ 2k/2 .
Dieses Resultat geht auf Erdös (1947) zurück.
2
Beweis: Da R(2, 2) = 2 und R(3, 3) = 6 gilt, nehmen wir k ≥ 4 an. Sei N < 2k/2 .
Wir färben alle Kanten unabhängig mit Wahrscheinlichkeit 12 rot oder blau. Also hat jede
n
Färbung die Wahrscheinlichkeit 2−( 2 ) . Sei
A ⊂ {1, . . . , n}, |A| = k.
Sei
AR = {Die Farbe jeder Kante in A ist rot}.
Also ist
k
P(AR ) = 2−(2) .
Damit folgt
P(∃A ⊆ {1, . . . , n}, |A| = k, A ist rot gefärbt)
[
X
= P
AR ) ≤
P(AR )
|A|=k
N −k
2 2.
=
k
|A|=k
Nun lässt sich zeigen (einfache Übung)
Nk
N
≤ k−1 .
2
k
Da außerdem N < 2k/2 und k ≥ 4 vorausgesetzt war, folgt
k
k
k2
k
Nk
N −(k2)
1
≤ k−1 2−(2) < 2 2 −(2)−k+1 = 2− 2 +1 ≤ .
2
k
2
2
Aus Symmetriegründen folgert man analog
1
P(∃A : |A| = k, A ist blau gefärbt) < .
2
Somit ist
P(∃A : |A| = k, A ist entweder ganz rot oder ganz blau gefärbt) < 1.
Also gibt es Färbungen, bei denen |A| = k auftreten, die beide Farben tragen.
2
Die probabilistische Methode hat heutzutage in viele Gebiete Einzug gehalten (vgl. das
Buch von Noga Alon und Joel Spencer).
Im Laufe dieser Vorlesung werden wir zunächst (und vielleicht ausschließlich) mit den Modellen G(n, p) und G(n, M) beschäftigt sein. Dabei werden verschiedene Aspekte dieser
Modelle betrachtet. Für manche wird es wichtig sein, den festen Wahrscheinlichkeitsraum
G(n, p) bzw. G(n, M) zu betrachten. Bei anderen werden wir eher einen Prozessaspekt
betrachten, also beispielsweise einen Graphen allmählich mit Kanten füllen und daher
zueinander passende Realisierungen von (G(n, M))0≤M ≤(n) betrachten. Beide Gesichts2
punkte illustrieren unterschiedliche Phänomene der Zufallsgraphen.
3
2
Die Gradfolge
Definition 2.1 Es sei G = (V, E) ein Graph und v ∈ V . Der Grad v, deg(v), ist definiert
als die Anzahl aller Nachbarn von v:
deg(v) = |{w : {v, w} ∈ E}|.
Die Folge der Grade ist eine der wichtigsten Charakteristika eines Graphen. Mit ihr wollen
wir uns in diesem Abschnitt befassen. Dabei legen wir stets das Modell G(n, p) zugrunde.
Sei also G ∈ G(n, p) (womit wir meinen, der Graph G wird gemäß G(n, p) gezogen). Sei
(di )ni=1 die Folge der Grade der Vertices, die wir schon in absteigender Reihenfolge sortiert
haben, also
d1 ≥ d2 ≥ . . . ≥ dn .
Den maximalen Grad d1 und den minimalen Grad dn bezeichnen wir auch mit ∆(G) bzw.
δ(G). Es sei Xk (G) die Anzahl der Knoten in G mit Grad genau k, Yk sei die Anzahl der
Knoten mit Grad mindestens k und Zn die Anzahl an Vertices mit Grad höchstens k, also
X
X
Yk =
Xk und Zk =
Xl .
l≥k
l≤k
Offenbar ist
dr ≥ k ⇔ Yk ≥ r und
dn−r ≤ k ⇔ Zk ≥ r + 1.
Wir beginnen mit einem langweiligen Fall.
Proposition 2.2 Falls p = o(n−3/2 ), so besteht G aus unabhängigen (unverbundenen)
Kanten.
Beweis: Es gilt
EY2
n
X
n−1 X
n−1 j
p
=
EXj ≤ n
j
j=2
j=2
≤ n·
n−1
X
(pn)j
j=2
j!
= o(1),
aufgrund der Voraussetzung.
2
Gilt zudem pn2 → 0, so gibt es in G asymptotisch P-f.s. gar keine Kanten, d. h.
P(G hat Kanten) → 0,
4
ist pn2 → ∞, aber p = o(n−3/2 ), so gibt es ein M, so dass
d1 = . . . = d2M = 1 und d2M +1 = . . . = dn = 0
gilt. M ist dabei von der Größenordnung p n2 ∼ pn2 /2. Mehr gibt es in diesem Fall kaum
zu sagen, daher nehmen wir von nun an p ≫ n−3/2 an (wobei wir a(n) ≫ b(n) schreiben,
wenn b(n) = o(a(n)) gilt).
Satz 2.3 Sei ε > 0 fest und
εn−3/2 ≤ p = p(n) ≤ 1 − εn−3/2 .
Es sei k = k(n) ∈ N. Setze
λk = λk (n) = nB(k; n − 1, p).
Dann gilt:
(i) Falls limn→∞ λk (n) = 0, dann ist lim P(Xk = 0) = 1.
(ii) Falls limn→∞ λk (n) = ∞, dann ist
lim P(Xk ≥ t) = 1
∀ t fest.
(iii) Falls
0 < lim λk (n) = lim λk (n) < +∞,
dann ist Xk asymptotisch Poi(λk )-verteilt.
Beweis: OBdA sei ρ ≤ 21 . Bemerke, dass
λk = nB(k; n − 1, p) = EXk .
Daher:
Daraus folgt (i).
P(Xk ≥ 1) ≤ EXk = λk (n).
Sei also von nun an lim λk (n) > 0. Wir zeigen nun, dass für jedes r ≥ 1, das r-te faktorielle
Moment von Xk
Er Xk := EXk (Xk − 1) . . . (Xk − r + 1)
(2.1)
asymptotisch wie λrk (n) geht. Da Xk die Anzahl der Vertices vom Grad k ist, ist Er Xk
die erwartete Anzahl geordneter r-Tupel von Vertices vom Grad k. Um also Er Xk zu
berechnen, fragen wir nach der Wahrscheinlichkeit, dass gegebene Knoten x1 , . . . , xr allesamt den Grad k haben. Dazu betrachten wir zunächst die Kanten, die die x1 , . . . , xr
untereinander verbinden. Wir nehmen an, dass es l solcher Kanten gibt, und dass xi zu
d˜i ≤ k der x1 , . . . , xr verbunden ist, dann folgt
r
X
d˜i = 2l
i=1
5
und xi muss mit k− d˜i der Knoten verschieden von x1 , . . . , xr verbunden werden. Aufgrund
der Unabhängigkeit der Kanten hat dieses Ereignis die Wahrscheinlichkeit
r
Y
i=1
B(k − d˜i ; n − r, p).
Wir unterscheiden zwei Fälle: Zum einen sei (p(n)) von 0 weg beschränkt. Dann folgt aus
lim λk (n) > 0, dass die relevanten Werte k(n) von der Ordnung pn sind, z. B.
1
3
pn ≤ k(n) ≤ pn
2
2
(dies lässt sich schon aus der Chebyshev-Ungleichung ablesen, genauer gilt sogar
p
k(n) = pn + O(n log n),
wie man aus dem Gesetz vom iterierten Logarithmus erkennt). Hieraus leitet man ab
(Übung), dass es eine Funktion φr (n) mit limn→∞ φr (n) = 0 gibt, so dass
B(k − d, n − r, p)
≤ φr (n)
−
1
B(k; n − 1, p)
für alle festen r und d gilt. Aus (2.1) folgt dann
Er (Xk ) ∼ (n)r B(k; n − 1, p)r ∼ λrk
(wobei
(n)r := n(n − 1) . . . (n − r + 1)).
Der zweite Fall ist, dass p = o(1) gilt. Dann schätzen wir Er Xk folgendermaßen ab:
Er Xk ≤ (n)r
R X
R
l=0
l
l R−l
pq
Pmax
d∗i =2l
r
Y
i=1
B(k − d∗i ; n − r, p),
(2.2)
wobei (n)r die Auswahlmöglichkeiten für die xi beschreibt, R = r2 und q = 1 − p ist, die
Summe für die inneren und das Maximum für die äußeren Verbindungen steht. Es wird
hierbei natürlich über alle d∗i mit d∗i ≤ min{r − 1, k} maximiert. Da lim λk (n) > 0 und
p = o(1) gilt, folgt k(n) = o(n) für alle relevanten k. Ist also 0 ≤ d ≤ min{r − 1, k(n)}
und ist n genügend groß, so folgt
d
B(k − di n − r, p)
(k)d
k
−d
≤
p <2 n .
b(K, N − R, P )
(n − r − k)d
p
Also folgt aus (2.2)
2l )
R X
k
R
pl 2 r
Er (Xk ) ≤ nr B(k, n − r, p)r 1 +
pn
l
l=1
)
(
R 2 l
X
k
2
.
≤ nr B(k, n − r, p)r 1 + 2r
2
pn
l=1
(
6
(2.3)
Die Annahme, dass lim λk (n) > 0 impliziert, dass k 2 = o(pn2 ). Wäre nämlich für ein
√
η > 0 k ≥ η pn für beliebig große n, so folgte
en k
n k
pk
p ≤ lim n
lim λk (n) ≤ lim n
k
k
epn k
= lim n
k
√ η√pn
e p
≤ lim n
η
≤ lim n2−n
1/5
= 0,
im Gegensatz zur Annahme. Da k 2 = o(pn2 ), folgt aus (2.3)
Er Xk ≤ nr B(k, n − r, p)r {1 + o(1)} = λrk {1 + o(1)}.
Betrachtet man nur unabhängige, d. h. unverbundene, Vertices, so ergibt sich
Er Xk ≥ q R (n)r B(k, n − r, p)r = λrk {1 + o(1)},
da q → 1 konvergiert. Also gilt
Er Xk ∼ λrk .
Berechnet man umgekehrt das r-te faktorielle Moment einer Poi(λk )-verteilten Zufallsvariablen Y , so ergibt sich
Er Y ∼ λrk . (Übung)
Gilt nun lim λk (n) = +∞, dann folgt aus E2 (Xk ) ∼ λ2k auch
EXk2 = λ2k (1 + o(1)).
Mithilfe der Chebyshev-Ungleichung ergibt sich
lim P(Xk ≥ t) = 1
n→∞
für jedes feste t. Gilt schließlich lim sup λk < +∞, so folgt die Behauptung aus dem
untenstehenden Satz 2.4 und
Er (Xk ) ∼ λrk .
2
Satz 2.4 Sei λ = λ(n) eine nicht-negative beschränkte Funktion auf N. Seien X1 , X2 , . . .
ganzzahlige Zufallsvariablen, so dass
lim (Er (Xn ) − λr ) = 0 r = 0, 1, . . . .
n→∞
Dann gilt auch
dT V (PXn , Pλ ) → 0
für die totale Variationsdistanz dT V .
7
Beweis: Siehe Bollobas, Random Graphs, Theorem 1.22.
2
Man kann den Beweis von Satz 2.3 so modifizieren, dass man entsprechende Resultate für
Yk und Zk erhält:
Satz 2.5 Sei ε > 0 und
εn−3/2 ≤ p(n) =: p ≤ 1 − εn−3/2
und sei k = k(n) ∈ N. Setze
µk = nB̄(k, n − 1, p) und
wobei
B̄(l, m, p) =
νk = n(1 − B̄(k + 1, n − 1, p)),
X
B(j; m, p).
j≥l
Dann gelten die Behauptungen aus Satz 2.3, wenn wir Xk und λk durch Yk und µk oder
Zk und νk ersetzen.
Dieses Resultat erlaubt es uns, die ersten und letzten Elemente der Gradfolge in Verteilung
zu bestimmen. Ein paar Resultate für moderat große Werte von p sehen folgendermaßen
aus:
Satz 2.6 Sei p = p(n) so dass
p(1 − p)
→ ∞.
(log n)3
Seien c > 0 und m ∈ N fest und x = x(c, n) so, dass
Z ∞
2
n·
e−y /2 dy = c,
x
d. h. approximativ für großes x,
1 n −x2 /2
√
e
= c.
2π x
Dann gilt
lim P(dm < pn + x(pqn)
n→∞
1/2
−c
)=e
m−1
X
k=0
mit q := 1 − p.
Beweis: Setze
K := ⌈pn + x(pqn)1/2 ⌉.
Dann ist
dm < pn + x(pqn)1/2
8
ck
k!
gleichbedeutend damit, dass höchstens m − 1 Vertices mindestens Grad K haben. Also:
YK ≤ m − 1.
Gemäß Satz 2.5 konvergiert die Verteilung von YK gegen P oi(µK ), wobei
µK := nB̄(K; n − 1, p)
ist. Nach Definition x gilt:
x(pqn)1/2 = o((pqn)2/3 ).
Aus dem Satz von de Moivre-Laplace folgt daher
c
· n = c.
µK ∼
n
Daraus folgt der Satz.
2
√
Der Wert von x in diesem Satz ist ungefähr 2 log n für jedes c. Außerdem gilt
√ log log n log(2 π)
1
1/2
1−
.
x(1, n) = (2 log n)
+o √
−
4 log n
2 log n
log n
Also gilt für jedes δ = o(1), dass x′ = x(1, n) + δ zu einem Wert c′ ∼ exp(−(2 log n)1/2 δ)
gehört. Somit lässt sich Satz 2.6 auch schreiben als
Satz 2.7 Sei
pqn
(log n)3
→ ∞ und y ∈ R fest. Dann gilt
√
m−1
X e−ky
p
log log n log(2 π)
y
−y
lim P dm < pn + 2pq log n 1 −
= e−e
−
+
.
n→∞
4 log n
2 log n
2 log n
k!
k=0
Betrachtet man d1 , so erhält dieses Resultat eine aus der Extremwerttheorie sehr bekannte
Form.
Korollar 2.8 Sei
pqn
(log n)3
→ ∞ und y ∈ R fest. Dann gilt für den Maximalgrad ∆ = d1
√
p
log log n log(2 π)
y
−y
lim P d1 < pn + 2pq log n 1 −
= e−e .
−
+
n→∞
4 log n
2 log n
2 log n
Für p-Werte, die sehr nahe bei 0 oder 1 sind, lassen sich noch aus Satz 2.4 über den
maximalen und minimalen Grad ableiten. Der Beweis des folgenden Satzes ist eine (etwas
längere) Übung.
Satz 2.9 Seien k, x und y fest, k ≥ 2, x > 0 und y ∈ R. Falls
p ∼ x n−
9
k+1
k
ist, dann folgt
P(∆(G) = k) → 1 − e−x
und
P(∆(G) = k − 1) → e−x
Falls
p≈
k /k!
.
(log n + k log log n + y)
n
gilt, so folgt
P(δ(G) = k) → 1 − e−
und
k /k!
P(δ(G) = k + 1) → e−e
e−y
k!
−y/k!
.
Wir wollen uns nun fragen, ob es eine Art Gesetz der großen Zahlen gibt, also Werte von p,
für die für fast alle G ∈ G(n, p) der Maximalgrad derselbe ist. Ähnliches lässt sich natürlich
auch für den Minimalgrad fragen. Schließlich können wir auch die Frage untersuchen, ob
die Knoten maximalen (oder minimalen) Grades eindeutig sind. Diese Analyse basiert auf
Satz 2.3. Zunächst zeigen wir, dass, falls fast alle G ∈ G(n, p) den gleichen Maximalgrad
haben, p = o( logn n ) gilt. Wir werden auch sehen, dass diese Bedingung beinahe hinreichend
ist.
Satz 2.10 Sei p ≤ 12 . Angenommen, es gibt eine Funktion D(n), so dass in G(n, p) gilt
lim P(∆(G) = D(n)) = 1.
n→∞
Dann folgt
p=o
log n
n
.
Beweis: Nach Satz 2.3 müssen wir
lim nB(D; n − 1, p) = +∞
n→∞
und
lim nB(D + 1; n − 1, p) = 0
n→∞
haben, um das gewünschte Resultat zu erzielen. Daher gilt
B(D + 1; n − 1, p)
p(n − D − 1)
= lim
= 0,
n→∞
n→∞ (1 − p)(D + 1)
B(D; n − 1, p)
lim
→ 0, da n − D − 1 nicht gegen 0 geht und 1 − p auch nicht gegen ∞. Unter
also pn
D
Ausnutzung von
nB(D; n − 1, p) = +∞
erhalten wir, dass auch für jedes c > 1
lim nB(⌈cpn⌉, n − 1, p) = +∞
10
gilt (denn c np
→ 0). Also:
D
lim n
n→∞
epn
cpn
cpn
= lim n
n→∞
e cpn
c
= +∞
Dies ist für alle c > 0 nur möglich, wenn p = o( logn n ) ist.
2
Satz 2.11 Sei p = o( logn n ) und k = k(n) ≥ pn so, dass max{λk (n), λk (n)−1 } minimal ist.
Dann gilt:
(i) Ist 0 < lim λk (n) ≤ lim λk (n) < ∞, dann ist
P(∆(G) = k(n)) ∼ 1 − e−λk (n)
und
P(∆(G) = kn (n) − 1) ∼ e−λk (n) .
(ii) Gilt lim λk (n) = +∞, so hat G P-f.s. maximalen Grad k(n) und gilt lim λk (n) = 0,
dann hat G P-f.s. maximalen Grad k(n) − 1.
Beweis: Man rechnet nach, dass
pn
λk+1
k
∼
→ ∞,
→0
pn
λk
k+1
und
λk
pn
∼
→0
λk−1
k
gilt. Also gilt λk+1 → 0 und λk−1 → ∞. Nach Satz 2.3 bedeutet λk−1 → ∞, dass G P-f.s.
einen Vertex mit Grad k(n) − 1 enthält. Also ist der Maximalgrad von G mindestens
k(n) − 1. Darüber hinaus gilt
P(∆(G) ≥ k + 1) = P(
n−1
X
j=k+1
Xj ≥ 1) ≤ E
n+1
X
EXj = O(λk+1) = o(1).
j=k+1
Also ist der Grad von G P-f.s. k(n) − 1, es sei denn es gibt einen Knoten vom Grad k(n),
in diesem Fall ist der Maximalgrad k(n).
P(∆(G) = k(n)) ∼ P(Xk ≥ 1)
und
P(∆(G) = k(n) − 1) ∼ 1 − P(∆(G) = k(n)).
Schließlich kennen wir das Verhalten von P(Xk ≥ 1) aus Satz 2.3.
2
Für p ≤ 21 ist das Verhalten des Minimalgrades etwas anders als das des Maximalgrades.
Es gelten die folgenden Sätze:
11
Satz 2.12
(i) Sei p ≤ 12 . Angenommen, es gebe eine Funktion d(n), so dass in G(n, p)
lim P(δ(n) = d(n)) = 1
n→∞
gilt. Dann folgt
p ≤ (1 + o(1)) log n/n.
(ii) Gilt p ≤ (1+o(1)) logn n , dann gibt es eine Funktion d(n), so dass δ(n) P-f.s. entweder
d(n) oder d(n) − 1 ist.
Beweis:
(i) Wir schreiben als p =
log n+ω(n)
.
n
Ist dann ω(n) = o(log n), so ist
n
λ0 = nB(0, n − 1, p) = n(1 − p)n−1 ∼ e−ω(n) = e−ω(n) .
n
Dann folgt aber mit Satz 2.3, dass G P-f.s. einen isolierten Knoten hat, wenn ω(n) →
für eine Konsante C gilt.
−∞ geht. Daher können wir annehmen, dass p ≥ log n+C
n
Wie im Beweis von Satz 2.10 sieht man ein, dass, falls
lim P(δ(n) = d(n)) = 1
n→∞
gilt, für jedes feste ε mit 0 < ε < 21 , gilt
εpn
e
√
2
pεpn e−pn+p εn → ∞.
n εpn
εp
Also:
1
1
log pn + pn(ε + ve log + ε − 1) → ∞.
2
ε
Da dies für beliebig kleine ε > 0 gilt, folgt, dass
log n
log n
.
+o
p≤
n
n
log n −
(ii) Geht wie im Beweis von Satz 2.11.
2
Wir wenden uns nun der Eindeutigkeit des Vertex mit maximalem Grad zu:
Satz 2.13
(i) Sei p ≤
1
2
und
pn
→ ∞,
log n
dann hat P-f.s. G einen eindeutigen Knoten von maximalem Grad und einen eindeutigen Knoten von minimalem Grad.
(ii) Ist p ≤ 21 und hat P-f.s. G einen endeutigen Knoten von maximalem und einen
eindeutigen Knoten von minimalen Grad, dann gilt
pn
→ ∞.
log n
12
Beweis:
(i) Sei k maximal mit
n·
Man überprüft schnell, dass
n−1
X
l=k
B(l, n − 1, p) ≥ 1.
k > pn, k ∼ pn, nB(k − 1; n − 1, p) → 0
und
B(k; n − 1, p)
→1
B(k − 1; n − 1, p)
gilt. Also kann man l = l(n) so wählen, dass
pn < l < k und nB(l, n − 1, p) → 0
n−1
X
B(l, n − 1, p) → ∞
und n
k=l
gilt. Variiert man l so, dass dies wahr bleibt, kann man ein m, pn < m < k finden,
so dass
n−1
X
n
B(i; n − 1, p) → ∞ und
n
k=m
n−1
X
i=m
B(i, n − 1, p)nB(m; n − 1, p) → 0
gilt. Wir behaupten nun, dass P-f.s. einen, aber nur einen Knoten vom Grad mindestens m gibt. Dies folgt, da die erwartete Anzahl von Knoten vom Grad m
E[Ym ] = n
n−1
X
i=m
B(i; n − 1, p)
ist. Also gilt E[Yn ] → ∞. Aus Satz 2.4 folgt daher, dass ∆(G) ≥ m P-f.s. Andererseits berechnet sich die Wahrscheinlichkeit, dass zwei Vertices den gleichen Grad
von mindestens m haben als
X
X
P[Xj ≥ 2] ≤
E2 (Xj )
j≥m
j≥m
(wobei E2 das 2te faktorielle Moment bezeichnet). Nun gilt
X
X
E2 (Xj ) =
n(n − 1)(pB(j − 1, n − 2, p)2 + (1 − p)B(j; n − 2, p)2 )
j≥m
j≥m
≤
X
j≥m
n2 B(j; n − 2, p)2
≤ nB(m − 1; n − 2, p)n
∼ nB(m, n − 1, p)n
Also
P
j≥m P(Xj
X
j≥m
X
j≥m−1
B(j; n − 2, p)
B(j; n − 1, p) → 0.
≥ 2) → 0. Dies beweist die Behauptung.
13
(ii) Ist eine Übung.
2
Das Gesamtbild ist also dieses:
• Ist p = 0( logn n ), so sind Minimal- und Maximalgrad P-f.s. festgelegt (und es gibt
mehrere Vertices von diesem Grad).
• Ist ε logn n ≤ p ≤ (1 + o(1)) logn n für ein ε > 0, so ist der Minimalgrad festgelegt
(und es gibt viele Knoten von diesem Grad), wäahrend das Maximalgrad nicht mit
Wahrscheinlichkeit 1 konzentriert ist.
• Ist (1 + ε) logn n ≤ p ≤ C logn n für ε0, C > 0, dann sind weder Maximal- noch
Minimalgrad f.s. festgelegt.
pn
→ ∞, p ≤
• Scließlich ist für log
n
bzw. minimalem Grad.
1
2
P-f.s. ein eindeutiger Knoten von maximalen
14
3
Schwellenfunktionen und Teilgraphen
Einer der bemerkenswertesten Entdeckungen von Erdös und Rény bei der Untersuchung
von Zufallsgraphen ist das Auftreten von Schwellen. Damit meinen wir, dass für viele
Eigenschaften eines Zufallsgraphen sich die Wahrscheinlichkeit ihres Auftretens in einem
sehr kleinen Fenster von 0 auf 1 (oder 1 auf 0) ändert, wenn man p varriiert.
Definition 3.1 Wir nennen A eine wachsende Eigenschaft (und schreiben G ∈ A, wenn
der G die Eigenschaft A hat), falls p → P[G ∈ A] wachsend ist. Ist p 7→ P[G ∈ A] fallend,
so heißt A fallend.
Bemerkung: Diese Definition ist ursprünglich nur eine Folgerung aus der ursprünglichen
Definition, die besagt, dass für G ∈ A und G′ ⊇ G auch G ∈ A gilt.
Definition 3.2 p̂ = p̂(n) heißt Schwellenfunktion für die wachsende Eigenschaft A, falls
P[G ∈ A] →
Dabei schreiben wir p ≪ p̂, falls
und p ≫ p̂, falls
0, falls p ≪ p̂(n)
.
1, falls p ≫ p̂(n)
p(n)
→0
p̂(n)
p(n)
→∞
p̂(n)
gilt.
Ähnliches ließe sich auch im Modell G(n, M) definieren, aber wir werden in Rahmen dieser
Vorlesung zunächst beim Modell G(n, p) verharren. Wir zeigen zunächst
Satz 3.3 Jede monotone Eigenschaft hat eine Schwellenfunktion.
Beweis: OBdA sei A eine monoton wachsende Grapheneigenschaft. Sei 0 < ε < 1 und
m ∈ N mit
(1 − ε)m ≤ ε.
Sei p so klein, dass P(G ∈ A) ≤ ε in G(n, p). Betrachte unabhängige Kopien G(1) , . . . , G(m)
von G ∈ G(n, p) auf derselben Knotenmenge. Ḡ sei ihre Vereinigung. Beachte, dass Ḡ ∈
G(n, p′ ) mit
p′ = 1 − (1 − p)m ≤ mp
gilt. Daher gilt
P(G(1) ∪ . . . ∪ G(m) ∈ A) ≤ P(Gmp ∈ A).
15
Andererseits ist A wachsend und daher hat G(1) ∪ . . . ∪ G(m) die Eigenschaft A, falls
G(i) ∈ A für irgendein i gilt. Daher folgt
P[G(1) ∪ . . . ∪ G(m) ∈
/ A] ≤ P[G(i) ∈
/ A für jedes i]
= (1 − P[G ∈ A])m = (1 − ε)m ≤ ε.
Also gilt
P(Ḡ ∈ A) ≥ P[G(1) ∪ . . . ∪ G(m) ∈ A] ≥ 1 − ε.
Also ist für alle p̃ ≥ mp Pp̃ (G ∈ A) ≥ 1 − ε. Bezeichne nun p(ε) den P -Wert für den
Pp(ε) [G ∈ A] = 1 − ε
gilt. Dann folgt für ε <
1
2
1
p(ε) ≤ p( ) ≤ p(1 − ε) ≤ mp(ε),
2
wobei m von ε, aber nicht von n abhängt. Also ist
1
≍ p(1 − ε).
p(ε) ≍ p
2
Aus der folgenden Proposition folgt dann, dass beispielsweise p( 21 eine Schwellenfunktion
ist.
2
Proposition 3.4 Angenommen, A ist eine wachsende Eigenschaft von Teilmengen in
G(n, p). Es sei p(ε) definiert, so dass Pp(ε) [G ∈ A] = ε. p̂(n) ist genau dann eine Schwellenfunktion, wenn
p(ε) =: p(ε; n) ≍ p̂(n)
für jedes ε ∈ (0, 1).
Beweis: Wir verweisen auf das Buch “Random graphs” von Janson, Luczak, Ruciński,
Proposition 1.23.
2
Ein berühmtes Beispiel für Schwellenfunktionen sind die Situationen, wo wir untersuchen,
ob ein G ∈ G(n, p) einen gegebenen Graphen, sagen wir Γ, als Teilgraphen enthalten, also
beispielsweise ob ein G ∈ G(m, p) ein Dreieck enthält. Offenbar ist dies eine wachsende
Eigenschaft, sie sollte also nach dem Vorhergehenden eine Schwellenfunktion sein.
Die Technik, dies herzuleiten, ist die Methode des ersten und zweiten Moments, die eigentlich nur aus der Tschebyschev-Ungleichung besteht. Da die Zufallsgraphen-Theoretiker
darauf sehr stolz sind, werden wir sie abstrakt und an einem Beispiel vorstellen.
Die erste Momentenmethode ist eine einfache Anwendung der Markov-Ungleichung: Für
jede Zufallsvariable X mit Werte in N0 gilt
P[X > 0] ≤ EX.
16
Die Methode des ersten Moments besteht nun darin zu zeigen, dass EXn = o(1) gilt und
sonst Xn = 0 asymptotisch fast sicher (das ist Graphentheorie-Slang für P[Xn = 0] → 1).
Die zweite Momentenmethode oder Methode des 2. Moments besteht aus einer Anwendung der Chebyschev-Ungleichung. Sei X eine Zufallsvariable mit EX > 0. Dann ist
{X = 0} ≤ {|X − EX| ≥ EX}, also
P(X = 0) ≤ P(|X − EX| ≥ EX) =
VX
.
(EX 2 )
(3.1)
Kann man also zeigen, dass die rechte Seite gegen 0 geht, so folgt Xn 6= 0 asymptotisch
p
VX
Xn
−→ 1.
fast sicher. Ähnlich sieht man, dass für Zufallsvariablen X mit (EX)
2 = o(1) gilt EX
n
Bemerkung 3.5 Ungleichung (3.1) lässt sich noch verbessern. Aus der Cauchy-SchwarzUngleichung (angewandt auf X = X1lX6=0 ) lässt sich z. B. gewinnen:
(EX)2 ≤ EX 2 P(X 6= 0),
also
(EX)2
P(X =
6 0) ≥
EX 2
und daher
P(X = 0) ≤ 1 −
VX
VX
(EX)2
=
=
.
2
2
EX
EX
(EX)2 + VX
Dies ist manchmal besser als (3.1).
Beispiel 3.6 Wir wenden uns für einen Moment von der Graphentheorie ab: Sei [n]p
der Wahrscheilichkeitsraum, der aus allen Teilmengen von {1, . . . , n} besteht, wobei jedes
Element A ⊆ {1, . . . , n} mit Wahrscheinlichkeit p|A| (1 − p)n−|A| gezogen wird, wir werfen
also für jede Zahl eine p-Münze und entscheiden, ob diese Zahl teilnimmt. Wir fragen nach
der Anzahl der arithmetischen Progressionen der Länge k in einem zufälligen Element aus
[n]p . Sei diese Xk und k ≥ 2 fest. Wir müssen zunächst eine asymptotische Abschätzung
der Anzahl der arithmetischen Progressionen der Länge k in {1, . . . , n}, f (n, k), haben.
Wir wählen f (n, k) ∼ n2 , da eine Progression durch die ersten beiden Elemente festgelegt
ist und es knapp n2 Wahlen dafür gibt. Für i = 1, . . . , f (n, k) sei nun Ii der Indikator,
der angibt, ob die i-te Progression in [n]p auftaucht oder nicht. Dann ist
f (n,k)
Xk =
X
Ii
i=1
und daher
EXk = f (n, k)pk = Θ(n2 pk ),
wobei wir an = Θ(bn ) schreiben, falls es Konstanten gibt, so dass für hinreichend großes
n gilt:
can ≤ bn ≤ Cbn
17
(also an = O(bn ) und bn = O(an )). Gilt also p ≪ n−2/k , so folgt E(Xk ) → 0 und nach der
ersten Momentenmethode gilt
P(Xk > 0) = o(1),
d. h. Xk = 0 asymptotisch f.s.
Ist andererseits p ≫ n−2/k , so gilt EXk → ∞, was natürlich alleine nicht genügt, um
P(Xk > 0) → 1 zu zeigen. Hier benutzen wir die zweite Momentenmethode. Beachte,
dass Ii und Ij unabhängig sind, falls die i-te und die j-te arithmetische Progression keine
gemeinsamen Elemente besitzt; in diesem Fall ist Cov(Ii , Ij ) = 0. Ansonsten schätzen wir
ab:
Cov(Ii, Ij ) ≤ EIi Ij .
Es gibt O(n3 ) Paare (Ii , Ij ), die ein Element gemeinsam haben und O(n2 ) Paare mit zwei
Elementen oder mehr gemeinsam. Im ersten Fall ist
Cov(Ii, Ij ) = p2k−1 ,
im zweiten
Cov(Ii , Ij ) ≤ pk .
Also gilt
f (n,k) f (n,k)
VXk =
X X
i=1
Cov(Ii , Ij ) = O(n3p2k−1 + n2 pk ).
j=1
Also folgt mit der zweiten Momentenmethode
1
1
P(Xk = 0) = O
= o(1).
+
np n2 pk
Also ist die Schwellenfunktion für das Auftreten einer arithmetischen Progression der
Länge k n−2/k .
Wir wollen nun sehen, was die Schwelle für das Auftreten von Teilgraphen ist.
Sei also Γ ein Graph und XΓ die Anzahl von Kopien von Γ, die in einer Realisation eines
Graphen G gemäß G(n, p) enthalten ist. Seien v = vΓ und e = eΓ die Anzahl von Knoten
bzw. Kanten von Γ. Betrachtet man den vollständigen Graphen auf n Knoten, Kn , dann
gibt es in Kn genau
n
v!/aut(Γ)
f (n, Γ) =
v
Kopien von Γ, wobei aut(Γ) die Größe der Automorphismengruppe von Γ ist. Für jede
Kopie Γ′ von Γ in Kn sei
IΓ′ = 1lΓ′ ⊆G(n,p)
der Indikator dafür, dass Γ′ in G(n, p) auftritt. Dann ist
(
0, falls p ≪ n−v/e
EXΓ = f (n, Γ)pe = Θ(nv pe ) →
∞, falls p ≫ n−v/e
18
und gemäß der Methode des ersten Moments folgt
P[XΓ > 0] = o(1),
falls p ≪ n−v/e .
(3.2)
Stimmt es auch, dass
P[XΓ > 0] = 1 − o(1),
falls p ≫ n−v/e .
Wir betrachten ein Beispiel:
Beispiel 3.7 Es sei H0 der Graph mit 4 Knoten und 5 Kanten:
und G0 und der Graph, der aus H0 entsteht, wenn man einen weiteren Knoten mit einer
einzigen Kante zu H0 hinzufügt (es gibt zwei nicht-isomorphe Arten dies zu tun, es ist
hier aber unerheblich, welche wir wählen):
An diesem Beispiel werden wir sehen, dass n−v/e nicht stets die Schwellenfunktion für das
Auftreten eines Subgraphen sein kann. Sei p(n) = n−9/11 , wobei daran nur wichtig ist,
dass
n−5/6 ≪ p ≪ n−4/5
gilt. Dann gilt
EXG0 = Θ(n5 p6 ) → ∞,
aber wendet man (3.2) auf H0 an, so sieht man, dass asymptotisch fast sicher keine Kopien
von H0 in G vorhanden sind, also können auch keine Kopien von H0 vorhanden sein.
Also ist die Lage für Teilgraph-Zählungen etwas komplizierter als für arithmetische Progressionen. Warum hat die Methode im vergangenen Beispiel nicht geklappt? Wenn man
etwas überlegt, ist die Antwort einfach. Der Graph H0 , der ein Teilgraph von G0 ist, hat
eine höhere “Dichte” ve als G0 . Man muss zunächst sicherstellen, dass p so groß ist, dass
H0 auftreten kann. Die “Extrakante” bekommt man dann gratis.
Wenn wir dies formal machen wollen, sollten wir daher zunächst die maximale Dichte
eines Teilgraphen definieren: Es sei Γ ein Graph. Wir setzen
e(H)
: H ⊆ Γ, v(H) > 0 .
m(Γ) = max
v(H)
Hierbei sind e(H) und v(H) die Kanten bzw. Knoten des Graphen H ⊆ Γ.
Satz 3.8 Sei Γ ein beliebiger Graph mit mindestens einer Kante. Dann gilt

1
 0, falls p ≪ n− m(Γ)
.
lim P(Γ ⊆ G(n, p)) =
1
n→∞
 1, falls p ≫ n− m(Γ)
1
Beweis: Wir beweisen′ die beiden Konvergenzen einzeln. Sei also zunächst p ≪ n− m(Γ) .
e(H )
Sei H ′ ⊆ Γ, so dass v(H
′ ) = m(Γ) gilt. Dann folgt aus (3.2), dass es asymptotisch f.s. keine
′
Kopien von H in G(n, p) gibt, also auch keine Kopie von Γ.
19
Um die umgekehrte Richtung zu beweisen, benutzen wir die Methode des zweiten Moments. Dazu brauchen wir eine obere Schranke für die Varianz von XΓ , die Anzahl der
Kopien von Γ in G(n, p), finden. Diese Abschätzung gleidern wir als Lemma aus. Setze
ΦΓ = ΦΓ (n, p) = min{EXΓ : H ⊆ Γ, eH > 0}.
Bemerke, dass
ΦΓ ≍
min
H⊆Γ,eH >0
nv(H) pe(H)
gilt.
Lemma 3.9 Sei Γ ein Graph mit wenigstens einer Kante. Dann gilt
X
n2vΓ −vH p2eΓ −eH
VXΓ ≍ (1 − p)
H⊆Γ
eH >0
(E(XΓ ))2
H⊆Γ,eH >0
EXH
2
(EXΓ )
.
= (1 − p)
ΦΓ
≍ (1 − p)
max
Hierbei hängen die Konstanten (wir schreiben an ≍ bn , falls es Konstanten c, C > 0 gibt
mit
can ≤ bn ≤ Can
für schließlich alle n) von Γ ab, aber nicht von n und von p ab. Insbesondere gilt
(EXΓ )2
VXΓ = O
ΦΓ
und, falls p von 1 weg beschränkt ist,
VXΓ ≍
(EXΓ )2
.
ΦΓ
Beweis: Falls für Γ′ , Γ′′ gilt, dass ihre Kantenmengen disjunkt sind, also
E(Γ′ ) ∩ E(Γ′′ ) = ∅,
dann sind die Indikatoren IΓ′ und IΓ′′ stochastisch unabhängig. Darüber hinaus gibt es
für jedes H ⊆ Γ
Θ(nvH n2(vΓ −vH ) ) = Θ(n2vΓ −vH )
Paare (Γ′ , Γ′′ ), von denen jedes eine Kopie von Γ ist und die in H überlappen. Also folgt
X
V(XΓ ) =
Cov(IΓ′ , IΓ′′ )
Γ′ ,Γ′′ ∼Γ
=
X
E(Γ′ )∩E(Γ′′ )6=∅
≈
≈
X
H⊆Γ,eH >0
X
H⊆Γ,eH >0
E(IΓ′ IΓ′′ ) − E(IΓ′ E(IΓ′′ )
n2vΓ −vH (p2eΓ −eH − p2eΓ )
n2vΓ −vH p2eΓ −eH (1 − p)
20
das letztere, weil der wesentliche Beitrag von H mit eH = 1 kommt.
2
Die folgende Beobachtung ist oft sehr nützlich:
Lemma 3.10 Sei Γ ein Graph mit eΓ > 0. Dann sind die folgenden Eigenschaften äquivalent:
(i) npm(Γ) → ∞;
(ii) nvH peH → ∞ für jedes H ⊆ Γ mit vH > 0;
(iii) E(XH ) → ∞ für jedes H ⊆ Γ mit vH > 0;
(iv) ΦΓ → ∞.
Beweis: Gemäß der Definition von m(Γ) und da p ≤ 1 gilt, ist (i) äquivalent zu
npeH /vH → ∞ für jedes H ⊆ Γ mit vH > 0.
Da
EXH ≈ nvH peH = (npeH /vH )vH ,
ist dies äquivalent zu (ii) und (iii). Schließlich ist nach Definition von ΦΓ (iv) äquivalent
zu
EXH → ∞ für jedes H ⊆ Γ mit eH > 0.
Dies ist äquivalent zu (iii), denn der Fall vH > 0 und eH = 0 ist trivial.
2
1
Nun können wir den Beweis von Satz 3.8 fertigstellen: Beachte, dass, falls p ≫ n− m(Γ) gilt,
nach Lemma 3.10 ΦΓ → ∞ gilt. Die Methode des zweiten Moments ergibt dann mithilfe
von Lemma 3.9
P(Γ 6⊆ G(n, p)) = P(XΓ = 0) ≤
1
V(XΓ )
) = o(1).
=
O(
(EXΓ )2
ΦΓ
2
Bemerkung 3.11 Aus dem obigen Beweis folgt auch, dass, falls
ΦΓ (n, p) → ∞,
dann gilt nicht nur
P(Γ 6⊆ G(n, p)) → 1,
sondern auch
XΓ
→ 1 in Wahrscheinlichkeit.
E(XΓ )
21
Bemerkung 3.12 Der obige Satz wurde in der hier gezeigten Form von Bollobas 1981
gezeigt. Schon 1960 bewiesen Erdös und Rényi dasselbe Resultat für balancierte Graphen,
das sind solche Graphen, für die m(Γ) = veΓΓ gilt.
Man kann sich nun fragen, wie schnell die Wahrscheinlichkeit, dass Γ 6⊆ G(n, p) im 2. Fall
von Satz 3.8 gegen 0 geht. Offenbar spielt die Größe ΦΓ dabei eine entscheidende Rolle.
Wir bereiten ein entsprechendes Resultat vor. Wichtig ist dabei eine Ungleichung, die aus
der mathematischen statistischen Mechanik stammt. Für einen Beweis verweisen wir auf
Grimmett/Stirzaker (1992, Problem 3.11.18b). Diese Ungleichung ist nach den Mathematikern und Physikern Fortuin, Kosteleyn und Ginibre als FKG-Ungleichung bekannt
geworden. Sei gehört zur Klasse der sogenannten Korrelationsungleichungen.
Hierzu betrachten wir {1, . . . , n} mit seiner Potenzmenge. Wir sagen, dass eine Funktion
f : P({1, . . . , n}) → R
wachsend ist, falls für A ⊂ B stets folgt f (A) ≤ f (B). f heißt fallend, falls aus A ⊆ B
folgt, dass f (A) ≥ f (B) gilt. Auf P({1, . . . , n}) konstruieren wir eine Wahrscheinlichkeit,
indem wir i mit Wahrscheinlichkeit pi an einer Teilmenge von {1, . . . , n} teilnehmen lassen
und mit Wahrscheinlichkeit 1 − pi nicht. Die so erhaltete zufällige Menge heißt Γp1 ,...,pn .
Dann gilt
Satz 3.13 (FKG-Ungleichung)
Falls X1 und X2 beide wachsende oder beide fallende Zufallsvariablen von Γp1 ,...,pn sind,
dann gilt
EX1 X2 ≥ EX1 EX2 ,
also Cov(X1, X2 ) ≥ 0. Insbesondere gilt für zwei wachsende (oder fallende) Familien von
Teilmengen von {1, . . . , n} Q1 und Q2
P(Γp1 ,...,pn ∈ Q1 ∩ Q2 ) ≥ P(Γp1 ,...,pn ∈ Q1 )P(Γp1,...,pn ∈ Q2 ).
Bemerkung 3.14 Eine wachsende Familie von Teilmengen Q ist ein System von Teilmengen, so dass aus A ∈ Q und A ⊆ B auch B ∈ Q folgt, 1lQ ist also im obigen Sinne
wachsend.
Analog definiert man fallend. Den zweiten Teil der Aussage von Satz 3.13 erhält man aus
dem ersten, wenn man für Xi = 1lQi betrachtet.
Eine wichtige Anwendung der FKG-Ungleichung erhält man ganz ähnlich: Sei S eine
Familie nicht-leerer Teilmengen von {1, . . . , n}. Für A ∈ S sei 1lA definiert als
1lA := 1l[A⊆Γp1 ,...,pn ] ,
wobei Γp1 ,...,pn wieder die zufällige Teilmenge von {1, . . . , n} ist, die man erhält, wenn
man i unabhängig von den anderen mit Wahrscheinlichkeit pi wählt. Offenbar ist jedes
1lA wachsend. Schließlich sei
X
X=
1lA ,
A∈S
d. h. X ist die Anzahl von A ∈ S, die in der zufälligen Menge Γp1 ,...,pn enthalten sind.
22
Korollar 3.15 Für X =
P
A∈S
1lA wie oben gilt
P(X = 0) ≥ exp −
EX
1 − maxi pi
.
Beweis: Man verwendet die FKG-Ungleichung folgendermaßen: Sei zu A ∈ S
A = {B ⊆ {1, . . . , n}, B ⊇ A}.
A ist wachsend. Für A, B ∈ S mit entsprechend definierten A und B gilt dann
P(IA + IB = 0) = P(Γp1 ,...,pn ∈ Ac ∩ Bc )
≥ P(Γp1 ,...,pn ∈ Ac )P(Γp1 ,...,pn ∈ Bc )
= (1 − E1lA )(1 − E1lB ).
Induktiv ergibt sich
P(X = 0) ≥
x
Y
A∈S
(1 − EIA ).
Wegen 1 − x ≥ e− 1−x und EIA ≤ max pi folgt
EX
P(X = 0) ≥ exp −
1 − maxA EIA
≥ exp −
EX
1 − maxi pi
.
2
Wir werden nun versuchen, eine ganz ähnliche obere Schranke herzuleiten. Dazu sei wieder
S ⊆ P({1, . . . , n}) und
X
X=
1lA .
(3.3)
A∈S
Es gilt dann
Satz 3.16 Für X wie in (3.3) und
λ = EX =
X
E1lA
und
A
¯ =
∆
X
E1lA 1lB ,
A,B
A∩B6=∅
sowie
ϕ(x) = (1 + x) log(1 + x) − x
gilt für 0 ≤ t ≤ EX
ϕ(− λt )λ2
t2
≤
exp(−
P(X ≤ EX − t) ≤ exp −
¯
¯ ).
∆
2∆
23
¯ auch die Diagonale A = B
Bemerkung 3.17
a) Beachte, dass die Definition von ∆
mit einschließt. Es ist manchmal zweckmäßig, diese separat zu betrachten; zu diesem
Zweck definieren wir
1 X
∆=
E1lA 1lB .
2 A6=B
A∩B6=∅
Damit ist ∆ dann die Summe über alle verschiedenen geordneten Paare und es
gilt
¯ = λ + 2∆.
∆
¯ ≥ λ. Gleichheit kann dabei nur gleten, wenn ∆ = 0
b) Offenbar gilt ∆ ≥ 0 und daher ∆
ist, d. h. wenn die Mengen A ∈ S disjunkt und damit die Indikatoren unabhängig
sind.
c) Eine entsprechende exponentielle obere Schranke für die unteren Enden P(x ≥
Ex + t) gibt es i. a. nicht, wie das folgende Beispiel zeigt. Sei λ ∈ N und Γ =
{0, 1, . . . , 2λ2}. Wir wählen nun Γp0 ,p1 ,...,p2λ2 aus Γ zufällig wie oben mit
p0 = λ−4 , pi = 1 − λ−4
pi = λ−1 − λ−4 + λ−8
1 ≤ i ≤ λ2
für
und
λ2 + 1 ≤ i ≤ 2λ2 .
für
S bestehe aus den Mengen
Ai =
(
{0, i} für 1 ≤ i ≤ λ2
für λ2 + 1 ≤ i ≤ 2λ2
{i}
.
Dann gilt
EX =
X
X
EIA =
A∈S
i∈{A1
λ2
=
X
λ
−4
+λ
−8
,...,2λ2 }
+
i=1
= λ2 ·
P(Γp0,...,p2λ2 ⊇ Ai
2
2λ
X
i=λ2 +1
1
= λ.
λ
λ−1 − λ−4 + λ−8
Außerdem gilt
∆ =
1 X
E1lA 1lB
2 A6=B
A∩B6=∅
1
=
2
=
<
1
2
X
E1lAi 1lAj
1≤i<j≤λ2
X
1≤i<j 1 ≤λ2
X
1≤i,j≤λ2
λ−4 (1 − λ−4 )2
λ−4 ≤ λ4 · λ−4 = 1.
24
Trotzdem gilt für c < ∞ und ε > 0, falls λ groß genug ist,
1
2
P(X > cλ) ≥ λ−4 (1 − λ−4 )λ ≥ λ−4 > exp(−ελ ).
2
Beweis von Satz 3.16: Sei
ψ(s) = Ee−sX ,
Zuerst zeigen wir, dass
woraus wir
s ≥ 0.
¯
−(log ψ(s))′ ≥ λe−s∆/λ ,
− log ψ(s) ≥
Z
0
s
s > 0,
λ2
¯
¯
λe−v∆/λ du = ¯ (1 − e−s∆/λ )
∆
folgern. Um (3.4) zu zeigen schreiben wir ψ ′ (s) als
X
−ψ ′ (s) = E[Xe−sX ] =
E[1lA e−sX ].
(3.4)
(3.5)
(3.6)
A
Für festes A ∈ S teilen wir X = YA + ZA auf, wobei
X
1lB .
YA =
B∩A6=∅
Wir wenden die FKG-Ungleichungen auf Γp1 ,...,pn bedingt auf 1lA = 1 an. Bedenken wir,
dass ZA und 1lA unabhängig sind und setzen pA := E1lA , so ergibt sich
E[1lA e−sX ] = pA E[e−sYA −sZA |1lA = 1]
≥ pA E[e−sYA |1lA = 1]E[e−sZA ]
(3.7)
−sYA
≥ pA E[e
|1lA = 1]ψ(s).
P
P
Nun ist λ = A∈S E1lA = A∈S pA , (3.6) und (3.7) zusammen ergeben mit der JensenUngleichung (einmal angewandt auf die bedingte Erwartung, einmal auf die Summe):
−ψ(s)′ X
≥
pA E[e−sYA |1lA = 1]
ψ(s)
A
X1
pA exp{−E(sYA |1lA = 1)}
≥ λ
λ
A
X1
≥ λ exp(−
pA E[sYA |1lA = 1])
λ
A
sX
E(YA 1lA ))
= λ exp(−
λ A
−(log ψ(s))′ =
¯
= λe−s∆/λ .
Somit gilt (3.4), also auch (3.5). Mithilfe der exponentiellen Markov-Ungleichung folgt
dann
λ2
¯
log P(X ≤ λ − t) ≤ log Ee−sX + s(λ − t) ≤ − ¯ (1 − e−s∆/λ ) + s(λ − t).
∆
25
Die rechte Seite wird für d = − log(1 − λt · ∆λ¯ minimiert, dies ergibt die erste Schranke.
Die zweite ergibt sich aus ϕ(x) ≥ x2 /2 für x ≤ 0 (Übung). Somit ist alles gezeigt.
2
Wenn wir in Satz 3.16 t = EX setzen, erhalten wir eine Abschätzung für die Wahrscheinlichkeit, dass überhaupt keine der Mengen aus S auftritt. Dies geben wir als eigenen Satz
an.
Satz 3.18 Sei wieder X =
P
A∈S
1lA , λ = EX und ∆ wie oben. Dann gilt:
(i) P(X = 0) ≤ exp(−λ + ∆);
2
λ
(ii) P(X = 0) ≤ exp(− 2(λ+2∆)
) = exp(− 2 P
λ2
A,B
A∩B6=∅
E1lA 1lB
).
Bemerkung 3.19 Beide Abschätzungen gelten für jedes λ und ∆, aber (i) ist langweilig,
wenn nicht ∆ < λ gilt. Tatsächlich ist (i) besser als (ii), wenn ∆ < λ2 , während (ii) für
größere ∆ besser ist (Übung).
Beweis von Satz 3.18: Wählt man in Satz 3.16 t = λ oder lässt in (3.5) direkt s → ∞
gehen und beachtet, dass
lim ψ(s) = P(X = 0),
s→∞
erhält man (ii).
(i) leiten wir aus dem Beweis von Satz 3.16 ab. Wir setzen dort
YA′ = YA − 1lA .
Damit ergibt sich
Z
∞
− log P(X = 0) = −
(log(ψ(s)))′ ds
Z ∞0 X
≥
pA E[e−sYA |1lA = 1]ds
=
0
A
X
pA E[
A
1
|1lA = 1].
YA
Ist 1lA = 1, so gilt
da YA′
1
1
1
≥ 1 − Y A′ ,
=
YA
Y A′ + 1
2
eine ganze Zahl ist, und daher
X
1
− log P(X = 0) ≥
pA (1 − YA′ |1lA = 1)
2
A
X
1
=
(pA − E(1lA YA′ ))
2
A
= λ − ∆.
26
2
Diese Abschätzungen wollen wir nun einsetzen, um Abschätzungen für die Wahrscheinlichkeit, dass ein Graph aus G(n, p) einen festen Graphen Γ nicht als Teilgraph enthält,
zu bekommen. Hierzu schauen wir uns den Beweis von Satz 3.8 noch einmal an. Hieraus
kann man ableiten, dass
1 − ΦΓ ≤ P(Γ 6⊆ G(n, p)) ≤ O(
1
).
ΦΓ
Das Schöne hieran ist, dass sich beide dieser Ungleichungen zu exponentiellen Ungleichungen verschärfen lassen.
Satz 3.20 Sei Γ ein Graph mit wenigstens einer Kante. Dann gilt für jede Folge p =
p(n) < 1
1
exp(−
φΓ ) ≤ P(Γ 6⊆ G(n, p)) ≤ exp(−Θ(ΦΓ )).
1−p
Beweis: Die linke der beiden Ungleichungen folgt aus Korollar 3.15. Hierbei setzen wir
in Korollar 3.15 X als XH ′ , die Anzahl von Kopien von H ′ in G(n, p), wobei H ′ ⊆ Γ der
Teilgraph mit der größten Dichte ist. Beachte, dass
E[XH ′ ] = ΦΓ .
Die andere Ungleichung folgt aus Satz 3.18 (ii). Hier setzen wir S = XΓ , die Anzahl der
Kopien von Γ in G(n, p) und 1lA = 1lΓ . Dann wird der Nenner des Exponenten zu
X XX
p2eΓ −eH = Θ((EXΓ )2 /ΦΓ ).
H⊆Γ
eH >0
Γ′
Γ′′
Daraus folgt die rechte Seite der Behauptung.
2
Wir schließen noch eine kleine Bemerkung über das Verhalten der Anzahl induzierter
Subgraphen an. Hierbei ist ein Graph Γ in G(n, p) induziert enthalten, wenn es vΓ
Knoten in G(n, p) gibt, dergestalt, dass alle Kanten, die in Γ enthalten sind, auch auf
diesen Kanten sind und alle Kanten, die nicht in Γ sind auch auf diesen Knoten nicht
vorhanden sind.
Für einen Graphen Γ sei YΓ die Anzahl der induzierten Kopien von Γ. Für festes p können
sich XΓ , die Zahl der Kopien von Γ und YΓ unterscheiden. Wir wollen aber sehen, dass
XΓ > 0 und YΓ > 0 dieselbe Schwellenfunktion haben. Der erste Teil von Satz 3.8 (die
1
Behauptung, dass für p ≪ n− m(Γ) YΓ = 0 ist) gilt sofort, denn es ist ja stets XΓ ≥ YΓ.
Für die andere Richtung sei JΓ′ eine 0-1-wertige Zufallsvariable, die für eine Kopie Γ′ von
Γ in Kn angibt, ob sie eine induzierte Kopie von Γ in G(n, p) ist (und dann 1 wird). Ist
p = o(1), so ist
vΓ
E[JΓ′ ] = peΓ (1 − p)( 2 )−eΓ ∼ E[1lΓ′ ].
27
Also gilt auch
E[XΓ ] ≈ E[YΓ ].
Um auch die Varianz abzuschätzen, bemerke, dass für zwei Kopien Γ′ und Γ′′ von Γ die
wenigstens eine gemeinsame Kante haben, gilt:
Cov(JΓ′ , JΓ′′ ) < E[JΓ′ JΓ′′ ]
≤ E[1lΓ′ 1lΓ′′ ]
≈ Cov(1lΓ′ , 1lΓ′′ ),
und wie für die 1lΓ′ gilt, dass für Γ′ , Γ′′ , die höchstens einen Knoten gemeinsam haben
Cov(JΓ′ , JΓ′′ ) = 0.
Schließlich gibt es (anders als für gewöhnliche Teilgraphen) noch einen dritten Fall: Γ′
und Γ′′ teilen sich t ≥ 2 Vertices, sind aber kantendisjunkt. Dann gilt
Cov(JΓ′ , JΓ′′ ) < E[JΓ′ , JΓ′′ ] < p2eΓ
und es gibt O(n2vΓ −t ) solcher Paare. Also gibt die Methode des zweiten Moments
P(YΓ = 0) ≤
≈
V(YΓ)
(EY )2
P Γ
Γ′ ,Γ′′
v(Γ′ )∩v(Γ′′ )6=∅
Cov(1lΓ , 1lΓ′ )
+
P
Γ′ ,Γ′′
v(Γ′ )∩v(Γ′′ )=∅
(EXΓ )2
Pv(Γ)−1 2v−t 2e
n
p Γ
≈ o(1) + t=2 2v 2e
= o(1).
n p Γ
Cov(JΓ′ , JΓ′′ )
E(YΓ )2
Somit haben in der Tat die Ereignisse {XΓ > 0} und {YΓ > 0} die gleichen Schwellenfunktionen. Das ist deshalb bemerkenswert, weil {YΓ > 0} keine monotone Eigenschaft
ist. In der Tat können ja sus Graphen mit mehr Kanten induzierte Subgraphen wieder
verschwinden. Es gibt daher auch noch eine zweite Schwellenfunktion für {YΓ > 0} für p
nahe bei 1, an der die induzierten Kopien von Γ wieder verschwinden.
28
4
Asymptotische Verteilungen
Nachdem wir uns im letzten Kapitel den Schwellenfunktionen gewidment haben, also einer
Art 0-1-Gestz für die Existenz von Teilgraphen, wollen wir nun ihre asymptotische Verteilung genauer studieren. Dies ist besonders interessant, wenn es überhaupt Teilgraphen
einer gewissen Gestalt gibt. Sind wir also an der Verteilung von XΓ interessiert (richtig
− 1
skaliert), so sollte zumindest p 6≪ b m(Γ) sein. Dabei sind zwei verschiedene Regime in1
, so sollte man zumindest für balancierte
teressant. Ist p von der Größenordnung n1 m(Γ)
Graphen Γ erwarten, dass es nicht zu viele Kopien von Γ in G(n, p) gibt, denn E(XΓ )
bleibt für wachsendes n endlich. Das Auftreten einer Kopie von Γ an einer festen Stelle
ist ein seltenes Ereignis, dies legt eine Poisson-Approximation für die Verteilung von XΓ
1
nahe. Ist hingegen p ≫ n− m(Γ) , so enthält G(n, p) viele Kopien von Γ und man könnΓ
te eventuell eine asymptotische Normalverteilung von X√Γ −EX
vermuten. Wir behandeln
VXΓ
beide Fälle separat, da sie auch methodisch Interessantes zu bieten haben.
Wir wollen für den Poisson-Ansatz etwas ausholen. Dieser wird mit der sogenannten
Steinschen Methode bewiesen, die in der 70er Jahren des 20. Jahrhunderts eingeführt
wurde, zunächst für die Approximation durch die Normalverteilung, später auch für
die Poissonverteilung, in der neuesten Forschung spielen auch andere Verteilungen eine wichtige Rolle. Wir haben eine Diät-Version der Steinschen Methode schon in der
Stochastik-Vorlesung gesehen. Nun stellen wir sie zunächst für die Poisson-Approximation
der Binomial-Verteilung vor. Wesentlich ist dabei stets eine charakteristische Gleichung
für eine Verteilung. Dies sieht bei der Poisson-Verteilung wie folgt aus: Sei g : N0 → R
eine beschränkte Abbildung und es sei Z eine Zufallsgröße, die P oi(λ)-verteilt ist. Dann
gilt
X
λj
Eλg(Z + 1) =
λg(j + 1) e−λ
j!
j≥0
und
E(Zg(Z)) =
X
l≥1
lg(l)
λl −λ X
λj
e =
λg(j + 1) e−λ .
l!
j!
j≥0
Zusammen ergibt dies
E[λg(Z + 1) − Zg(Z)] = 0
für alle beschränkten Abbildungen g : N0 → R und Z ∼ P oi(λ). Dies ist die sogenannte
Steinsche Gleichung (im Poisson-Fall auch Stein-Chen-Gleichung). Ist nun f : N0 → R
beschränkt mit
Ef (Z) = 0 (für Z ∼ P oi(λ)),
so lässt sich f schreiben as
f (j) = λgf,λ (j + 1) − jgf λ (j),
29
j ≥ 0.
Dabei ist die beschränkte Abbildung gf,λ definiert durch
j
j! X
gf,λ (j + 1) = j+1
πλ (k)eλ f (k)
λ
k=0
∞
j! X
πλ (k)eλ f (k)
= − j+1
λ
k=j+1
(4.1)
gf,λ (0) : = 0
(letztere Gleichung gilt, da
0 = Ef (Z) =
j
X
πλ (k)f (k) +
k=0
∞
X
πλ (k)f (k)
k=j+1
gilt und dies können wir natürlich mit jeder positiven Zahl, z. B.
j!
eλ
λj+1
multiplizieren).
Der Nachweis erfolgt induktiv: Für j = 0 ist
gf,λ (1) =
f (0)
f (0) λ
=
e πλ (0)
λ
λ
und
gf,λ (j + 1) = (f (j) + jgf,λ (j))
1
λ
j−1
1
j (j − 1)! X
1
πλ (k)eλ f (k)
f (j)πλ (j)
+
=
λ
πλ (j) λ λj
k=0
j−1
j!
j! X
λ
=
πλ (j)eλ f (k).
f
(j)π
(j)e
+
λ
λj+1
λj+1 k=0
Die Beschränktkeit von gf,λ folgt mit den Bezeichnungen
kgf,λ k := sup |gf,λ (j)|
und
j≥0
kf k := sup |f (j)|
j
aus der eben gezeigten Darstellung:
∞
X
λk−(j+1)
kgf,λ k ≤ kf k
j! ≤ kf keλ .
k!
k=j+1
Wir haben also gezeigt
Lemma 4.1 Sei f eine beschränkte Funktion von N0 nach R. Dann ex. eine beschränkte
Lösung gf,λ : N0 → R von
λgf,λ (j + 1) − jgf,λ (j) = f (j),
j≥0
genau dann, wenn
Ef (Z) = 0
mit Z ∼ P oi(λ).
30
Bemerkung 4.2
(i) Ist Z ∼ P oi(λ) und f : N → R mit Ef (Z) 6= 0, so folgt
|gf,λ(j)| → ∞ für
j→∞
(Übung).
(ii) Es sei X eine Zufallsvariable mit Werten in N0 . Dann gilt sogar: X ist P oi(λ)verteilt genau dann, wenn für jede beschränkte Funktion g : N0 → R gilt
E[λg(X + 1) − Xg(X)] = 0.
Diese Gleichung charakterisiert also die Poisson-Verteilung zum Parameter λ. Dies
ist schnell einzusehen. Betrachte
−λ
f (j) := h(j) − e
∞
X
h(k)
k=0
λk
k!
für jedes j ∈ N0 . Hierbei sei h : N0 → R eine beschränkte Funktion. f ist beschränkt,
z. B. durch 2khk und erfüllt
Ef (Z) = 0 für
Z ∼ P oi(λ)
(so ist f gerade gebaut). Nach Lemma 4.1 gibt es eine beschränkte Funktion gf,λ mit
0 = E[λgf,λ (X + 1) − Xgf,λ (X)] = E[h(X) − e−λ
∞
X
h(k)
k=0
λk
].
k!
Wählen wir speziell
hA (j) := 1l{j∈A}
für A ⊆ N0 , so folgt aus der rechten Gleichung
P(X ∈ A) − P oi(λ)(A) = 0
(mit P oi(λ)(A) =
P
j∈A
∀ A ⊆ N0
πλ (j)). Also ist X ∼ P oi(λ).
Schreibe nun gf := gf,λ und es sei Z ∼ P oi(λ). Weiter sei A ⊆ N0 und
fA (j) := 1l{j∈A} − P oi(λ)(A), j ≥ 0.
Offenbar ist fA beschränkt und
EfA (Z) = 0 ∀ A ≤ N0 .
Nach Lemma 4.1 existiert also eine (beschränkte) Lösung der Gleichung
λgA,λ(j + 1) − jgA,λ (j) = 1l{j∈A} − P oi(λ)(A)
für j ≥ 0 und jedes A ⊆ N0 . Mit der Bezeichnung
Um := {0, 1, . . . , m}
31
(4.2)
folgt aus der allgemeinen Form der Lösung (4.1) für gf,λ :
gA,λ(j + 1) =
j! λ
e (P oi(λ)(A ∩ Uj ) − P oi(λ)(A)P oi(λ)(Uj )).
λj+1
(4.3)
Dies ist die sogenannte Stein-Lösung. Wir setzen (ohne Einschränkung) gA,λ (0) = 0.
Den Nutzen der Stein-Gleichungen (4.2) und ihrer Lösung (4.3) sieht man schnell ein:
Ersetzen wir j ∈ N0 durch eine Zufallsvariable W mit Werten in N0 , so folgt aus (4.2)
durch Bilden des Erwartungswertes
E[λgA,λ (W + 1) − W gA,λ(W )] = P(W ∈ A) − P oi(λ)(A).
(4.4)
Da wir uns für Poisson-Approximationen interessieren, wollen wir die rechte Seite gleichmäßig in A abschätzen. Nach (4.4) können wir dies ebenso gut in der linken Seite von
(4.4) tun. Wir werden in der Folge sehen, dass sich dies recht gut bewerkstelligen lässt
und dass die Schranken, die wir erhalten, keine a priori-Kenntnis über die Verteilung von
W erfordern. Allgemein besteht die Steinsche Methode aus den Schritten:
(i) Stelle eine für die potenzielle Limesverteilung charakteristische Gleichung auf und
löse diese. Untersuche die Lösungen.
(ii) Setze die zu approximierende Zufallsvariable W in die Lösung ein und schätze ab.
Wir führen dies nun am Beispiel der Poisson-Verteiung aus. Es seien I1 , . . . , In unabhängige Zufallsvariablen mit
P(Ij = 1) = pj = 1 − P(Ij = 0) 0 < pj < 1 1 ≤ j ≤ n.
Sei ferner
W =
n
X
Ij :
λ := EW =
j=1
und
n
X
pj
j=1
Wi =
n
X
Ij .
j=1
j6=i
Schreibe wieder gA := gA,λ . Es folgt nun
E(Ii gA (W )) = E[Ii gA (Wi + 1] = pi EgA (Wi + 1),
denn definitionsgemäß ist Ii stochastisch unabhängig von Wi . Also ist
E[λgA (W + 1) − W gA (W )] =
n
X
i=1
pi (E(gA (W + 1) − E(gA (Wi + 1))).
Nun ist W = Wi , es sei denn Ii = 1, was mit Wahrscheinlichkeit pi eintritt. Also folgt:
|P(W ∈ A) − P oi(λ)(A)| ≤ 2 sup |gA (j)|
j≥0
32
n
X
i=1
p2i
bzw.
|P(W ∈ A) − P oi(λ)(A)| ≤ sup |gA (j + 1) − gA (j)|
j
n
X
p2i .
i=1
Dies führt zu einer Abschätzung der Güte der Poisson-Approximation im unabhängigen
Fall, falls wir
kgk := kgA,λk := sup |gA,λ(j)|
oder
j
∆g := ∆gA,λ := sup |gA,λ(j + 1) − gA,λ (j)|
j
gleichmäßig in A ⊆ N0 beschränken können. Wir entscheiden uns für ∆g: Sei Z ∼ P oi(λ).
Dann ist
X
fA (j) =
(1lj=k − P(Z = k)).
k∈A
Sei nun gk die Lösung der Stein-Gleichung zu
fk (j) := 1l{j=k} − P(Z = k), j ≥ 0.
Es sollte dann
gA (j) =
X
gk (j)
(4.5)
k∈A
gelten, denn bei endlich vielen Summanden folgt dies sofort aus der Linearität der SteinGleichung. Um (4.5) einzusehen, sei A ⊆ N0 beliebig. Wir führen eine Abschneidetechnik
ein, um (4.5) auf den endlichen Fall zu reduzieren. Es ist für ein M > 0
X
fA =
fk + fA∩{k:k>M }.
k∈A
k≤M
Sei
{> M} := {k : k > M}.
Da wir fA als endliche Summe dargestellt haben, gilt für die zugehörige Stein-Lösung:
X
gk + gA∩{>M } .
gA =
k∈A
k≤M
Nun ist definitionsgemäß
fA∩{>M } (j) = −P(Z ∈ A ∩ {> M})
für alle j ≤ M. Es sei j < M, dann gilt für die Stein-Lösung gemäß (4.3)
−P(Z ∈ A ∩ {> M})
j
j(j − 1)
j!
gA∩{>M } (j + 1) =
1+ +
+ ...+ j ,
λ
λ
λ2
λ
also
|gA (j + 1) −
X
k∈A
k≤M
g(j + 1)| ≤ const(λj )P(Z ∈ A ∩ {> M}) ≤ const(λj )P(Z > M).
33
Die rechte Seite konvergiert für M → ∞ gegen 0, also folgt (4.5). Aus (4.1) erhalten wir
nun
1
j
j(j − 1)
j!
gk (j + 1) = (fk (j) + fk (j − 1) +
fk (j − 2) + . . . + j fk (0)).
2
λ
k
λ
λ
Nach Definition ist:


 −P(Z = k) j < k
fk (j) =
1 − P(Z = k) j = k .

 −P(Z = k) j > k
Also folgt für j < k
gk (j + 1) = −
j
j!
P(Z = k)
(1 + + . . . + j ) ≤ 0.
λ
λ
λ
Daher ist
gk (j + 1) − gk (j) ≤ 0,
also gk monoton fallend. Sei nun j > k. Mit der Stein-Lösung (4.3) ist
gk (j + 1) = eλ j!λ−j−1(πλ (k)1lk≤j − πλ (k)P oi(λ)(Uj ))
= eλ j!λ−j−1πλ (k)(1 − P oi(λ)(Uj ))
≥ 0.
Verwenden wir diese Darstellung und k ≤ j − 1 (denn k < j), so folgt
∞
∞
j X λl X λl
−
λ l=j+1 l!
l!
l=j
!
∞
∞
X
1
λl X λl
≤
(j − 1)!πλ (k)
−
λ
l!
l!
1
(j − 1)!πλ (k)
gk (j + 1) − gk (j) =
λ
l=j
!
l=j
= 0.
Somit ist gk auch in diesem Bereich monoton fallend, womit nur der Zuwachs gk (k + 1) −
gk (k) positiv ist. Hierfür gilt:
∞
k−1 j
X
X
λj
λ
(k − 1)!
k!
P(Z = k)(
)+
P(Z = k)(
)
gk (k + 1) − gk (k) =
k+1
k
λ
j!
λ
j!
j=0
j=k+1
−λ
= e
∞
k−1
1 X λj
1 X λj
+
)
(
λ j=k+1 j!
k j=0 j!
≤ e−λ (
−λ
∞
k
1 X λj
1 X λj
+
)
λ
j!
λ j=1 j!
j=k+1
e
(eλ − 1)
λ
1
(1 − e−λ )
=
λ
1
≤ min(1, ).
λ
=
34
Hierbei ist die Ungleichung in Zeile 3 für k = 1 eine Gleichheit. Aus dem soeben Gezeigten
folgt mit (4.5)
X
(gk (j + 1) − gk (j));
gA (j + 1) − gA (j) =
k∈A
hier sind alle Summanden negativ, bis auf den Fall j = k. Also ist
gA (j + 1) − gA (j) ≤ gj (j + 1) − gj (j),
falls j ∈ A, sonst ist gA (j + 1) − gA (j) ≤ 0. Es gilt also
gA (j + 1) − gA (j) ≤
1
(1 − e−λ )
λ
für alle A ⊆ N und alle j > 0. Wenn
gA (j + 1) − gA (j) ≥ 0 ∀ j ≥ 0,
so folgt
∆g ≤
1
(1 − e−λ ).
λ
Im Fall gA (j + 1) − gA (j) < 0 ist
0 < −gA (j + 1) + gA (j) = gAc (j + 1) − gAc (j) ≤
1
(1 − e−λ ),
λ
denn
fA + fAc = fZ+ ≡ 0.
Somit folgt für die zugehörigen Stein-Lösungen
gA = −gAc .
Wir haben somit gezeigt
Satz 4.3 Für die Zuwächse der Stein-Lösungen gilt gleichmäßig in A ⊆ N0 :
∆g ≤
1
1
(1 − e−λ ) ≤ min(1, ).
λ
λ
Bemerkung 4.4 Ist A = {1}, so ist die erste Schranke exakt
∆g =
1
(1 − e−λ ).
λ
Wir bekommen so auch eine Schranke für die Poisson-Approximation für eine Summe von
Ber(pi ) verteilten unabhängigen Zufallsvariablen.
35
Satz 4.5 (Barbour, Hell 1984)
Pn
Pn
I
.
Es
sei
λ
=
Seien
I
Ber(p
)-verteilt
und
unabhängig
und
es
sei
W
=
i
i
i
i=1 EIi =
i=1
Pn
i=1 pi , dann gilt
n
X
1
−λ
dT V (L(W ), P oi(λ)) ≤ (1 − e )
p2 .
λ
i=1
Hierbei bezeichnet L(W ) die Verteilung von W und dT V ist der Abstand der totalen Variation zweier Maße µ, ν auf demselben Raum (Ω, F ):
dT V (µ, ν) =
1
sup |µ(A) − ν(A)|.
2 A∈F
Wir wollen nun versuchen an der Schranke, bei der überhaupt Graphen einer gewissen
Gestalt Γ auftreten, in G(n, p) eine Poisson-Approximation für diese Anzahl zu liefern.
− 1
Wir haben im vorhergehenden Kapitel gesehen, dass diese Schwelle bei p m(Γ) liegt, weil
oberhalb dieser Schwelle garantiert ist, dass auch der dichteste Teilgraph von Γ auftritt.
1
Allerdings kann es geschehen, dass wenn Γ nicht selbst dieser Teilgraph ist, bei p− m(Γ)
“explosionsartig” viele Kopien von Γ auftreten, weil wir zu jeder Kopie von H ⊆ Γ, der
dichtesten Teilmenge von Γ, “gratis” alle Anhängsel bekommen. Daher werden wir uns
auf Graphen konzentrieren, die keine dichteren Teilgraphen besitzen.
Definition 4.6 Ein Graph Γ heißt strikt balanciert, falls
m(Γ) =
und für alle H ⊆ G gilt
eH
vH
eΓ
vΓ
< m(Γ).
Anders als in der vorhergehenden Diskussion betrachten wir nun zwar wieder eine Zufallsvariable W der Gestalt
X
W =
Iα ,
α∈Ξ
wobei Iα Indikatoren sind, diese sind aber nicht mehr unabhängig. Wir stellen eine von
mehreren Ansätzen vor, um dieses Problem zu behandeln. Sei also W wie oben. Zu jedem
α ∈ Ξ (der Indexmenge) sei Ξα definiert als
Ξα := {β ∈ Ξ : β 6= α, Iβ und Iα sind stochastisch abhängig}.
Setze weiterhin
Zα :=
X
Iβ
und Wα′ =
X
Iβ .
β∈Ξc
α
β6=α
β∈Ξα
Also ist für jedes α
W = Iα + Zα + Wα′ .
Weiter sei
πα := EIα
und λ := EW =
X
α∈Ξ
Dann gilt
36
πα .
Satz 4.7 Mit obiger Notation gilt
XX
1 X 2 X
EIα Iβ ).
dT V (L(W )), P oi(λ)) ≤ min(1, )(
πα +
πα EZα +
λ α
α
α β∈Ξ
α
Bemerkung 4.8 Offenbar
P P ist diese Schranke qualitativ “gut”, so lange die πα klein und
die Abhängigkeiten α β∈Ξα EIα Iβ klein sind.
Beweis: Nach dem eingangs gesagten gilt für alle A ⊆ N0
P(W ∈ A) − P oi(λ)(A) = E(λgA (W + 1) − W gA (W )),
wobei gA = gA,λ die Stein-Lösung (4.3) ist. Wir bestimmen wieder für jedes A ⊆ N0 die
rechte Seite der obigen Gleichung. Zunächst berechnen wir E[Iα gA (W )]. Es gilt offenbar
Iα gA (W ) = Iα gA (Zα + Wα′ + 1),
da Ia ∈ {0, 1}. Wir schreiben die rechte Seite als
Iα gA (Wα′ + 1) + Iα (gA (Zα + Wα′ + 1) − gA (Wα′ + 1)).
Somit folgt ähnlich wie eben für die Zuwächste von gA
|Iα gA (W ) − Iα gA (Wα′ + 1)| ≤ Iα Zα ∆g
bzw. nach Bildung des Erwartungswertes
|E(Iα gA (W )) − E(Iα gA (Wα′ + 1))| ≤ E(Iα Zα )∆g.
Nun ist nach Definition Wα′ unabhängig von Iα , also ist
E(Iα gA (Wα′ + 1)) = πα E(gA (Wα′ + 1)).
Summiert man über α ∈ Ξ, so folgt
X
XX
EIα Iβ .
|E(W gA (W )) −
πα E(gA (Wα′ + 1))| ≤ ∆g
α∈Ξ
α∈Ξ β∈Ξα
Weiter gilt
E(λgA (W + 1) =
X
πα E(gA (W + 1))
α∈Ξ
und mit W − Wα′ = Iα + Zα folgt
|gA (W + 1) − gA (Wα′ + 1)| ≤ ∆g(Iα + Zα ).
Also
|E(λgA (W + 1)) −
X
α∈Ξ
πα E(gA (Wα′ + 1))| ≤ ∆g
= ∆g
X
α∈Ξ
X
α∈Ξ
37
πα E[Iα + Zα ]
(πα2 + πα EZα ).
Zusammen folgt somit unsere Behauptung durch Anwendung der Dreiecksungleichung:
|P(W ∈ A) − P oi(λ)(A)| = |E(λgA (W + 1) − W gA (W )|
X
X
XX
EIα Iβ ).
≤ ∆g(
πα2 +
πα EZα +
α∈Ξ
α∈Ξ
α∈Ξ β∈Ξα
Die obige Abschätzung von ∆g vervollständigen den Beweis.
2
Bemerkung 4.9PFür unabhängige
Zufallsvariable ist für jedes α ∈ Ξ die Menge Ξα leer,
P
also Zα = 0 und α∈Ξ β∈Ξα EIα Iβ = 0, also folgt
|P(W ∈ A) − P oi(λ)(A)| ≤ ∆g
X
α
1 X 2
πα2 ≤ min(1, )
π ,
λ α α
also die Schranke aus Satz 4.5.
Beispiel 4.10 (k-Runs)
Es seien X1 , . . . , Xn iid Zufallsvariablen mit
P(Xi = 1) = p = 1 − P(Xi = 0) i = 1, . . . , n.
Sei k ∈ N fix. Sei für α ∈ {1, . . . , n}
α+k−1 mod(n)
Iα =
Y
Xi
i=α
und
W =
X
Iα .
α
Somit beschreibt W die Anzahl der 1-Runs der Länge k unter der Xi (wenn wir sie
kreisförmig anordnen). Natürlich sind die Iα abhängig, wenn die zugehörigen Indexmengen
überlappen. Es gilt
EIα = pk =: π, also EW = npk .
Zu α ∈ {1, . . . , n} sei
Ξα = {α − (k − 1), . . . , α − 1, α + 1, . . . , α + (k − 1)},
wobei die Additionen und Subtraktionen wieder “modulo n” zu verstehen sind. Wir wollen
nun mittels Satz 4.5 einsehen, dass für kleine p eine Poisson-Approximation für W gültig
ist. Es ist
X
X
(πα2 + πα EZα ) =
π 2 + π 2 (2k − 2) = nπ 2 (2k − 1).
α
α
Weiter rechnet man nach, dass
XX
α
EIα Iβ = 2nπ
k−1
X
i=1
β∈Ξα
38
pi
gilt. Beispielsweise ist für β = α + 1
EIα Iβ = P(Iα = Iβ = 1) = pk · p = πp
oder für β = α + 2
EIα Iβ = P(Iα = Iβ = 1) = pk p2 = πp2 .
Setzen wir λ := EW = nπ und wenden Satz 4.5 an, so ergibt sich
k−1
X
1
dT V (L(W ), P oi(λ)) ≤
(1 − e−λ )((2k − 1)kπ 2 + 2πn
pi )
nπ
i=1
−λ
= (1 − e
k
)((2k − 1)p + 2
= (1 − e−λ )((2k − 1)pk +
= O(p).
k−1
X
pi )
i=1
2p(1 − pk−1 )
)
1−p
Wendet man eine Approximation durch eine zusammengesetzte Poisson-Approximation
an, so lässt sich die Güte der Approximation noch verbessern; darauf soll hier aber nicht
eingegangen werden.
Wir wollen nun Satz 4.8 verwenden, um eine Poisson-Approximation für die Anzahl der
Teilgraphen, die isomorph zu einem gegebenen, strikt balancierten Graphen sind, abzuleiten. Wir zeigen:
Satz 4.11 (Barbour, 1982)
Sei H strikt balanciert mit vH = k und eH = l ≥ 1. Ist p(n) so, dass
lim p(n)nk/l = c > 0 gilt,
n→∞
so folgt für
λ=
cl
|aut(H)|
(wobei |aut(H)| die Größe der Automorphismengruppe von H ist) und
X
W =
IH = XH
die Konvergenz
lim dT V (L(W ), P oi(λ)) = 0.
n→∞
Zunächst überlegen wir, dass λ die richtige Größe hat. Es gibt auf Kn
n
k!|aut(H)|
k
39
(|aut H| sei die Größe der Automorphismengruppe von H) Kopien von H. Jede hat die
Wahrscheinlichkeit pl aufzutauchen in G(n, p). Also ist
l n
k!/|aut(H)| ≈ nk pl /|aut(H)|
EW = EXH = p
k
cl
k/l l
= (pn ) /|aut(H)| ≈
= λ.
|aut(H)|
Wir führen noch ein wenig Notation ein: Sei H wie oben strikt balanciert. Dann sei
(i) mt (H) = max{ρ(F ) :=
e(F )
v(F )
: F ⊂ H : |v(F )| = t}; offenbar gilt mt (H) < ρ(H) für
6=
1 ≤ t ≤ v(H).
(ii) it (H) = ρ(H) − mt (H) für 1 ≤ t ≤ k. Es gilt i1 (H) = ρ(H), da m1 (H) = 0 gilt.
(iii) ε = ε(H) := min1≤t≤k it (H) heißt der Balance-Index. Es gilt
ε ≤ i1 (H) = ρ(H) und ε > 0.
Lemma 4.12 Sei H strikt balanciert, Γ beliebig mit H 6⊆ Γ und |V (Γ ∩ H)| ≥ 1. Dann
gilt
|E(H ∪ Γ)| ≥ |E(H)| + |E(Γ)| − ρ(H)|V (H ∩ Γ)| + ε.
Bemerkung 4.13 Dieses Lemma ist für Γ mit Γ ⊇ H nicht wahr, denn dann ist
|E(H)| + |E(Γ)| − ρ(H)|V (H ∩ Γ)| = |E(H)| + |E(Γ)| − |E(H)| = |E(H ∪ Γ)|.
Beweis von Lemma 4.12: Es ist
|E(H ∩ Γ)| =
≤
=
≤
ρ(H ∩ Γ)|V (H ∩ Γ)|
m|V (H∩Γ)| |V (H ∩ Γ)|
(ρ(H) − i|V (H∩Γ)| )|V (H ∩ Γ)|
ρ(H)|V (H ∩ Γ)| − ε.
Wegen
folgt die Behauptung.
|E(H ∪ Γ)| = |E(H)| + |E(Γ)| − |E(H ∩ Γ)|
2
Lemma 4.14 Sei H strikt balanciert und H1 6= H2 Kopien von H in Kn mit
|V (H1 ∩ H2 )| > 0.
Sei F := H1 ∪ H2 . Dann gilt
|E(F )| ≥ ρ(H)|V (F )| + ε.
40
Beweis: Wir setzen in Lemma 4.12 H := H1 und Γ := Hi . Dann ist nach Voraussetzung
H1 ⊆ H2 . Also folgt aus Lemma 4.12
|E(F )| =
≥
=
=
|E(H1 ∪ H2 |
2|E(H)| − ρ(H)|V (H1 ∩ H2 )| + ε
2|E(H)| − ρ(H)(2|V (H)| − V (H1 ∪ H2 )| + ε
ρ(H)|V (H1 ∪ H2 )| + ε.
2
Beweis von Satz 4.11: Wähle Indizes α, β so, dass
|V (α ∩ β)| = s
gilt. Also α ∪ β genau 2k − s Knoten. Es gilt dann
E(Iα Iβ ) ≤ pρ(H)(2k−s)+ε = p2|E(H)|−sρ(H)+ε
(da 0 < p < 1) und somit
XX
α∈Ξ β∈Ξα
k X
sl
n−k
k
k!p2l− k +ε
E(Iα Iβ ) ≤ |Ξ|
k−s
s
s=2
= O(
= O(
k
X
sl
nk nk−s p2l− k +ε )
s=2
k
X
(npl/k )2k−s pε )
s=2
ε
= O(p (npl/k )2k−2 )
= O(pε ).
Satz 4.7 liefert daher
dT V (L(W ), P oi(λ)) ≤ O(pl ) + O(n−2 ) + O(pε ),
denn mit πα = EIα = pl und
Zα :=
X
Iβ =
k
X
X
Iβ
s=2 |V (α∩β)|=s
β∈Ξα
folgt
X
α∈Ξ
πα2
2
= |Ξ|p =
n
k
k! 2l
p = λpl = O(pl )
|aut(H)|
41
und
EZα ≤
k X
k n−k
s=2
= O(
k−s
s
k
X
pl k!
pl nk−s )
s=2
= O(nk−2pl )
= O(n−2).
2
Wir wollen uns nun dem Fall zuwenden, in dem wir wieder die Graphen in G(n, p) zählen,
die zu einem gegebenen blau?? Graphen Γ isomorph sind, aber in dem
1
p ≫ n− m(Γ)
gilt. Wie schon eingangs dieses Kapitels erwähnt, sollte man hier eine asymptotische
Normalverteilung von XΓ (richtig normiert) erwarten. Es gibt mehrere Techniken, dieses
Resultat zu zeigen. Wir verwenden hier ebenfalls die Steinsche Methode, diesmal für die
Normalverteilung. Wir stellen auch diese Methode zunächst für Folgen von i.i.d. Zufallsvariablen vor. Wir zeigen also
Satz 4.15 Es sei (Xn ) eine Folge von i.i.d. Zufallsvariablen mit EXn = 0 und V(Xn ) > 0
für alle n. Sei
n
X
2
Xi .
σ = V(Xn ) und Sn =
i=1
Dann gilt
L
S
√ n
nσ 2
⇒ N (0, 1).
Im Vergleich zum (bekannten) Beweis über Fourier-Transformierte hat der Beweis über
die Steinsche Methode den Vorteil, auch eine Konvergenzgeschwindigkeit zu liefern, etwa
in dem Sinn von
Satz 4.16 (Berry, Esséen 1945)
In der Situation von Satz 4.15 sei zusätzlich ξ := E(X13 ) < +∞. Dann gilt
kξ
Sn
≤ t − Φ(t)| ≤ 3 √ ,
sup |P √
2
σ n
t∈R
nσ
wobei Φ die Verteilungsfunktion der N (0, 1)-Verteilung ist.
Wie im Poisson-Fall beginnen wir mit einer Charakterisierung der Grenzverteilung.
42
Lemma 4.17 Es sei Z eine reellwertige Zufallsvariable. Z ist N (0, 1)-verteilt genau
dann, wenn für jede stetige, stückweise differenzierbare Funktion f : R → R mit
Z
2
|f ′ (x)|e−x /2 dx < +∞
(4.6)
gilt
E[f ′ (Z) − Zf (Z)] = 0.
Beweis: Sei Z ∼ N (0, 1) und es gelte (4.6). Mittels partieller Integration folgt
Z
1
2
′
f ′ (w)e−w /2 dw
Ef (Z) = √
2π ZR
1
2
= √
f (z)ze−z /2 dz
2π R
= E(Zf (Z)).
Nun zeigen wir die umgekehrte Richtung: Es gelte für jede reelle, stetige, stückweise
differenzierbare Funktion mit (4.6)
E[f ′ (Z) − Zf (Z)] = 0.
Dann können wir dies insbesondere für
Z y
2
y 2 /2
fω0 (y) := e
(hω0 (x) − N (hω0 ))e−x /2 dx
−∞
mit
hω0 (x) :=
und
1
N (hω0 ) := √
2π
anwenden. Es gilt
fω′ 0 (y)
y 2 /2
= e
(
Z
1 für x ≤ ω0
0 für x > ω0
hω0 (x)e−x
−y 2 /2
(hω0 (y) − N (hω0 ))e
y 2 /2
+ ye
= hω0 (y) − N (hω0 ) + yfω0 (y).
2 /2
dx = Φ(ω0 )
Z
y
−∞
(hω0 (x) − N (hω0 ))e−x
Also löst fω0 die Differentialgleichung
f ′ − yf = hω0 − Φ(ω0 ).
Somit folgt
0 = E[fω′ 0 (Z) − Zfω0 (Z)]
= E[hω0 (Z) − N (hω0 )]
= P(Z ≤ ω0 ) − Φ(ω0 ).
43
2 /2
dx
Bleibt zu zeigen, dass fω0 stetig und stückweise differenzierbar ist und
Z
2
|fω′ 0 (x)|e−x /2 dx < +∞
R
gilt. Da
fω′ 0
existiert, ist fω0 stetig. Da allgemein h − N (h) und mit
Z y
2
y 2 /2
UN (h)(y) := e
(h(x) − N (h))e−x /2 dx
−∞
auch yUN (h)(y) stückweise stetige Funktionen sind, ist UN (h)(·) stückweise stetig differenzierbar, also insbesondere fω0 . Zeigen wir noch, dass
Z
1
2
√
|UN′ (h)(x)|e−x /2 dx < +∞,
2π R
ist das Lemma bewiesen. Nach Voraussetzung gilt
Z
2
|h(x) − N (h)|e−x /2 dx < +∞.
Nach Definition von N (h) gilt ferner
Z
2
(h(x)) − N (h)e−2 /2 dx = 0
R
und daher
ω 2 /2
Z
ω
2
(h(x) − N /h))e−x /2 dx
−∞
Z ∞
2
2
+ω /2
= −e
(h(x) − N (h))e−x /2 dx.
UN (h)(ω) = e
ω
Daraus leiten wir mittels partieller Integration ab:
Z ∞
Z ∞ Z ∞
2
−x2 /2
|xUN (h)(x)|e
dx ≤
x(
|h(y) − N (h)|e−y /2 dy)dx
0
x
Z0 ∞
y2
2
=
|h(y) − N (h)|e−y /2 dy.
2
0
Dieses Integral ist für h = hω0 endlich. Analog schätzt man das Integral über Rn ab. Dies
ergibt die Behauptung.
2
Die Idee, die wir aus dem letzten Lemma gewinnen ist: Wenn wir
E[f ′ (W ) − W f (W )]
für eine große Klasse von Funktionen f durch eine kleine Zahl kontrollieren können, so
ist L(W ) dicht bei N (0, 1). Die obigen Rechnungen zeigen zudem, dass für glatte h die
Steingleichung
f ′ (x) − xf (x) = h(x) − N (h)
durch UN (h)(x) gelöst wird und dass außerdem gilt:
E[UN+ (h)′ (W ) − W UN (h)(W )] = Eh(W ) − N (h).
Wieder besteht die Idee darin, die rechte Seite dieser Gleichung zu kontrolieren, um die
linke Seite unter Kontrolle zu halten. Hierzu zeigen wir
44
Lemma 4.18 Ist h : R → R beschränkt und differenzierbar und UN (h) wie oben
Z y
2
y 2 /2
UN (h)(y) = e
(h(x) − N (h))e−x /2 dx
−∞
die Lösung der Stein-Gleichung. Dann gilt
r
π
kUN (h)k ≤
kh − N (h)k
2
(4.7)
kUN (h)′ k ≤ 2kh − N (h)k
(4.8)
kUN (h)′′ k ≤ 2kh′ k.
(4.9)
und
Beweis: Wie schon oben bemerkt ist
w
Z
w 2 /2
2
(h(x) − N (h))e−x /2 dx
−∞
Z ∞
2
2
+w /2
= −e
(h(x) − N (h))e−x /2 dx.
UN (h)(w) = e
w
Für (4.7) beachte, dass für w ≤ 0
|UN (h)(w)| ≤ sup |h(x) − N (h))(
x≤0
und für w ≥ 0
Z
w
w 2 /2
|UN (h)(w)| ≤ sup |h(x) − N (h)|e
d w2 /2
(e
dw
und
Z
w
−x2 /2
e
folgt
w 2 /2
dx) = 1 + we
−∞
1
Φ(w) ≤ √
2π
2 /2
dy)ew
2 /2
−∞
x≥0
Da
e−y
Z
∞
e−y
2 /2
dy.
w
Z
w
e−x
2 /2
dx
−∞
Z
2
w
e−w /2
x
2
√
− e−x /2 dx =
|w|
|w| 2π
−∞
d w2 /2
(e
dw
Z
w
e−x
2 /2
dx) > 0.
−∞
Somit wird das Maximum von |UN (h)(w)| bei w = 0 angenommen. Hier ergibt sich
r
π
kUN (h)k ≤
kh − N (h)k.
2
Für (4.8) bemerken wir, dass die Steingleichung für UN (h) für w ≥ 0 impliziert:
Z ∞
2
′
w 2 /2
e−x /2 dx).
sup |(UN (h) (w)| ≤ kh − N (h)k(1 + sup we
w≥0
w≥0
45
w
Da
1
1 − Φ(w) ≤ √
2π
folgt
Z
∞
w
2
e−w /2
x −x2 /2
e
dx = √
w
w 2π
sup |(UN (h)′ (w)| ≤ 2kh − N (h)k.
w≥0
∗
Setzen wir h (w) = h(−w), so gilt
UN (h∗ )(w) = UN (h)(−w),
daraus folgt (4.8).
Zum Beweis von (4.9) leiten wir die Steingleichung
UN′ (h) − wUN (h) = h − N (h)
ab:
(UN (h))′′ (w) = UN (h)(w) + w(UN (h))′ (w) + h′ (w)
= (1 + w 2 )UN (h)(w) + w(h(w) − N (h)) + h′ (w).
Wir drücken (UN (h))′′ in Termen von h′ aus. Dazu berechnen wir mittels partieller Integration:
Z
1
2
(h(x) − h(y))e−y /2 dy
h(x) − N (h) = √
2π Z
Z ∞ Z y
x Z x
1
2
′
−y 2 /2
(
h (z)dz)e
−
(
h′ (z)dz)e−y /2 dy)
= √ (
2π −∞ y
x
Z x
Z z
Z ∞x
Z ∞
1
2
′
−y 2 /2
′
= √ (
h (z)(
e
dy) −
h (z)(
e−y /2 dy)dz)
2π −∞
−∞
x
x
Z x
Z ∞
=
h′ (z)Φ(z)dz −
h′ (z)(1 − Φ(z))dz.
−∞
x
Somit folgt
w 2 /2
Z
w
2
(h(x) − N (h))e−x /2 dx
Z−∞
Z ∞
w Z x
2
2
w /2
′
= e
(
h (z)Φ(z)dz −
h′ (z)(1 − Φ(z))dz)e−x /2 dx
−∞
x
Z−∞
Z w
w
2
2
= ew /2 (
h′ (z)Φ(z)(
e−x /2 dx)dz
z
Z w −∞
Z z
2
′
−
h (z)(1 − Φ(z))(
e−x /2 dx)dz
Z−∞
Z −∞
∞
w
2
−
h′ (z)(1 − Φ(z))(
e−x /2 dx)dz)
w
Z−∞
Z ∞
w
√
2
w /2
′
((1 − Φ(w))
h (z)Φ(z)dz + Φ(w)
h′ (z)(1 − Φ(z))dz).
= − 2πe
(UN (h))(w) = e
−∞
46
w
Damit erhalten wir
′′
′
(UN (h)) (w) = h (w) + (w −
√
2
w 2 /2
2π(1 + w ))e
Z
√
2
+(−w − 2π(1 + w )Φ(w)
∞
w
(1 − Φ(w))
Z
w
h′ (z)Φ(z)dz
−∞
h′ (z)(1 − Φ(z))dz.
Offenbar ist für alle w > 0
w+
√
2
2π(1 + w 2 )ew /2 Φ(w) > 0.
(4.10)
Für w < 0 ist aber (wieder mittels partieller Integration)
√
2
w + 2π(1 + w 2 )ew /2 Φ(w)
Z w
Z w −x2 /2
2
ew /2
e
−x2 /2
2 w 2 /2
w 2 /2
e
dx + w e
(−
= w+e
−
dx)
w
x2
−∞
−∞
Z w
w2
2
w 2 /2
= e
(1 − 2 )e−x /2 dx > 0.
x
−∞
Also stimmt (4.10) für alle w. Wendet man es auf −w an, so ergibt sich
√
2
−w + 2π(1 + w 2 )ew /2 (1 − Φ(w)) > 0 für alle w.
Weiter ist
Z
und
(4.11)
2
w
e−w /2
>0
Φ(z)dz = wΦ(w) + √
2π
−∞
2
∞
e−w /2
> 0.
(1 − Φ(z))dz = −w(1 + Φ(w)) + √
2π
w
Mit der Positivität all dieser Ausdrücke erhalten wir
Z
√
2
e−w /2
kUN (h) k ≤ (1 + sup([−w + 2π(1 + w )e
]
(1 − Φ(w))][wΦ(w) + √
2π
w
2
√
e−w /2
2 w 2 /2
Φ(w)][−w(1 − Φ(w)) + √
])kh′ k
+[w + 2π(1 + w )e
2π
′
= 2kh k.
′′
2
w 2 /2
Dies beweist (4.9).
2
Wir wollen nun sehen, dass sich unsere Arbeit gelohnt hat:
Beweis von Satz 4.15: Die Klasse aller stetig differenzierbaren Abbildungen von R
nach R ist konvergenzdeterminierend für die schwache Konvergenz (das findet man z. B.
in “Weak Convergence of Probability Measures” von Billingsley und beweist es wie im
Portmanteau-Theorem). Sei also σ = 1 und ξ = E|Xi |3 < ∞. Sei Wn := √Snn und h : R → R
stetig differenzierbar mit kh′ k < ∞. Sei wieder
Z y
2
y 2 /2
UN (h) =: f := e
(h(x) − N (h))e−x /2 dx.
−∞
47
Weiter sei
Xi
Wni := Wn − √ .
n
Wir entwickeln f mit Hilfe der Taylor-Formel
f (Wn ) −
(f (Wni )
+ (Wn −
Wni )f ′ (Wni ))
=
Z
wn
Wni
(Wn − t)f ′′ (t)dt.
Also:
Z
Xi
Xi2 ′ i
Xi W n
Xi
′′
i
(Wn − t)f (t)dt .
f (Wn ) = E √
E √ f (Wn ) − √ f (Wn ) −
n
n
n
n Wni
Die rechte Seite können wir kontrollieren:
Z
Xi W n
′′
(Wn − t)f (t)dt |
|E √
n Wni
Z
Xi W n
≤ E| √
|Wn − Wni |kf ′′ kdt|
n Wni
≤ kf ′′ kE
|Xi |3
.
n3/2
Die Zufallsvariablen Xi und Wni sind stochastisch unabhängig, also ist
E(Xi f (Wni )) = EXi Ef (Wni ) = 0
und
E(Xi2 f ′ (Wni )) = EXi2 Ef ′ (Wni ) = Ef ′ (Wni ).
Somit erhalten wir
1
ξ
Xi
|E √ f (Wn ) − Ef ′ (Wni )| ≤ kf ′′ k 3/2 .
n
n
n
Dies ergibt:
|Eh(Wn ) − N (h)| = |E[f ′ (Wn ) − Wn f (Wn )]|
n
n
n
X
Xi
1X ′ i
1X ′ i
′
√ f (Wn )]|
f (Wn ))| + |E[
f (Wn ) −
≤ |E(f (Wn ) −
n i=1
n i=1
n
i=1
n
n
X 1
1X
Xi
≤
E|f ′ (Wn ) − f ′ (Wni )| +
| Ef ′ (Wni ) − E( √ f (Wn )|.
n i=1
n
n
i=1
Den ersten Summanden können wir mithilfe des Mittelwertsatzes durch
n
n
1 X ′′ E|Xi |
1X
E|f ′ (Wn ) − f ′ (Wni )| ≤
kf k √
n i=1
n i=1
n
abschätzen. Aufgrund der Jensen-Ungleichung ist
E|Xi | ≤ ξ 1/3 ,
48
also insgesamt mit den Vorüberlegungen und kf ′′ k ≤ 2kh′ k
Eh(Wn ) − N (h)| ≤
2kh′ kξ 1/3 2kh′ kξ
√
+ √ .
n
n
2
Bemerkung 4.19 Wir haben hier zusätzlich zu den Voraussetzungen von Satz 4.15 noch
angenommen, dass E|Xi |3 < ∞ ist. Einen Beweis ohne diese Annahme findet man im
Skript “Wahrscheinlichkeitstheorie I”. Dort bekommt man natürlich auch keine Schranke
an die Konvergenzgeschwindigkeit. Der Beweis liefert nicht ganz Satz 4.16, denn dort
müsste man Indikatoren
h(x) = 1l(−∞,w] (x)
betrachten, diese sind im entscheidenden Punkt schlecht differenzierbar. Approximiert man
h durch stetig differenzierbare Funktionen, so muss man eine Balance halten: Je genauer
die Approximation wird, desto größer wird die Schranke an die Ableitung. Ohne weitere
1
Ideen lässt sich so eine Ordnung von C · n1/4
erzielen. Ein induktiver Beweis von Bolthausen (1984) zeigt allerdings, dass sich Satz 4.16 mit einer Schranke der Form c/n1/2
mithilfe der Steinschen Methode beweisen lässt. Wir wollen hier davon absehen.
Wir wenden uns nun der Situation mit abhängigen Zufallsvariablen zu: Hier gilt die folgende Verallgemeinerung von Satz 4.15, die 1989 von Barbour, Karoński und Ruciński
gezeigt wurde (wir verzichten darauf, einen Beweis zu geben):
Satz 4.20 Sei W eine Zufallsvaria¡ble, die folgendermaßen zerlegen werdenkann: Für
eine endliche Indexmenge Ξ und Ξα ⊆ Ξ für α ∈ Ξ und quadratisch integrierbare Zufallsvariablen Xα , Wα , Zα , Zαβ , Wαβ und Vαβ , α ∈ Ξ, β ∈ Ξα gilt:
X
W =
Wα
α∈Ξ
W = Wα + Zα fürjedes α ∈ Ξ
X
Zαβ für jedes α ∈ Ξ
Zα =
β∈Ξα
Wα = Wαβ + Vαβ
α ∈ Ξ, β ∈ Ξα ,
für
wobei weiterhin gelte: EXα = 0 ∀α, Wα ist unabhängig von Xα und Wαβ ist unabhängig
von dem Paar (Xα , Zαβ ). Dann gibt es eine universelle Konstante C, so dass für σ 2 =
V W und
1
W̃ = W
σ
gilt
d1 (L(W̃ ), N (0, 1)) ≤
XX
C X
2
(E|Xα Zαβ Vαβ | + E|Xα Zαβ |E|Zα + Vαβ |).
(
E(|X
|Z
)
+
α
α
σ 3 α∈Ξ
α∈Ξ β∈Ξ
α
49
Bemerkung 4.21 Die Abstandsfunktion d1 zwischen zwei Zufallsvariablen X und Y ist
gegeben durch
d1 (X, Y ) = sup(|Eh(X) − Eh(Y )| : sup |h(x)| + sup |h′ (x)| ≤ 1.
x
x
Wir schreiben in Missbrauch von Notationen N (0, 1) für eine Gaußsche Zufallsvariable
ebenso wie für ihre Verteilung. Konvergenz in d1 impliziert Verteilungskonvergenz.
Natürlich besteht die wesentliche Herausforderung in Anwendungen in der Konstruktion einer geeigneten Zerlegung der Indexmenge. Haben die (Xα )α∈Ξ genügend viel Unabhängigkeit, so geht das recht gut. Um dies in ein quantifizierbares Resultat umzuwandeln führen wir den Abhängigkeitsgraphen der (Xα ) ein. Dieser hat Ξ als Knotenmenge
und eine Kante zwischen α und β, wenn Xα und Xβ abhängig sind. Sind also A, B ⊆ Ξ so,
dass A und B in diesem Graphen keine verbindenden Kanten haben, so sind die Familie
(Xα )α∈A und (Xβ )β∈B unabhängig. Man definiert in diesem Abhängigkeitsgraphen L auf
natürliche Weise die Umgebung NL (α1 , . . . , αr ) einer Knotenmenge {α1 , . . . , αr } ⊆ V (L).
NL (α1 , . . . , αr ) =
r
[
{β ∈ V (L)∃i, so dass β = αi oder {αi , β} ∈ E(L)}.
(4.12)
i=1
Dann gilt das folgende Resultat
Satz 4.22 Sei
W =
X
Xα ,
α∈Ξ
wo (Xα )α∈Ξ eine Familie von Zufallsvariablen mit Abhängigkeitsgraph L ist und für die
EXα = 0 ∀ α ∈ Ξ gilt. Sei
σ 2 = VW
mit 0 < σ 2 < ∞. Dann gilt für die Umgebung NL (α) wie in (4.12) und eine universelle
Konstante C:
C X X
d1 (L(W̃ ), N (0, 1)) ≤ 3
(E|Xα Xβ Xγ | + E|Xα Xβ |E|Xγ ).
σ α
β,γ∈NL (α)
Beweis: Wir verwenden Satz 4.20 mit
Ξα = NL (α)
X
Wα =
Xβ
β ∈N
/ L (α)
X
Zα =
Xβ
β∈NL (α)
Zαβ = Xβ
X
Wαβ =
Xγ
γ ∈N
/ L (α)∪NL (β)
X
und Vαβ =
γ∈NL (β)\NL (γ)
50
Xγ .
Dann sieht man, dass
X
XX
(E|Xα Zαβ Vαβ | + E|Xα Zαβ |E|Zα + Vαβ |)
E(|Xα |Zα2 ) +
α
α∈Ξ
≤ 2
X
X
α β,γ∈NL (α)
β∈Bα
(E|Xα Xβ Xγ | + E|Xα Xβ |E|Xγ |),
was das Resultat impliziert (hierbei dürfen wir annehmen, dass EXα2 < +∞ gilt, denn
sonst ist die rechte Seite unserer Behauptung sowieso unendlich).
2
Als Konsequenz hieraus erhalten wir
Satz 4.23 Sei (Sn )n∈N eine Folge von Zufallsvariablen mit
X
Xnα ,
Sn =
α∈Ξn
wobei für jedes n (Xnα )α eine Familie von zentrierten Zufallsvariablen mit Abhängigkeitsgraph Ln ist. Angenommen, es gibt Zahlen Mn und Qn , so dass
X
E|Xnα | ≤ Mn
(4.13)
α∈Ξn
und für alle α1 , α2 ∈ Ξn
X
α∈NLn (α1 ,α2 )
Sei σn2 = VSn . Dann gilt für S̃n =
E(|Xnα | |Xnα1 , Xnα2 ) ≤ Qn .
(4.14)
Sn
σn
d1 (L(S̃n ), N (0, 1)) = O
Mn Q2n
σn3
.
Insbesondere gilt
d
S̃n −→ N (0, 1),
falls
Mn Q2n
→ 0.
σn3
Beweis: OBdA EXnα = 0 (sonst ersetzen wir Xnα durch Xnα − EXnα ). Bemerke, dass
dies nichts an den Bedingungen (4.13) und (4.14) ändert, wenn wir Qn und Mn durch 2Qn
und 2Mn ersetzen. Das folgende Lemma (dessen Beweis wir allerdings schuldig bleiben)
zeigt, dass
X X
(E|Xα Xβ Xγ | + E|Xα Xβ |E|Xγ |) ≤ 2Mn Q2n .
(4.15)
α β,γ∈NL (α)
Damit folgt die Behauptung aus Satz 4.22.
2
51
Lemma 4.24 Unter den Voraussetzungen von Satz 4.23 gilt (4.15).
Beweis: Übung, siehe Janson, Luczak, Rucziński, Lemma 6.17.
2
Als eine Anwendung betrachten wir wieder die Statistik, die die Anzahl von Teilgraphen
in G(n, p) zählt. Sei Γ dazu wieder ein fester Graph mit eΓ > 0. Es sei p(n) diesmal so,
dass mit n → ∞
npm(Γ) → ∞
und desweiteren
n2 (1 − p) → ∞
gilt. Sei (Γα )α∈Ξn die Familie von Teilgraphen in Kn , die zu Γ isomorph sind, sei
Iα = 1lΓα ⊆G(n,p)
und
Xα = Iα − EIα .
Sei
XΓ =
X
Iα .
α∈Ξn
Dann ist
XΓ − EXΓ =
X
Xα .
α∈Ξn
Wir überprüfen zunächst (4.13) und (4.14). Sei dazu Ln der Abhängigkeitsgraph der Iα ,
d. h. α, β ∈ Ξn sind in Ln verbunden, falls die zu α und β gehörigen Graphen Gα und Gβ
in Kn eine gemeinsame Kante haben. Nun beachte, dass
E|Xα | = 2EIα (1 − EIα ) ≤ const(Γ)(1 − p)EIα
und daher
X
α
also gilt (4.13) mit
E|Xα | ≤ const(Γ)(1 − p)EXΓ ,
Mn = const(Γ)(1 − p)EXΓ = O((1 − p)EXΓ ).
Nun seien α1 , α2 ∈ Ξn gegeben. Setze
F = Γα1 ∪ Γα2
und für jedes α ∈ Ξn
Fα = Γα ∩ F.
Bemerke, dass
6 0.
α ∈ NLn (α1 , α2 ) ⇔ |e(Fα )| =
52
Es gibt weniger als 2vF ≤ 22vΓ solcher Teilgraphen F und für jedes H ⊆ F gibt es
O(nvΓ −vH ) Wahlen von α, so dass Fα = H gilt, wobei
E(|Xα | |Xα1 , Xα2 ) ≤ E(|Iα | |Xα1 , Xα2 ) + EIα ≤ 2peΓ −eH .
Da weiterhin jedes Fα isomorph zu einem Subgraphen von Γ ist, folgt, dass
X
α∈NL (α1 ,α2 )
E(|Xα | |Xα1 , Xα2 ) ≤ B sup
H⊂Γ
eH ≥1
EXΓ
EXΓ
=B
EXH
ΦΓ
für ein geeignetes B, das von Γ abhängt und ΦΓ wie in Abschnitt 3. Also gilt auch (4.14)
mit
EXΓ
EXΓ
Qn = B
=O
.
ΦΓ
ΦΓ
Also gilt:
Mn Q2n
const(Γ)(1 − p)(EXΓ )3 Φ−2
Γ
=
=O
σn3
σn3
denn aus Lemma 3.9 folgt
σn2 ≈ V(XΓ ) ≈ (1 − p)
1
−1/2
√
,
Φ
1−p Γ
(EXΓ )2
.
ΦΓ
Da nun
ΦΓ = min{EXH : H ⊆ Γ : eH > 0}
1
− m(Γ)
gegen ∞ geht, wenn p ≫ n
, folgt
Satz 4.25 Sei Γ fest mit eΓ > 0. Falls nun mit n → ∞
npm(Γ) → ∞
n2 (1 − p) → ∞,
und
dann gilt
X̃Γ → N (0, 1).
Bemerkung: Es gilt sogar:
d1 (L(X̃Γ ), N (0, 1)) = O
53
1
1
√
1 − p Φ1/2
Γ
!
→ 0.
5
Verzweigungsprozesse
Da wir in der Folge einen kritischen Zufallsgraphen häufig mit einem Verzweigungsprozess vergleichen wollen, stellen wir ein kurzes Kapitel üver Verzweigungsprozesse voran.
Ein Verzweigungsprozess ist das einfachste Modell einer sich zeitlich entwickelnden Population. Wir stellen uns vor, dass jeder Organismus in einer Population zu diskreten
Zeitpunkten eine Anzahl von Kindern zeugt und dann stirbt. Wir nehmen an, dass die
Verteilung der Nachkommenzahl über alle Organismen die gleiche ist und wir bezeichnen
diese mit (pi )i , wobei pi die Wahrscheinlichkeit sei, dass ein Organismus i Kinder zeugt.
Es sei Zn die Größe der Population zur Zeit n, mit Z0 = 1 und
Zn−1
Zn =
X
Xn,i .
i=1
Es ist dabei Xn,i ein Dreiecksschema von unabhängigen Zufallsvariablen, die gemäß (pi )i
verteilt sind, oftmals nennen wir eine wolche Zufallsvariable auch X.
Der erste zentrale Satz der Theorie der Verzweigungsprozesse besagt nun, dass der Prozess
ausstirbt, wenn E[X] ≤ 1 (mit der Ausnahme, dass für X ≡ 1 der Prozess natürlich
überlebt), während es mit positiver Wahrscheinlichkeit überlebt, wenn EX ≥ 1. Sei
η = P[∃n : Zn = 0]
die Aussterbewahrscheinlichkeit des Prozesses. Dann gilt:
Satz 5.1 Sei der Verzweigungsprozess Zn wie oben definiert. Dann gilt:
• EX < 1 ⇒ η = 1;
• EX = 1, P(X = 1) < 1 ⇒ η = 1;
• EX > 1 ⇒ η < 1.
Ist weiter GX die erzeugende Funktion von X
EsX =: GX (s),
dann ist η die kleinste Lösung von
η = GX (η).
Beweis: Sei ηn = P(Zn = 0). Da {Zn = 0} ⊆ {Zn+1 = 0} gilt, dass ηn steigend ist, da
ηn ≤ 1 ∀ n, gilt
ηn → η für n → ∞.
Sei
Gn (s) = EsZn
54
die erzeugende Funktion der n-ten Generation. Wenn X nur Werte in N0 annimmt, ist
P(X = 0) = GX (0). Also folgt
ηn = Gn (0).
Zerlegt man nach der Größe der ersten Generation, folgt
Gn (s) = Es
Zn
=
∞
X
i=0
Zn
pi E[s |Z1 = i] =
∞
X
pi Gin−1 (s).
i=0
Schreiben wir also GX für GX1,1 , so gilt
Gn (s) = GX (Gn−1 (s)).
Setzen wir s = 0 ein, bekommen wir für ηn die Gleichung
ηn = GX (ηn−1 ).
Für n → ∞ konvergiert ηn gegen η, also folgt wegen der Stetigkeit von G:
η = GX (η).
Ist P(X = 1) = 1, so gibt es stets ein Individuum, und mehr ist nicht zu sagen. Ist
P(X1 ≤ 1) = 1 und p0 > 0,
so folgt
P(Zn = 0) = 1 − (1 − p0 )n → 1.
Sei also P(X ≤ 1) < 1. Angenommen, ψ ∈ [0, 1] erfüllt
GX (ψ) = ψ.
Dann ist η ≤ ψ, denn η0 = 0 ≤ ψ und da GX monoton wachsend ist, folgt dann induktiv
ηn = GX (ηn−1 ) ≤ GX (ψ) = ψ.
Lassen wir n → ∞ gehen, so folgt η ≤ ψ. Also ist η der kleinste Fixpunkt von GX .
Bemerke, dass GX (wachsend, wie schon bemerkt und) konvex ist, denn
|n|
GX (s) = E[X(X − 1)sX−2 ] ≥ 0.
Ist, wie angenommen, P(X ≤ 1) < 1, so ist
|n|
GX (s) > 0,
also GX strikt konvex. Also kann die Gleichung
s = GX (s)
(5.1)
höchstens 2 Lösungen in [0, 1] haben. Eine Lösung ist stets s = 1. Da GX (0) > 0, gibt
es genau eine Lösung von (5.1), wenn G′X (1) ≤ 1 gilt, während es für G′X (1) > 1 zwei
Lösungen gibt. Also ist
55
• η = 1, wenn G′X (1) < 1,
• η < 1, wenn G′X (1) > 1.
Ist G′X (1) = 1, so gibt es wieder genau eine Lösung, es sei denn GX (s) = s, was zu
P(X = 1) äquivalent ist. Da
G′X (1) = EX,
folgt die Behauptung.
2
Oftmals werden wir statt η die Überlegenswahrscheinlichkeit ζ verwenden, die durch
ζ = 1 − η,
d. h.
ζ = P(Zn > 0, ∀ n ∈ N)
definiert ist. Wir studieren nun die Größe der gesamten Nachkommenschaft T des Verzweigungsprozesses, definiert durch
T =
∞
X
Zn .
n=0
Sei GT die erzeugende Funktion von T :
GT (s) = EsT .
Das zentrale Resultat ist hier
Satz 5.2 Für einen Verzweigungsprozess mit i.i.d. Nachkommenverteilung (sei X die
zugehörige Zufallsvariable) gilt
GT (s) = s GX (GT (s)).
Beweis: Wir bedingen wieder auf die Größe der ersten Generation und benutzen, dass,
wenn Z1 = i die totale Nachkommenschaft Tj des j-ten Kindes der ersten Generation für
j = 1, . . . , i aus i.i.d. Zufallsvariablen besteht, die so verteilt sind wie T . Benutzt man
also
i
X
T =1+
Tj
j=1
(wobei
P0
j=1
Tj ≡ 0), erhält man
GT (s) =
∞
X
pi E[sT |Z1 = i]
i=0
∞
X
= s
pi E[sT1 +...+Ti ]
i=0
= s
∞
X
pi GT (s)i
i=0
= sGX (GT (s)).
56
2
Wir wollen uns nun der mittleren Generationsgröße eines Verzweigungsprozesses zuwenden
und daraus Folgerungen ableiten.
Satz 5.3 Für alle n > 0 gilt
E[Zn ] = µn ,
wobei
µ := EZ1 = EX.
Beweis: Da
Zn−1
Zn =
X
Xα,i ,
i=1
folgt durch Bedingen auf Zn−1
EZn = µE[Zn−1 ].
2
Übung 5.4
Zn
µn
ist ein Martingal.
Satz 5.5 Für einen Verzweigungsprozess mit i.i.d. verteilter Nachkommenschaft, die so
verteilt ist wie die Zufallsvariable X, für die EX = µ < 1 gelte, gilt
ET =
1
.
1−µ
Beweis: Übung.
2
Vergleicht man einen Verzweigungsprozess mit einem Zufallsgraphen, so ist es oftmals
praktischer, einen Markov-Kette-Standpunkt einzunehmen. Seien also X1 , X2 , . . . i.i.d.
mit PX1 = PX1,1 . Sie S0 , S1 , . . . rekursiv definiert durch
S0 = 1
(5.2)
Si = Si−1 + Xi − 1 =
i
X
j=1
Xj − (i − 1).
Sei T der kleinste Index, für den St = 0 gilt, d. h.
T = min{t : St = 0} = min{t : X1 + . . . + Xt = t − 1}.
Gibt es so ein t nicht, setzen wir T = +∞. Diese Beschreibung ist äquivalent zu der Beschreibung eines Verzweigungsprozesses, aber beschreibt den Verzweigungsprozess anders,
57
so ist es beispielsweise aus dieser Beschreibung die Generationsgröße abzulesen. Um zu
sehen, dass die beiden Beschreibungen übereinstimmen, zeigen wir, dass die Verteilung
der Zufallsvariable T im Markovkettenbild mit der Zufallsvariable T im normalen Bild des
Verzweigungsprozesses, der totalen Nachkommenschaft übereinstimmt. Um dies zu sehen,
beachte, dass sich der zum Verzweigungsprozess gehörige Baum folgendermaßen erkunden
lässt: Sei X1 die Anzahl der Kinder des ursprünglichen Individuums und sei S1 = X1 − 1.
Starten wir in der Wurzel, so gibt es S1 Individuen, deren Nachkommenschaft wir noch
nicht untersucht haben. Wir behaupten, dass sich für alle Si eine solche Interpretation
geben lässt.
′
Lemma 5.6 Der Prozess (Si )i hat dieselbe Verteilung wie der Prozess (Si′ )∞
i=1 , wobei Si
die Anzahl der nocht nicht untersuchten Nachkommen in dem Explorationsprozess der Population eines Verzweigungsprozesses, nachdem man sukzessiv i Nachkommen untersucht
hat.
Beweis: Wir beweisen dies induktiv nach i. Für i = 0 ist nichts zu zeigen. Sei die
Behauptung für Si−1 gezeigt. Ist Si−1 = 0, so gilt die Behauptung, denn dann ist der
gesamte Stammbaum untersucht, und die Anzahl der untersuchten Individuen ist so groß
wie der Stammbaum, also T .
Sei also Si−1 > 0. Dann wählen wir ein beliebiges noch nicht untersuchtes Individuum
und bezeichnen mit der Zufallsvariable Xi die Anzahl seiner Nachkommen. Da die NachD
kommenzahla verschiedener Individuen unabhängig ist, ist Xi = Z1 . Somit haben wir,
nachdem wir die Kinderzahl des i-ten Individuums festgestellt und somit das i-te Individuum erkundet haben, Xi Individuen hinzugewonne, die wir erkunden müssen, während
wir eines (das i-te) erkundet haben. Die Anzahl der noch nicht erkundeten Individuen ist
also Si−1 + Xi − 1 = Si .
2
Es folgt also auch, dass T als Stoppzeit definiert in Verteilung gleich der gesamten Nachkommenschaft ist.
Der zu der Rekursion gehörige Verzweigungsprozess ist der folgende: Wir beginnen mit
einem aktiven Individuum. Zur Zeit i, wählen eines der aktiven Individuen aus, geben ihm
Xi Kinder und setzen es selbst auf inaktiv, während die Kinder (falls Xi > 0) auf “aktiv”
gesetzt werden. Das wird fortgesetzt, bis es keine aktiven Mitglieder der Population mehr
gibt. Dann ist Si die Anzahl der aktiven Knoten, nachdem i Individuen untersucht wurden.
Der Prozess stoppt für St ≡ 0, kann aber für alle t definiert werden, weil der Wert von
T dabei unberührt bleibt. Tatsächlich macht der Explorationsprozess nur für i ≤ T Sinn,
da nur dann Si ≥ 0 ∀ i ≤ T gilt. Er kann aber für alle i ≥ 0 definiert werden, was wir
bei Gelegenheit benutzen werden.
Sei weiter H = (X1 , . . . , XT ) die Geschichte des Prozesses bis zur Zeit T . Wir lassen
auch T = ∞ zu, in welchem Fall H unendliche Länge hat. Eine Folge (x1 , . . . , xt ) ist eine
mögliche Geschichte genau dann, wenn si > 0 für alle i < t und st = 0 gilt, dabei ist
si =
i
X
j=1
xj − (i − 1).
58
Dann gilt für jedes endliche t < ∞
P(H = (x1 , . . . , xt )) =
t
Y
px i .
(5.3)
i=1
(5.3) legt die Verteilung des Verzweigungsprozesses fest, bedingt darauf, dass er ausstirbt.
Wir können diese Perspektive also einnehmen, um die Verteilung des Verzweigungsprozesses bedingt darauf, dass er ausstirbt, zu beschreiben. Wir nennen dafür zwei Verteilungen
p und p′ ein konjugiertes Paar, falls
p′x = η x−1 px
für alle x gilt. Hierbei ist η die Aussterbewahrscheinlichkeit, die zur Nachkommenverteilung (px ) gehört, also
η = GX (η).
Übung 5.7 Zeigen Sie, dass p′ = (p′x ), x ∈ N, eine Wahrscheinlichkeitsverteilung ist.
Der Grund, p′ einzuführen ist der folgende:
Satz 5.8 Seien p und p′ konjugierte Wahrscheinlichkeiten. Dann hat der Verzweigungsprozess mit Nachkommenverteilung p bedingt darauf, dass er ausstirbt, die gleiche Verteilung wie ein Verzweigungsprozess mit Nachkommenverteilung p′ .
Beweis: Es genügt zu zeigen, dass für jede endliche Geschichte (x1 , . . . , xn ) die Wahrscheinlichkeit (5.3) die gleiche ist für einen Verzweigungsprozess mit Nachkommenverteilung p, bedingt darauf, dass er ausstirbt und für einen Verzweigungsprozess mit Nachkommenverteilung p′ . Sei also t < ∞ fest. Beachte zunächst, dass
P(H = (x1 , . . . , xt )|Aussterben)
=
P({H = (x1 , . . . , xt )} ∩ {Aussterben})
P(Aussterben)
(5.4)
= P({H = (x1 , . . . , xt )})/η,
da jede endliche Geschichte impliziert, dass der Prozess ausstirbt. Benutzen wir nun (5.3)
zusammen mit
t
Y
i=1
px i =
t
Y
p′xi
i=1
Pt
= η
t−
= η
Y
1
η xi −1
i=1
xi
t
Y
i=1
i=1
59
p′xi ,
p′xi
da x1 + . . . + xt = t − 1. Setzt man in (5.4) ein, erhält man
P(H = (x1 , . . . , xr )|Aussterben) = P′ (H(x1 , . . . , xr )),
wobei P′ die zu p′ Verteilung von H ist.
2
Übung 5.9
a) Zeigen Sie, dass für die erzeugende Funktion des dualen Prozesses
Gd (s) = E′ sX1 gilt
1
Gd (s) = GX (ηs).
η
b) Zeigen Sie, dass, falls für den Originalprozess η > 1 gilt,
E′ [X] < 1
gilt, der zu p′ gehörige Verzweigungsprozess ist also subkritisch.
Ein weiterer Aspekt der Markov-Ketten-Perspektive ist, dass man handlich die Aussterbewahrscheinlichkeit berechnen kann, wenn der Baum eine gewisse Größe hat:
Satz 5.10 Habe ein Verzweigungsprozess wieder i.i.d. Nachkommen, die verteilt sind wie
X mit µ = EX > 1. Dann gilt
P[k ≤ T < ∞] ≤
e−Ik
,
1 − e−I
wobei I gegeben ist durch
I = sup(t − log EetX ).
t≤0
Bemerkung 5.11 Eine Kernaussage von Satz 5.10 ist, dass die Überlebenswahrscheinlichkeit hoch ist, wenn wir erst einmal eine gewisse Populationsgröße erreicht haben. Bemerke, dass für µ > 1 und EetX < ∞ für alle t ∈ R gilt
I = sup(t − log EetX ).
t
Allerdings wird EetX ∀ t ∈ R nicht in Satz 5.10 vorausgesetzt. Da X ≥ 0 ist, ist allerdings EetX für t ≤ 0 immer endlich. Da auch die Ableitung von
t 7→ t − log EetX
in t = 0 1 − EX < 0 ist, wird das Supremum in t < 0 angenommen. Daher folgt auch
I > 0 ohne weitere Annahmen.
60
Beweis von Satz 5.10: Wenn T = s ist, so folgt Ss = 0, also
X1 + . . . + Xs = s − 1 ≤ s.
Daher folgt
P(k ≤ T < ∞) ≤
∞
X
s=k
P(Ss = 0) ≤
∞
X
s=k
P(X1 + . . . + Xs ≤ s).
Benutzt man die obere Schranke aus dem Satz von Cramér, dann ist
P(
n
X
s=1
Xi ≤ n) = e−nJ(1)
mit
J(a) = sup[ta − log EetX1 ]
t≤0
(da 1 < EX1 ), so ergibt sich
P(k ≤ T < ∞) ≤
∞
X
e−sI =
s=k
e−Ik
.
1 − e−I
2
Wir wollen uns nun der Konvergenz der Populationen der n-ten Generation zuwenden.
Da diese im kritischen und subkritischen Fall ausstirbt, konvergiert Zn gegen 0 und mehr
ist nicht zu sagen. Für µ > 1 gilt
lim P(Zn = k) = 0,
n→∞
es sei denn, k = 0 und
P( lim Zn = 0) = 1 − P(lim Zn = ∞) = η,
n→∞
wobei η eben wieder die Aussterbewahrscheinlichkeit des Prozesses ist. Sei η < 1, d. h.
die Population überlebt mit positiver Wahrscheinlichkeit, und in diesem Fall ist
lim Zn = +∞.
Satz 5.12 Für einen Verzweigungsprozess mit i.i.d. Nachkommenverteilung (wie X) mit
µ = EX > 1, gilt
Zn
→ W∞ P-f.s.
µn
für eine P-f.s. endliche Zufallsvariable W∞ .
61
Beweis: Da
Zn
µn
ein Martingal mit
E
1
Zn
= n EZn = 1
n
µ
µ
ist, folgt die Behauptung aus dem Martingalkonvergenzsatz.
2
Allerdings lässt sich über W∞ wenig sagen.
Ein wichtiger Satz über die Konvergenz im superkritischen Fall ist der folgende
Satz 5.13 (Kesten-Stigun)
Für einen Verzweigungsprozess mit i.i.d. Nachkommenvertielung (wie X) mit EX = µ > 1
gilt
P(W∞ = 0) = η ⇔ E[X log X] < ∞.
In diesem Fall gilt auch E[W∞ ] = 1, während für E[X log X] = ∞ gilt P[W∞ = 0] = 1.
Der Beweis soll hier nicht gegeben werden. Er befindet sich z. B. im Buch von Arthreya
und Ney, Branching Processes.
Satz 5.11 impliziert, dass
P[W∞ > 0] = 1 − η,
so dass W∞ > 0 bedingt darauf, dass der Prozess überlebt.
Es bleibt die Frage, was passiert, wenn EX log X = +∞. In diesem Fall hat Seneta gezeigt,
dass es eine geeignete Normalisierung (cn )∞
n=1 gibt, so dass
lim c1/n
=µ
n
n→∞
und
Zn
konvergiert gegen einen nicht-entarteten Grenzwert
cn
gilt. Allerdings ist cn = 0(µn ), so dass P(W∞ = 0) = 1 gilt. Wir wollen uns nun mit
den Individuen beschäftigen, die für immer überleben. Diese formen wieder einen Verzweigungsprozess, wie wir gleich sehen werden. Dazu seien Zn∞ diejenigen Individuen der
n-ten Generation, deren Nachkommen nie aussterben. Dann gilt
Satz 5.14 Der Prozess (Zn∞ )∞
n=1 ist wieder ein Verzweigungsprozess mit Nachkommen(∞)
(∞)
∞ ∞
verteilung (p ) = (pk )k=0 , die durch p0 = 0 und
∞ 1 X j j−k
(∞)
pk =
η (1 − η)k pj
(5.5)
ζ
k
j=k
für k ≥ 1 gegeben ist. Da
(∞)
µ(∞) = EZ1
= µ = EZ1 ,
(5.6)
ist dieser Verzweigungsprozess superkritisch mit der gleichen durchschnittlichen Anzahl
der Nachkommen wie der ursprüngliche Prozess.
62
Bemerkung 5.15 Es ist interessant, dieses Resultat mit Satz 5.8 zu vergleichen. Im superkritischen Regime ist der Verzweigungsprozess bedingt darauf, dass er ausstirbt, wieder
ein Verzweigungsprozess mit der dualen Verzweigungsrate, während die Individuen, bedingt auf Überleben, wieder einen superkritischen Verzweigungsprozess formen.
Beweis: Sei A∞ := {Zn → ∞}. Wir zeigen induktiv, dass für jedes n ≥ 0 die Verteilung
(∞)
von (Zk )nk=0 bedingt auf A∞ gleich ist zu der von (Ẑk )nk=0 , wobei (Ẑk ) ein Verzweigungsprozess mit Nachkommenverteilung p(∞) wie in (5.5) ist. Für n = 0 gilt natürlich
(∞)
Z0
= 1 = Ẑ0 .
Angenommen, die Annahme gelte für n. Die Behauptung für n + 1 folgt dann, falls wir
(∞)
(∞)
zeigen können, dass die bedingte Verteilung von Zn+1 gegeben (Zk )nk=0 gleich der von
Ẑn+1 gegeben (Ẑk )nk=0 ist. Letztere ist natürlich gleich der einer unabhängigen Summe
(∞)
(∞)
von Ẑn unabhängigen Zufallsvariablen. Die Verteilung von Zn+1 gegeben (Zk )nk=0 ist
(∞)
(∞)
gleich PZn+1 |Zn und jedes Individuum mit unendlich vielen Nachkommen, das wir in der
n-ten Generation betrachten, erzeugt eine zufällige i.i.d. Zahl von Nachkommen in der
(∞)
n + 1-sten Generation mit der gleichen Verteilung wie Z1 bedingt auf A∞ . Also bleibt
zu zeigen
(∞)
(∞)
P(Z1 = k|A∞ ) = pk .
Für k = 0 sind offenbar beide Seiten 0. Für k ≥ 1 bedingen wir auf Z1 . Für k ≥ 1
(∞)
impliziert Z1 = k, dass Z1 ≥ k und dass A∞ auftritt, also
(∞)
P(Z1
1
(∞)
P(Z1 = k)
ζ
1X
(∞)
P[Z1 = k|Z1 = j]P(Z1 = j)
=
ζ j≥k
1 X j j−k
η (1 − η)k · pj ,
=
ζ j≥k k
= k|A∞ ) =
da jedes der j Partikel mit Wahrscheinlichkeit ζ = 1 − η unendlich viele Nachkommen
hat, so dass
(∞)
P(Z1 = k|Z1 = j) = B(k; j, 1 − η).
Nun zeigen wir noch (5.6). Wir beginnen mit dem Fall µ < ∞. Dann ist
µ
(∞)
=
∞
X
(∞)
kpk
k=0
∞
X
∞ k X j j−k
=
η (1 − η)k pj
k
ζ
k=0
j=k
j
∞
1 X X j j−k
η (1 − η)k
k
pj
=
k
ζ j=0 k=0
∞
∞
X
1X
=
pj ζj =
jpj = µ.
ζ j=0
j=0
63
Der Fall µ = +∞ wird ähnlich bewiesen, indem man geeignet abschneidet. (Übung)
2
Die Sätze 5.12 und 5.14 ergeben folgendes Bild: Es gilt
Zn f.s.
−→ W∞ ,
µn
wobei P(W∞ > 0) = ζ, falls EX log X < +∞. Andererseits folgt aus Satz 5.14, dass
(∞)
bedingt auf A∞ (Zn )∞
n=0 auch ein Verzweigungsprozess mit durchschnittlicher Nachkommenzahl µ ist, der mit Wahrscheinlichkeit 1 überlebt. Also
(∞)
Zn
µn
wobei – bedingt auf A∞ – gilt:
(∞)
(∞)
−→ W∞
,
P−f.s.
(∞)
P(W∞
> 0) = 1,
während aber andererseits Zn ≤ Zn ∀ n gilt. Dies wirft automatisch die Frage auf,
(∞)
was die bedingten Größen von Zn und Zn auf A∞ sind. Die Antwort hierauf gibt
Satz 5.16 Bedingt darauf, dass der Prozess überlebt, gilt
(∞)
Zn
f.s.
−→ ζ.
Zn
Beweis: Sei zunächst µ < ∞. Wendet man die Sätze 5.14 und 5.12 an und dass, bedingt
(∞)
aufs Überleben, E[Z1 ] = µ gilt, erhalten wir, dass es W (∞) gibt mit
(∞)
Zn
→ W (∞)
µn
fast sicher.
Außerdem folgt aus Satz 5.13 und der Tatsache, dass die Aussterbewahrscheinlichkeit von
(∞)
Zn 0 ist, dass
P(W (∞) > 0) = 1.
Weiter folgt abermals aus Satz 5.12 angewandt auf (Zn ) bedingt auf das Überleben, dass
(∞)
Zn
gegen W∞ bedingt auf {W∞ > 0} konvergiert. Also konvergiert ZZnn fast sicher gegen
µn
einen endlichen und positiven Grenzwert R.
(∞)
Um zu sehen, dass dieser gleich ζ ist, benutzen wir, dass die Verteilung von Zn
Zn = k, B(k, ζ) ist. Also gilt für n → ∞, dass bedingt auf das Überleben
konvergiert.
gegeben
→ ζ
(∞)
Zn
Zn
Für µ = +∞ bleiben wir den Beweis hier schuldig.
2
64
6
Der Poisson-Verzweigungsprozess
Wir betrachten in diesem Abschnitt einen besonderen Verzweigungsprozess, den Poissonschen Verzweigungsprozess, bei dem die Nachkommenzahl P oi(λ)-verteilt ist. Wir schreiben zur Abkürzung Xλ für die P oi(λ)-verteilte Zufallsvariable und Tλ für die gesamte
Nachkommenschaft. Wir berechnen als erstes
GXλ (s) =
∞
X
si e−λ
i=0
λi
= eλ(s−1) .
i!
Daher gilt für die Aussterbewahrscheinlichkeit ηλ
ηλ = eλ(ηλ −1) .
(6.1)
Für λ ≤ 1 hat (6.1) nur eine Lösung ηλ = 1, also stirbt der Prozess sicher aus (was auch
klar ist, weil λ = E[Xλ ]). Für λ > 1 gibt es zwei Lösungen von (6.1), die kleinere der
beiden erfüllt ηλ ∈ (0, 1). Da in diesem Fall für die Gesamtpopulationsgröße Tλ gilt
Pλ (Tλ < ∞) < 1
gilt, wissen wir
Pλ (Tλ < ∞) = ηλ .
Wir erinnern, dass H = (Xλ,1 , . . . , Xλ,Tλ ) die Geschichte des Verzweigungsprozesses ist.
Dann gilt, bedingt auf Aussterben, dass der Poissonsche Verzweigungsprozess eine Verzweigungsrate p′λ gegeben durch
p′λ,i = ηλi−1 pλ,i = e−ληλ
(ληλ )i
i!
gilt. Dies ist wieder eine Poisson-Verteilung, diesmal mit Rate
µλ = ληλ .
Es gilt mit µ := µλ
µe−µ = ληλ e−ληλ = λe−λ .
Daher nennen wir (µ, λ), 0 < µ < 1 < λ ein konjugiertes Paar, falls µe−µ = λe−λ gilt. Da
x → xe−x
erst wachsend und dann fallend mit einem Maximum von 1e bei x = 1 ist, hat diese
Gleichung genau zwei Lösungen, eine kleiner als 1(µ) und eine größer as 1(λ). Satz 5.8
liest sich somit für Poisson-verteilte Nachkommen so:
Satz 6.1 Sei µ < 1 < λ konjugiert. Der Poissonsche Verzweigungsprozess mit Rate λ hat,
bedingt auf das Aussterben, dieselbe Verteilung wie ein Poissonscher Verzweigungsprozess
mit Rate µ.
Für die gesamte Nachkommenschaft Tλ gilt
65
Satz 6.2 Für einen Verzweigungsprozess mit i.i.d. P oi(λ)-verteilten Nachkommenzahlen
gilt
(λn)n−1 −λ
Pλ (Tλ = n) =
e .
n!
Übung 6.3 Für alle λ und alle genügend großen k gilt
Pλ (k ≤ Tλ < ∞) ≤ e−kIλ ,
wobei
Iλ = λ − 1 − log λ.
Für den Beweis von Satz 6.2 benötigen wir ein paar Begriffe. Wir nennen einen Baum
auf n Knoten einen markierten Baum auf {1, . . . , n}, wenn alle Knoten eine Marke aus
{1, . . . , n} tragen und jede dieser Marken genau einmal vorkommt. Damit können wir
auch die Kanten in einem solchen Graphen markieren (durch die Knoten). Zwei markierte
Bäume auf {1, . . . , n} sind gleich, wenn sie die gleichen n−1 Kanten besitzen. Jeder solche
Baum ist natürlich äquivalent zu einem aufspannenden Baum des Kn .
Der folgende Satz ist auch als Satz von Cayley bekannt:
Satz 6.4 (Satz von Cayley)
Es gibt genau nn−2 markierte Bäume der Größe n oder äquivalent nn−2 aufspannende
Bäume des Kn .
Beweis: Wir beweisen den zweiten Teil der Aussage. Dazu zeigen wir zuerst, dass jeder
aufspannende Baum eines endlichen Graphen einen Vertex vom Grad 1 hat. Nimmt man
nämlich das Gegenteil an, so hat jeder Vertex einen Grad von mindestens 2. Dann können
wir von irgendeinem Vertex eine Tour beginnen, indem wir eine seiner Kanten durchlaufen,
von da an laufen wir weiter, indem wir nur unbenutzte Kanten verwenden. Da der Graph
endlich ist, kommen wir irgendwann mit diesem Prozess am ein Ende. Da aber jeder
Vertex einen Grad mindestens zwei hatte, sind wir an einem Punkt angekommen, den wir
schon einmal besucht haben. Also ist der Graph kein Baum.
Nun sei t(n, d1 , . . . , dn ) die Anzahl der aufspannenden Bäume des Kn mit Graden d1 , . . . , dn ,
d. h. deg(vi) = di . Daher ist
X
t(n, d1 , . . . , dn )
d1 ,...,dn
die Gesamtzahl der aufspannenden Bäume des Kn . Ist eines der di gleich 0, so ist
t(n, d1 , . . . , dn ) = 0. Aus Symmetriegründen hängt t(n, d1 , . . . , dn ) nur von {d1, . . . , dn }
ab, aber nicht von deren Reihenfolge. Daher können wir
d1 ≥ . . . ≥ dn = 1
annehmen. Für n = 2 ist nichts zu zeigen. Für n ≥ 3 ist vn (der Vertex mit Grad dn = 1)
mit einem der vi verbunden und es gilt di ≥ 2. Da dafür jeder andere Vertex in Frage
kommt, gilt
n−1
X
t(n, d1 , . . . , dn ) =
t(n − 1, d1, . . . , di − 1, . . . , dn−1 ).
(6.2)
i=1
66
Für n = 3 gilt offenbar
t(3, d1, d2 , d3 ) =
1
d1 − 1, d2 − 1, d3 − 1
=
n−2
d1 − 1, . . . , dn−1 − 1
(6.3)
(nachrechnen).
Nun gilt für die linke Seite von (6.3) die Rekursion (6.2). Die rechte Seite erfüllt offenbar
die Rekursion für Multinomialkoeffizieten
X
n n−1
n
.
=
d1 , . . . , di − 1, . . . , dn
d1 , . . . , dn
i=1
Daraus leitet man induktiv ab, dass
t(n, d1 , . . . , dn ) =
=
=
n−1
X
t(n − 1, d1 , . . . , di − 1, . . . , dn )
i=1
n−1
X
i=1
n−3
d1 − 1, . . . , di − 2, . . . , dn−1 − 1
n−2
d1 − 1, . . . , di − 1, . . . , dn−1 − 1, dn − 1
(wobei der letzte Eintrag dn − 1 nichts ändert, da dn = 1 ist). Nun lassen sich die Multinomialkoeffizienten aus der Gleichung
X
n
n
xr1 . . . xrkk
(x1 + . . . + xk ) =
r1 , . . . , rk 1
gewinnen. Ersetzen wir hier k durch n, n durch n − 2 und ri durch di − 1 und xi durch
1, so ergibt sich
X
t(n, d1 , . . . , dn ).
nn−2 =
d1 ,...,dn
2
Der Satz von Cayley ergibt nun
Lemma 6.5 Für n ≥ 2 gilt:
n−1
X
1
i! n
i=1
X
1 +...+ni =n−1
n −1
i
Y
nj j
j=1
nj !
=
nn−1
.
n!
Beweis: Man benutzt, dass ein Baum mit n Knoten eindeutig bestimmt ist durch den
Grad des ersten Vertex (sagen wir i) und den markierten Unterbäumen, die von den i
direkten Nachbarn des Knoten 1 ausgehen. Sind diese Teilbäume n1 , . . . , ni groß, so gilt
n1 + . . . + ni = n − 1. Es gibt
(n − 1)!
n1 ! . . . ni !
67
Möglichkeiten, die (n − 1) verbleibenden Marken {2, . . . , n} in i Gruppen einzuteilen. Es
n −2
n −2
n −1
gibt ferner nj j Bäume der Größe nj , so dass es nj nj j = nj j Bäume der Größe nj mit
einem ausgezeichneten Vertex gibt. Nun ändert sich der Baum der Größe n nicht, wenn wir
die i Bäume, die an 1 angehängt sind, permutieren, und es gibt i! solcher Permutationen.
Also gibt es insgesamt
i
1 (n − 1)! Y nj −1
n
i! n1 ! . . . ni ! j=1 j
Arten, die i Bäume, die direkt an 1 hängen, zusammen mit den direkten Nachbarn von 1
auszuwählen. Summiert man dies über i, erhält man, dass die Gesamtzahl von Bäumen
der Größe n gleich ist zu
n−1
X
1
i! n
i=1
X
(n − 1)!
1 +...+ni =n−1
n −1
i
Y
nj j
nj !
j=1
.
Aufgrund von Cayleys Satz folgt daher
nn−2 =
n−1
X
1
i! n
i=1
X
(n − 1)!
1 +...+ni =n−1
n −1
i
Y
nj j
j=1
nj !
.
Dividiert man dies durch (n − 1)! und benutzt, dass
nn−1
nn−2
=
,
(n − 1)!
n!
so erhält man die Behauptung.
2
Wir können uns nun an den Beweis von Satz 6.2 machen:
Beweis von Satz 6.2: Wir gehen induktiv vor. Für n = 1 ist Tλ = 1 genau dann, wenn
das erste Individiuum ohne Nachkommen stirbt. Das hat Wahrscheinlichkeit e−λ . Aber
auch die rechte Seite von Satz 6.2 ist für n = 1 gleich e−λ . Dies ist der Induktionsanfang.
Für den Induktionsschritt bedingen wir auf die Anzahl i der Kinder des Ursprungs. Sei
die Anzahl der gesamten Nachkommen der i Kinder n1 , . . . , ni , also ist Tλ = n äquivalent
zu n1 + . . . + ni = n − 1. Somit gilt
Pλ (Tλ = n) =
n−1
X
e−λ
i=1
λi
i!
X
i
Y
Pλ (Tλ = nj ).
n1 +...+ni =n−1 j=1
Nach Induktionsvoraussetzung ergibt sich (beachte, dass nj ≤ n − 1 für alle j):
(λnj )nj −1 −λnj
Pλ (Tλ = nj ) =
e
.
nj !
Setzen wir das ein und beachten, dass
i
X
j=1
(nj − 1) = n − i − 1,
68
so erhalten wir
Pλ (Tλ = n) =
i−1
X
e−λ
i=1
λi
i!
−λn n−1
= e
λ
i
Y
(λnj )nj −1
X
n1 +...+ni =n−1 j=1
n−1
X
i=1
1
i! n
X
nj !
e−λnj
i−1 nj −1
Y
nj
1 +...+ni =n−1 j=1
nj !
.
Nach Lemma 6.5 ist dies gleich:
Pλ (Tλ = n) = e−λn λn−1 nn−1 /n!
wie behauptet.
2
Für den kritischen Poisson-Prozess leitet man mit der Stirlingschen Formel ab, dass
1
1
Pλ (Tλ = n) = √ n−3/2 (1 + O( ))
n
2π
gilt; das ist ein Beispiel für das “Powerlaw”-Verhalten im kritischen Punkt. n−3/2 ist dabei
sogar allgemeiner das Verhalten der kritischen Verteilung der Gesamtpopulationsgröße für
jede Nachkommenverteilung mit endlicher Varianz.
Im nächsten Kapitel wollen wir den Erdös-Rényi-Graphen G(n, p) mithilfe von Verzweigungsprozessen untersuchen. Hierfür brauchen wir noch, dass für λ > 1 die Aussterbewahrscheinlichkeit hinreichend glatt ist.
Korollar 6.6 Sei ηλ die Aussterbewahrscheinlichkeit des Poissonschen Verzweigungsprozesses mit Rate λ. Dann gilt für alle λ > 0:
d ηλ (λ − µλ )
ηλ =
< ∞,
dλ λ(1 − µλ )
wobei (µλ , λ) ein duales Paar bilden.
Beweis: Es gilt
ηλ = Pλ (Tλ < ∞) =
Daher folgt
∞
X
n=1
e−λn
(λn)n−1
.
n!
X
∞
∞
n−2
n−1
X
d
−nλ (λn)
−nλ (λn)
−
e
.
e
0 ≤ − ηλ =
dλ
(n − 1)!
(n − 2)!
n=2
n=1
Andererseits gilt:
Eλ [Tλ |Tλ < +∞] =
∞
∞
X
(λn)n−1
1 X −λn (λn)n−1
1
ne−λn
e
=
,
Pλ [Tλ < ∞] n=1
n!
ηλ n=1
(n − 1)!
69
so dass
d
ηλ
ηλ
ηλ = ηλ Eλ [Tλ |Tλ < ∞] − Eλ [Tλ |Tλ < ∞] + .
dλ
λ
λ
Hierbei haben wir benutzt, dass
−
∞
X
n=2
e−λn
∞
X
(λn)n−2
(λn)n−2
=
e−λn (n − 1)
(n − 2)!
(n − 1)!
n=1
=
∞
X
n=1
∞
(λn)n−1 X −λn (λn)n−2
−
e
λ (n − 1)! n=1
(n − 1)!
−λn 1
e
∞
n−1
X
ηλ
−λn 1 (λn)
=
Eλ [Tλ |Tλ < ∞] −
e
λ
λ n!
n=1
ηλ
1
Eλ [Tλ |Tλ < ∞] − Pλ [Tλ < ∞].
λ
λ
Wegen des Dualitätsprinzips und Satz 5.5 folgt
=
Eλ [Tλ |Tλ < ∞] =
1
,
1 − µλ
wobei µλ = ληλ gilt. Daher erhalten wir schließlich
0≤−
d
ηλ
1
ηλ
ηλ (λ − µλ )
ηλ =
(1 − ) +
=
.
dλ
1 − µλ
λ
λ
λ(1 − µλ )
2
Um tatsächlich einen Poissonschen Verzweigungsprozess bei der Analyse eines Zufallsgraphen verwenden zu können, benötigen wir noch eine Vorübung, deren Resultat plausibel
erscheint.
Wir werden im nächsten Kapitel G(n, p)-Graphen mit p ∼ nλ anschauen. Daher hat jeder Knoten ein B(ki n − 1, nλ )-verteilte Nachbarschaft, die zumindest lokal wie ein Baum
aussieht. Es liegt nahe, nicht nur eine B(ki n − 1, nλ )-Verteilung sondern alle durch unabhängige P oi(λ)-Verteilungen zu ersetzen. Dass man dies auch darf, besagt der folgende
Satz.
Satz 6.7 Für einen Verzweigungsprozess mit B(n, p) verteilter Nachkommenschaft und
gesamter Nachkommenzahl T und einen zweiten Verzweigungsprozess mit P oi(λ)-verteilter
Nachkommenschaft und totaler Populationsgröße λ gilt, wenn λ = np:
P(T ≥ k) = P(Tλ ≥ k) + ek (n),
wobei
also insbesondere
k−1
2λ2 X
|ek (n)| ≤
P(Tλ ≥ s),
n s=1
|ek (n)| ≤
gilt.
70
2kλ2
n
Beweis: Der Beweis benutzt - ähnlich wie der Beweis des Poissonschen Grenzwertsatzes in
der Stochastik - ein Kopplungsargument. Die entsprechenden Verzweigungsprozesse sind
durch ihre B(n, p)- bzw. P oi(λ)-verteilte i.i.d. Nachkommenschaft Xi bzw. Xiλ eindeutig
bestimmt. In der Stochastik haben wir gesehen, dass sich Xi an Xiλ so koppeln lässt, dass
P(Xi 6= Xiλ ) ≤
λ2
.
n
Wir zerlegen:
P(T ≥ k) = P(T ≥ k, Tλ ≥ k) + P(T ≥ k, Tλ < k)
und
P(Tλ ≥ k) = P(T ≥ k, Tλ ≥ k) + P(Tλ ≥ k, T < k).
Nach Substraktion dieser beiden Gleichungen voneinander ergibt sich
|P(T ≥ k) − P(Tλ ≥ k)| ≤ P(T ≥ k, Tλ < k) + P(Tλ ≥ k, T < k).
Wir bemerken, dass {T ≥ k} messbar ist bezüglich der X1 , . . . , Xk−1 : Tatsächlich gilt ja
{T < k} = {T ≥ k}c genau dann, wenn es ein t < k gibt, so dass X1 + . . . + Xt = t − 1
gilt. Somit können wir folgern, dass für {T ≥ k, Tλ < k} ∪ {T < k, Tλ ≥ k} ein s < k
existieren muss mit Xs 6= Xsλ . Also
P(T ≥ k, Tλ < k) ≤
k−1
X
s=1
P(Xi = Xiλ , ∀ i ≤ s − 1, Xs 6= Xsλ , T ≥ k)
(wobei wieder die (Xiλ ) i.i.d. P oi(λ)- und die (Xi ) davon unabhängige i.i.d. B(n, p)Variablen sind). Nun bemerke, dass, wenn Xi = Xiλ für alle i ≤ s − 1 und T ≥ k
insbesondere auch gilt
X1λ + . . . + Xiλ ≥ i ∀ i ≤ s − 1,
λ
also {Tλ ≥ s}. Darüber hinaus hängt {Tλ ≥ s} nur von X1λ , . . . , Xs−1
ab und ist daher
λ
unabhängig von dem Ereignis {Xs 6= Xs }. Daher erhalten wir
P(T ≥ k, Tλ < k) ≤
k−1
X
s=1
P(Tλ ≥ s, Xs 6= Xsλ ) =
k−1
X
s=1
P(Tλ ≥ s)P(Xs 6= Xsλ ).
Durch die Kopplungsschranke
P(Xs 6= Xsλ ) ≤
λ2
,
n
so dass
k−1
k+1
λ2 X
λ2 X
λ
P(Tλ ≥ s)P(Xs 6= Xs ) ≤
P(Tλ ≥ s)
P(T ≥ k, Tλ < k) ≤
n s=1
n s=1
und ebenso
k−1
P(T λ ≥ k, T < k) ≤
71
λ2 X
P(Tλ ≥ s).
n s=1
Also insgesamt:
k−1
2λ2 X
|P(T ≥ k) − Pλ (Tλ ≥ k)| ≤
P(Tλ ≥ s).
n s=1
2
Wir werden nun noch einmal ein Resultat über Irrfahrt verwenden, um die Verteilung der
Gesamtpopulationsgröße herzuleiten. Unser Ziel dabei ist
D
Satz 6.8 Für einen Verzweigungsprozess mit i.i.d. Nachkommenverteilung Z1 = X gilt
P(T = n) =
1
P(X1 + . . . + Xn = n − 1),
n
wobei die (Xi ) i.i.d. Kopien von X sind.
Bemerkung 6.9 Wir beweisen später sogar eine etwas allgemeinere Aussage, nämlich
P(T1 + . . . + Tk = n) =
k
P(X1 + . . . + Xn = n − k),
n
(6.4)
wobei T1 , . . . , Tk k unabhängige Zufallsvariablen mit der Verteilung von T sind (oder wir
stellen uns einen Verzeigungsprozess mit k Ursprungsindividuen vor).
Der Beweis benutzt wieder die Markov-Ketten-Darstellung des Verzweigungsprozesses
zusammen mit dem Treffzeiten-Satz für Irrfahrten. Um diesen zu beschreiben sei (Yi)i
eine Folge von i.i.d. Zufallsvariablen mit Werten in Z und
Sn = k +
n
X
Yi
i=1
die Irrfahrt mit Start in k und Zuwachsverteilung PY1 . Sei
T0 = inf{n ≥ 0 : Sn = 0}
die erste Treffzeit der 0 der Irrfahrt. Dann gilt
Satz 6.10 Falls die (Yi )i von oben
P(Yi ≥ −1) = 1
erfüllen, so gilt für die Verteilung von T0 :
Pk (T0 = n) =
k
P(Sn = 0).
n
72
(6.5)
Bemerkung 6.11 Noch erstaunlicher wird das Resultat, wenn man unter {Sn = 0} bedingt. Das Resultat heißt dann
Pk (T0 = n|Sn = 0) =
k
n
(die Wahrscheinlichkeit, dass man zur Zeit n zum ersten Mal in 0 ist, gegeben, man ist in
0 ist nk ), unabhängig von der Verteilung der Yi .
Unter der Voraussetzung von Satz 6.10 können wir nun (6.4) und damit auch Satz 6.8
beweisen.
Beweis von Satz 6.8: Beachte, dass L(T1 + . . . + Tk ) gleich der Verteilung einer Treffzeit
einer Irrfahrt ist, die in k startet und deren Zuwachs Yi = Xi − 1 ist, wobei (Xi )i die
Nachkommenzahlen der Knoten sind. Da Xi ≥ 0 ist, gilt Yi ≥ −1, daraus folgt (6.4), also
Satz 6.8. (Die Details sind eine Übung.)
2
Es bleibt der Beweis von Satz 6.10:
Beweis von Satz 6.10: Wir zeigen (6.5) für alle k ≥ 0 durch Induktion über n ≥ 1. Für
n = 1 gleichen beide Seiten 0 für k > 1 und k = 0, und sie gleichen P(Yi = −1) für k = 1.
Dies ist der Induktionsanfang.
Für n ≥ 2 sind beide Seiten gleich 0 für k = 0. Also sei k ≥ 1. Wir bedingen auf den
ersten Schritt und erhalten:
∞
X
Pk (T0 = n) =
Pk (T0 = n|Y1 = s)P(Y1 = s).
s=−1
Aus der Markoveigenschaft folgt:
Pk (T0 = n|Y1 = s) = Pk+s (T0 = n − 1) =
k+s
Pk+s (Sn−1 = 0),
n−1
wobei die letzte Gleichheit aus der Induktionsvoraussetzung folgt (was wegen k ≥ 1 und
s ≥ −1, also k + s ≥ 0 erlaubt ist). Dies führt zu
Pk (T0 = n) =
∞
X
k+s
Pk+s (Sn−1 = 0)P(Y1 = s).
n
−
1
s=−1
Mit dem Satz von der totalen Wahrscheinlichkeit folgt unter Ausnutzung von
Pk+s (Sn−1 = 0) = Pk (Sn = 0|Y1 = s),
dass
∞
1 X
(k + s)Pk (Sn = 0|Y1 = s)P(Y1 = s)
Pk (T0 = n) =
n − 1 s=−1
= P(Sn = 0)(k + Ek [Y1 |Sn = 0])
73
1
.
n−1
Offenbar ist E[Yi |Sn = 0] unabhängig von i, so dass
n
1X
Ek [Y1 |Sn = 0] =
Ek [Yi |Sn = 0]
n i=1
n
X
k
1
Yi |Sn = 0] = − ,
Ek [
=
n
n
i=1
da ja
Pn
i=1
Yi = Sn − k = −k sein muss, wenn Sn = 0 ist. Also erhalten wir schließlich
Pk [T0 = n] =
1
k
k
(k − )Pk (Sn = 0) = Pk (Sn = 0).
n−1
n
n
2
Bemerkung 6.12 Der Treffzeiten-Satz ist eng verwandt mit dem Ballot-Theorem, das
eine lange Geschichte hat und auf Bertrand zurückgeht:
∞
Satz 6.13 Sei (Sn ) eine Irrfahrt
Pn aus i.i.d. Zuwächsen (Xi )i=1 , wobei die Xi ≥ 0 und
ganzzahlig sind. Ist also Sn = i=1 Xi , so gilt
P0 (Sm < m für alle 1 ≤ m ≤ n|Sn = n − k) =
74
k
.
n
7
Der Phasenübergang im Erdös-Rényi-Graph
Wie schon mehrfach angekündigt, wollen wir in diesem Abschnitt den Phasenübergang
im Modell G(n, p) studieren. Dieser ist vergleichbar zum Phasenübergang in den Verzweigungsprozessen: Dort überlebte ein Verzweigungsprozess mit µ = EX > 1 mit positiver
Wahrscheinlichkeit, während er für µ < 1 ausstirbt. Etwas Ähnliches beobachtet man im
Erdös-Rényi-Graph. Für p = nλ und λ < 1 besteht der Graph aus vielen “kleinen” Komponenten, für λ > 1 besteht der Graph aus beinahe allen Knoten. Der Vergleich zwischen
G(n, p) und Verzweigungsprozess wird auch bei den Beweisen eine wichtige Rolle spielen.
Wir beginnen mit ein wenig Notation. Sei
[n] := {1, . . . , n}.
Für s, t ∈ [n] schreibe s ↔ t, wenn es einen Pfad in G von s nach t gibt. Für v ∈ [n] sei
C(v) die Zusammenhangskomponente von v, also
C(v) = {x ∈ [n] : x ↔ v}.
Die Größe von C(v) sei |C(v)|. Die größte Zusammenhangskomponente ist einfach eines
der Cluster C(v), die am größten sind. Also
|Cmax | = max{|C(v)| : v ∈ [n]}.
Natürlich ist |Cmax | eindeutig Cmax , allerdings ist es nicht notwendig.
Wir geben zunächst einen Mechanismus an, mit dem man für v ∈ [n] C(v) finden kann.
Dies ist eng verwandt mit der Markov-Ketten-Perspektive eines Verzweigungsprozesses.
Dazu geben wir einen Explorationsprozess an, in dessen Verlauf Knoten drei Stadien
durchlaufen: Knoten können aktiv, neutral oder inaktiv sein. Sie wechseln ihren Zustand
dabei wie folgt: Zur Zeit t = 0 ist nur v aktiv, alle anderen Knoten sind neutral. Wer
setzen S0 = 1. Zur Zeit t wählen wir einen beliebigen aktiven Vertex w und erkunden
alle Kanten {w, w ′}, w ′ durchläuft dabei alle neutralen Knoten. Dabei werden die w ′
auf “aktiv” gesetzt; danach setzen wir w auf inaktiv und St als die neue Anzahl aktiver
Knoten. Wenn es keine weiteren Vertizes mehr gibt, d. h. wenn erstmals St = 0 gilt, hält
der Prozess an und die Zusammenhangskomponente C(v) ist die Menge aller inaktiven
Knoten. Hieraus folgt auch |C(v)| = t. Während des gesamten Prozesses ist |C(v)| nach
unten beschränkt durch die Anzahl aktiver und inaktiver Vertices. Sei wt der t-te aktive
Vertex, von dem schon alle Kanten, die zu neutralen Knoten gehen, untersucht sein mögen.
Sei Xt die Anzahl der neutralen Vertices mit {wt , w} ∈ G. Sei St die Anzahl aktiver Knoten
zur Zeit t. Ähnlich wie beim Verzweigungsprozess kann man
S0 = 1 und St = St−1 + Xt − 1
(7.1)
schreiben. Dabei ist Xt die Anzahl der Knoten, die durch die Untersuchung des t-ten
Knoten aktiv werden und nach dieser Untersuchung wird der t-te untersuchte Knoten
inaktiv. Dies zeigt (7.1).
Dies stimmt natürlich für jeden Graphen. Nun spezialisieren wir uns auf G(n, p). Dann
hängt die Verteilung von Xt von der Anzahl der aktiven Knoten zur Zeit t = 1, d. h.
75
St−1 , ab. Dies ist allerdings die einzige Abhängigkeit von der Konfiguration der aktiven,
neutralen und inaktiven Knoten. Genauer hat jeder neutrale Knoten w ′ eine Wahrscheinlichkeit p, aktiv zu werden. Da zu jedem Zeitpunkt die Kanten zu inaktiven und aktiven
Vertices nicht mehr untersucht werden und zur Zeit t, t − 1 Knoten inaktiv sind und St−1
aktiv, ist
Xt ∼ B(n − (t − 1) − St−1 , p)
(7.2)
verteilt. Wir bemerken, dass (7.1) die gleiche Rekursion ist wie (5.1) bei Verzweigungsprozessen. Der einzige Unterschied zwischen den zwei Prozessen besteht in der Verteilung
der Xt , die in (7.2) vom gegenwärtigen Prozess abhängt, während sie für Verzweigungsprozesse i.i.d. ist. Allerdings ist die Folge (7.2) “fast” i.i.d., so lange t und St nicht zu groß
sind.
Sei T wieder definiert durch
T = inf{t : St = 0},
wobei |C(v)| = T . Ähnliches hatten wir auch für Verzweigungsprozesse gesehen. Ähnlich
wie dort ergibt die Rekursion (7.1) und (7.2) nur für St−1 ≥ 1 Sinn, trotzdem können wir
den Prozess auch für St−1 = 0 definieren, dann bleibt er 0.
Im folgenden wollen wir den Graphen G(n, p) untersuchen, wobei p in der Größenordnung
von n1 , genauer
λ
p=
n
ist.
Wir werden nun die oft zitierte Verwandschaft zwischen einem G(n, nλ )-Zufallsgraphen und
einem P oi(λ)-Verzweigungsprozess informell beschreiben. Tatsächlich sind dies die grundlegenden Beweisideen, allerdings werden wir die heuristisch vorgetragenen Argumente
nicht direkt benutzen. Sei also λ > 0. S1λ , . . . , X1λ , X2λ , H λ beziehen sich auf einen Verzweigungsprozess mit P oi(λ)-verteilten i.i.d. Zuwächsen. Ebenso seine S0 , S1 , . . . , X1 , X2 , . . . , H
die entsprechenden Größen für G(n, nλ ). Die Si sind oben definiert, ebenso die Xi . Für die
Definition der λ-Variablen erinnern wir an Kapitel 6. Insbesondere haben wir dort gesehen,
dass für jegliche mögliche Vergangenheit (x1 , . . . , xt ) gilt
λ
Pλ (H = (x1 , . . . , xt )) =
t
Y
Pλ (Xiλ = xi ),
i=1
wobei die X(λi ) i.i.d. P oi(λ)-verteilt sind. Andererseits gilt in G(n, nλ )
P(H = (x1 , . . . , xn )) =
n
Y
i=1
P(Xi = xi |X1 = x1 , . . . , Xi−1 = xi−1 ),
wobei, bedingt auf X1 = x1 , . . . , Xi−1 = xi−1 die Zufallsvariable Xi B(n−(i−1)−si−1 , nλ )verteilt ist. Sind λ und i fest, so gilt (leichte Übung)
λi
λ
lim P(B(m(n), ) = i) = e−λ
n→∞
n
i!
76
für eine Folge m(n) = n(1 + o(1)). Also gilt für jedes feste t
lim P(H = (x1 , . . . , xt )) = P(H λ = (x1 , . . . , xt )).
n→∞
Somit ist die Verteilung der endlichen Zusammenhangskomponenten in G(n, nλ ) eng verwandt mit der Geschichte eines Verzweigungsprozesses mit i.i.d. P oi(λ)-Nachkommenverteilung. Auf dieser Verwandschaft bauen die folgenden Beweisideen auf.
Wir beginnen mit zwei stochastischen Ordnungsresultaten. Wir schreiben dabei für zwei
Zufallsvariablen X, Y
X Y,
wenn
P(X ≥ k) ≤ P(Y ≥ k) ∀ k
(dabei können wir uns natürlich X und Y auf einem gemeinsamen Wahrscheinlichkeitsraum vorstellen oder nicht, das ist belanglos).
Satz 7.1 Für jedes k ≥ 1 gilt
PG(n,p) (|C(1)| ≥ k)) ≤ Pn,p (T ≥ k),
d. h.
|C(1)| T.
Dabei ist C(1) die Zusammenhangskomponente der 1 in G ∈ G(n, p), p = nλ und Pn,p das
Maß eines Binomialen Verzweigungsprozesses mit Parameter n und p und T die Größe
seiner Gesamtpopulation.
Beweis: Dies folgt aus der Explorationsbeschreibung der Zusammenhangskomponente
und derselben Beschreibung für T . Wir bemerken, dass in der Beschreibung für T gilt
St = St−1 + Xt − 1,
(7.3)
wobei die Xt ∼ B(n, p)-verteilt sind, während (7.3) auf die Beschreibung von C(1) auch
zutrifft, diesmal aber mit Xt , in denen der Parameter der Binomialverteilung fällt. Somit
ist XtC (das Xt für die Zusammenhangskomponente) stochastisch dominiert durch XtT
(das Xt ∼ B(n, p) für den Verzweigungsprozess). Hieraus folgt die Behauptung (Details
sind eine Übung).
2
Wir können die Größe von C(1) aber auch stochastisch von unten beschränken. Allerdings
nicht durch eine feste Zufallsvariable. Dies ist der Inhalt des folgenden Satzes:
Satz 7.2 Für alle k ∈ [n] gilt
PG(n, λ ) (|C(1)| ≥ k) ≥ Pn−k,p (T ≥ k).
n
Dabei ist die linke Seite wie in Satz 7.1, die rechte Seite beschreibt die Tails der Verteilung der Gesamtpopulation eines Verzweigungsprozesses mit i.i.d. B(n − k, nλ )-verteilten
Zuwächsen.
77
Da die rechte Seite von k abhängt, haben wir keine stochastische Dominanz im oben
definierten Sinn.
Beweis: Wir wählen wieder einen Kopplungsansatz. Bei dem üblichen Explorationsprozess für C(1) definieren wir zunächst die Knoten n − k + 2, . . . , n als “verboten”, was
bedeutet, dass wir Kanten, die in diesen Knoten enden, nicht untersuchen. Also gibt es
nun vier mögliche Zustände für die Knoten. Während des Explorationsprozesses werden
wir den Pool an verbotenen Knoten so verwalten, dass die Gesamtzahl an aktiven, inaktiven und verbotenen Knoten gleich k ist.
Mit unserer Initialisierung haben wir dies bereits richtig gemacht, denn zu Beginn gibt es
genau einen aktiven, keinen inaktiven und k − 1 verbotene Vertices. Natürlich geht das
nur so lange gut wie die Summe aus aktiven und inaktiven Vertices höchstens k ist. Dies
stellt aber kein Problem dar, denn sobald die Summe aus aktiven und inaktiven Vertices
größer oder gleich k ist, sehen wir, dass das Ereignis {|C(1)| ≥ k} eintritt.
Wieder untersuchen wir nur Kanten zu neutralen Knoten. Wenn wir eine Kante zu einem
solchen Knoten finden, dann setzen wir diesen Knoten auf “aktiv” und den verbotenen
Knoten mit dem größten Index auf “neutral”.
Somit ist die Anzahl der neutralen Vertices auf n − k festgelegt. Formal sei (Iij ) eine i.i.d.
Ber(p)-Folge. Setze
Xi =
X
Ivi,j
und Xi≤ =
j∈Ai−1
X
Ivi,j .
j∈Ai−1,k
Dabei ist Ai−1 die Menge der aktiven und Ai−1,k die Menge der aktiven, nicht-verbotenen
Vertices zur Zeit i − 1, also
|Ai−1,k | = n − k.
Dann ist (Xi≤ ) eine i.i.d. Folge von B(n − k, p) Zufallsvariablen.
Solange die Anzahl der aktiven und inaktiven Vertizes höchstens k ist, folgt, dass die Anzahl der verbotenen, aktiven und inaktiven Vertizes genau k ist. Wir haben also einen Verzweigungsprozess mit Binomial-verteilter Nachkommenschaft mit den Parametern n − k
und p. Da die Parameter damit unterhalb des entsprechenden Prozesses für die Exploration von C(1) liegen, folgt die Behauptung.
2
Die allgemeine Strategie für die Untersuchung der größten Komponente |Cmax | ist nun die
folgende: Wir benutzen die stochastische Schranke aus Satz 7.1 und 7.2, um die |Cmax |
durch Binomiale Verzweigungsprozesse abzuschätzen. Diese können wir gemäß den Resultaten aus Abschnitt 6 mit einem P oi(λ)-Verzweigungsprozess, wenn p ≈ nλ . Die Resultate,
die wir über diesen Prozess erhalten haben, werden uns hilfreich sein, unsere Resultate zu
erreichen.
Da das Verhalten eines P oi(λ)-Verzweigungsprozesses sehr unterschiedlich ist, je nachdem, ob λ < 1, λ = 1 oder λ > 1 gilt, können wir auch für das Verhalten von |Cmax |
unterschiedliche Resultate erwarten, je nachdem ob np < 1 oder np > 1 gilt.
78
Wir beginnen mit dem subkritischen Fall. Sei also zunächst
λ = np < 1.
Sei Iλ die Ratenfunktion
Iλ = λ − 1 − log λ.
Beachte, dass Iλ ≥ 0 und Iλ = 0 ⇔ λ = 1. Das nächste Resultat ist das wesentliche für
den subkritischen Fall; es zeigt, dass mit großer Wahrscheinlichkeit
|Cmax | ≤ a log n
für jedes a > I1λ gilt. Das andere wichtige Resultat, Satz 7.4, zeigt, dass diese Grenze auch
scharf ist, genauer, dass
|Cmax | ≥ a log n
gilt, für alle a <
1
.
Iλ
Genauer lesen sich diese Resultate wie folgt:
Satz 7.3 Sei λ < 1. Dann gibt es für jedes a >
1
Iλ
ein δ = δ(a, λ) > 0, so dass
PG(n, λ ) (|Cmax | ≥ a log n) = O(n−δ ).
n
Satz 7.4 Sei λ < 1. Dann gibt es für jedes a <
1
Iλ
ein δ = δ(a, λ) > 0, so dass
PG(n, λ ) (|Cmax | ≥ a log n) = O(n−δ ).
n
Bemerkung 7.5 Insgesamt erhält man aus den beiden Sätzen
|Cmax | P 1
−→ .
log n
Iλ
(Der Beweis ist eine Übung.)
Die Beweisstrategie für Satz 7.3 und Satz 7.4 ist die folgende: Sei
Z≥k =
n
X
v=1
1l{|C(v)|≥k}
die Anzahl an Knoten, die in einer Zusammenhangskomponente der Größe mindestens k
enthalten sind. Offenbar ist
|Cmax | = max{k : Z≥k ≥ k}.
(7.4)
Somit können wir Schranken an |Cmax | beweisen, indem wir ein geeignetes Z≥k betrachten.
Insbesondere gilt
{|Cmax | ≥ k} = {Z≥k ≥ k}.
(7.5)
79
Übung 7.6 Beweisen Sie (7.4) und (7.5).
Satz 7.3 werden wir mithilfe der Methode des ersten Moments beweisen. Wir berechnen
Eλ (Z≥k ) := EG(n, λ ) [Z≥k ] = nPλ (|C(1)| ≥ k)
n
und wir benutzen Satz 7.1, um Pλ (|C(1)| ≥ kn ) für kn = a log n und a >
schränken. Daher gilt mit großer Wahrscheinlichkeit
Z≥kn = 0,
1
Iλ
zu be-
d. h. |Cmax | ≤ kn .
Der Beweis von Satz 7.4 folgt aus einer Zweiten Momenten-Ungleichung. Dazu beschränken
wir zunächst die Varianz von Z≥k (siehe unten). Danach benutzen wir Satz 7.2, um
Eλ [Z≥kn ] für kn = a log n für a < I1λ zu beschränken. Wir werden sehen, dass diese
Schranken ausreichen, um einzusehen, dass
Zkn > 0 mit Wahrscheinlichkeit, die gegen 1 konvergiert,
also: |Cmax | ≥ kn .
Beweis von Satz 7.3: Wegen Satz 7.1 gilt
Pλ (|C(v)| > t) ≤ Pn,p (T > t),
wobei T die Gesamtnachkommenschaft eines B(n, nλ )-Verzweigungsprozesses ist. Um die
λ
rechte Seite zu studieren, seien (X̂i )∞
i=1 i.i.d. B(n, p) = B(n, n )-Variablen und
Ŝt = X̂1 + . . . + X̂t − t − 1.
Dann gilt nach den Eingangsüberlegungen:
Pn,p (T > t) ≤ Pn,p (Ŝt > 0) = Pn,p (X̂1 + . . . + X̂t ≥ t).
Man berechnet, dass
Pn,p (X̂1 + . . . + X̂t ≥ t) ≤ e−tIλ
(Übung). Somit erhalten wir zusammen mit der einleitenden Übung für kn = a log n
Pλ (|Cmax | ≥ a log n) ≤
≤
=
≤
=
für a >
1
,
Iλ
hier ist δ = aIλ − 1.
Pλ (Z≥kn ≥ 1)
Eλ (Z≥kn )
nPλ (|C(1)| ≥ a log n)
n1−aIλ
O(n−δ )
2
Wir geben noch einen zweiten Beweis, der auf einer Verteilungsgleichheit von St beruht,
die auch für λ > 1 sehr nützlich ist. Das Resultat besagt, dass St auch Binomialverteilt
ist, jedoch mit einer anderen Erfolgswahrscheinlichkeit.
80
Proposition 7.7 Für alle t ∈ {1, . . . , n} gilt
St + (t − 1) ∼ B(n − 1, 1 − (1 − p)t ).
Beweis: Sei Nt die Anzahl der zur Zeit t noch nicht untersuchten Knoten, d. h.
Nt = n − t − St .
Offenbar gilt für jede Zufallsvariable X, dass
X ∼ B(m, p)
ist, genau dann, wenn
Y := m − X ∼ B(m, 1 − p).
Wir zeigen daher
Nt ∼ B(n − 1, (1 − p)t ).
Das ist heuristisch klar, wenn man bedenkt, dass jeder Vertes {1, . . . , n} unabhängig von
allen anderen eine Wahrscheinlichkeit von (1−p)t hat, während der ersten t Explorationen
neutral zu bleiben. Formaler: Bedingt auf St−1 gilt
Xt ∼ B(n − (t − 1) − St−1 , p).
Setzen wir also N0 = n − 1 und
Nt =
=
=
=
=
n − t − St
n − t − St−1 − Xt + 1
n − (t − 1) − St−1 − B(n − (t − 1) − St−1 , p)
Nt−1 − B(Nt−1 , p)
B(Nt−1 , 1 − p)
und wir erhalten den gewünschten Beweis induktiv über t.
2. Beweis von Satz 7.3: Mithilfe der vorhergehenden Proposition erhalten wir
Pλ (|C(v)| > t) ≤ Pλ (St > 0) ≤ Pλ (Bin(n − 1, 1 − (1 − p)t ) ≥ t).
Benutzen wir die Bernoulli-Ungleichung
1 − (1 − p)t ≤ tp,
erhalten wir
tλ
≥ t)
n
tλ
≤ min e−st Eλ [esB(n, n ) ]
Pλ (|C(v)| > t) ≤ Pλ (B(n,
s≥0
tλ s
(e − 1)]n
n
−st tλ(es −1)
≤ min e (e
),
= min e−st [1 +
s≥0
s≥0
81
2
wobei wir in der letzten Ungleichung
1 + x ≤ ex
benutzt haben. Daher erhalten wir
Pλ (|C(v)| > t) ≤ e−Iλ t
und der Rest des Beweises folgt wie im ersten Beweis.
2
Nun wenden wir uns dem Beweis von Satz 7.4 zu. Wir definieren
χ≥k (λ) = Eλ [|C(v)|1l{|C(v)|≥k} ],
was aus Symmetriegründen nicht von v abhängt.
Proposition 7.8 Für alle n und k ∈ [n] gilt:
Vλ [Z≥k ] ≤ nχ≥k (λ).
Beweis: Wir benutzen:
X
Vλ (Z≥k ) =
[Pλ (|C(i)| ≥ k, |C(j)| ≥ k) − Pλ (|C(i)| ≥ k)Pλ (|C(j)| ≥ k].
i,j
Den ersten Summanden spalten wir auf, je nachdem, ob i ⇔ j oder nicht:
Pλ (|C(i)| ≥ k, |C(j)| ≥ k) = Pλ (|C(i)| ≥ k, i ↔ j) + P(|C(i)| ≥ k, |C(j)| ≥ k, i 6↔ j),
da im ersten Fall C(i) = C(j) gilt. Ist |C(i)| = l und i 6↔ j, dann bilden die Knoten und
Kanten außerhalb von C(i) einen Zufallsgraphen mit n − l Knoten. Da die Wahrscheinlichkeit des Eriegnisses {|C(j)| ≥ k} im G(n, p) wachsend in n ist, folgt
Pλ (|C(j)| ≥ k|C(i) = li 6↔ j) ≤ Pλ (|C(j)| ≥ k).
Also können wir folgern:
Pλ (|C(i)| = l, |C(j)| ≥ k, ı 6↔ j) − Pλ (|C(i)(= l)Pλ (|C(j)| ≥ k) ≤ 0.
Daraus erhalten wir:
Nλ (Z≥k ) ≤
n
X
i,j=1
Pλ (|C(i)| ≥ k, i ↔ j).
Daher erhalten wir wieder aus der Austauschbarkeit (Symmetrie) aller Knoten:
X
Vλ (Z≥k ) ≤
Pλ (|C(i)| ≥ k, i ↔ j)
=
i,j
n X
n
X
i=1 j=1
Eλ [1l{|C(i)|≥k} 1l{j∈C(i)} ].
82
Da
Pn
j=1 1l{j∈C(i)}
= |C(i)| gilt, gelangen wir zu:
Vλ (Z≥k ) ≤
X
i
Eλ [|C(i)|1l{C(i)≥k} ]
= n · Eλ [|C(1)|1l{|C(1)|≥k} ]
= nχ≥k (λ).
2
Beweis von Satz 7.4: Es genügt zu zeigen, dass
Pλ [Z≥kn = 0] = O(n−δ )
mit kn = a log n und a < I1λ gilt. Hierfür verwenden wir die Chebyshev-Ungleichung.
Hierfür leiten wir eine untere Schranke für Eλ [Z≥k ] und eine obere Schranke für Nλ [Z≥k ]
her. Für die erste dieser Schranken benutzen wir
Eλ [Z≥k ] = nP≥k (λ),
wobei
P≥k (λ) := Pλ (|C(v)| ≥ k)
ist. Sei k = kN− = a log n. Nach dem, was wir eingangs dieses Kapitels gesehen haten,
ist für T aus einem B(n − kn , nλ )-Verzweigungsprozess:
P≥k (λ) ≥ Pn−kn ,p (T ≥ a log n)
mit p = nλ . Nach Satz 6.2 und Übung 6.3 gilt für einen P oi(λn )-Verzweigungsprozess mit
n
) und dessen vollständiger Nachkommenzahl T ∗ :
λn = λ( n−k
n
2
aλ log n
∗
∗
Pn−kn ,p (T ≥ a log n) = Pλn (T ≥ a log n) + O
.
n
Ebenso folgt aus Satz 6.2
P∗λn (T ∗
≥ a log n) =
∞
X
P∗λn (T ∗
k=a log n
∞
X
(λn k)k−1 −λn k
= k) =
e
.
k!
k=a log n
Mithilfe der Stirlingformel, der Definition von Iλ und
Iλn = Iλ + o(1)
ergibt sich
∞
1 X
1
√
e−Iλn k (1 + o(1)) = e−Iλ a log n(1+o(1)) .
P(T ≥ a log n) =
3
λ
2πk
k=a log n
∗
Daher folgt mit kn = a log n für jedes 0 < α < 1 − aIλ
Eλ [Z≥kn ] = nP≥kn (λ) ≥ n(1−Iλ a)(1+o(1)) ≥ nα .
83
Als nächstes beschränken wir die Varianz von Z≥kn unter Ausnutzung von Proposition
7.8. Es gilt
n
X
χ≥kn (λ) =
t=kn
P≥t (λ) ≤
−(kn −1)Iλ
e
1 − e−Iλ
= O(n−αIλ ).
≤
n
X
e−Iλ (t−1)
t=kn
Somit folgt aus Proposition 7.8
Vλ (Z≥kn ) ≤ nχ≥kn (λ) ≤ O(n1−αIλ ),
während
Eλ [Z≥kn ] ≥ nα .
Also folgt
Pλ (Z≥kn = 0) ≤
Vλ (Z≥kn )
Eλ [Z≥kn ]2
≤ O(n1−αI−2α)
= O(n−δ ),
wenn wir
δ = 2α − (1 − Iλ α)
und 0 < α < 1 − Iλ α wählen, so dass
δ = 2α − (1 − Iλ α) > 0
ist. Schließlich verwenden wir noch
Pλ (|Cmax | < kn ) = Pλ (Z≥kn = 0),
was Satz 7.4 beweist.
2
Nun wenden wir uns dem superkritischen Bereich zu, d. h. wir fählen ein festes λ > 1.
Wir schreiben
ζλ = 1 − η − λ
für die Überlebenswahrscheinlichkeit des Poisson(λ)-Verzweigungsprozesses. Dann gilt
Satz 7.9 (Gesetz der großen Zahlen für die riesige Komponente)
Sei λ > 1. Dann gibt es für jedes ν ∈ ( 21 , 1) ein δ = δ(ν, λ) > 0, so dass
Pλ (|Cmax | − ξλ n| ≥ nν ) = O(n−δ ).
84
Satz 7.9 kann folgendermaßen interpretiert werden: Ein Knoten hat eine große Zusammenhangskomponente mit Wahrscheinlichkeit ζλ . Daher gibt es Θ(ζλn) ?? Knoten mit einer
großen Zusammenhangskomponenten. Satz 7.9 sagt, dass all diese großen Komponenten
tatsächlich dieselbe sind.
Wir skizzieren zunächst die Beweisstrategie für Satz 7.9. Diese basiert auf einer Analyse
der Anzahl der Knoten, die in einer Zusammenhangskomponente der Größe mindestens
k liegen.
n
X
1l{|C(v)|≥k} .
Z≥k =
v=1
Wir wählen zunächst
k = kn = K log n
für ein k > 0 hinreichend großes K. Bemerke, dass
E[Z≥ kn ] = nPλ [|C(v)| ≥ kn ].
Wir berechnen
Pλ [|C(v)| ≥ kn ]
mithilfe von Satz 7.2. Genauer berechnen wir die Verteilung der Clustergröße in Proposition 7.10 (weiter unten), die besagt, dass für kn = K log n und K hinreichend groß
Pλ [|C(v)| ≥ kn ] = ζλ(1 + o(1)).
Dann zeigen wir, dass es für k = kn = K log n für k > 0 (hinreichend groß) keine Zusammenhangskomponente der Größe zwischen kn und αn für jedes α < ζ. Dies machen wir
mit der ersten Momentenmethode: Die erwartete Anzahl an Knoten in solchen Zusammenhangskomponenten ist gleich
Eλ [Z≥kn − Z≥αn ]
und wir benutzen die oben beschriebene Schranke aus Proposition 7.10 und ebenso Proposition 7.11 (ebenfalls unten), die besagt, dass für jedes α < ζλ ein J > 0 existiert, so
dass
Pλ [kn ≤ |C(v)| ≤ αn] ≤ e−kn J .
Daher gibt es für hinreichend großes k > 0 kein Cluster der Größe zwischen kn und αn.
Anschließend benutzen wir eine Varianzabschätzung für Z≥k in Proposition 7.12, die impliziert, dass mit großer Wahrscheinlichkeit und für alle ν ∈ ( 21 , 1) gilt
|Z≥kn − Eλ [Z≥kn ]| ≤ nν .
(7.6)
Schließlich benutzen wir, dass für 2α > ζλ , bedingt auf dem Ereignis, dass es keine Cluster
der Größe zwischen kn und αn gibt und auf dem Ereignis (7.6) gilt:
Z≥kn = |Cmax |.
Der Beweis von Satz 7.9 folgt dann aus einer Kombination dieser Fakten.
85
Proposition 7.10 Sei λ > 1. Dann gilt im Limes n → ∞ und für kn = a log n, wobei
a > I1λ mit Iλ definiert wie oben
Pλ [|C(v)| ≥ kn ] = ζλ + O(
kn
).
n
Beweis: Wir benutzen die Abschätzung aus Satz 7.1 und die Abschätzung für die Größe
des Clusters eines Verzweigungsprozesses:
Pλ (|C(v)| ≥ kn ) ≤ Pn, λ (T ≥ kn ) ≤ P∗λ (T ≥ kn ) + O(
n
kn
),
n
wobei T und T ∗ die Gesamtbevölkerung eines Binomial- bzw. Poisson-Verzweigungsprozesses
sind. Um den Beweis zu vervollständigen benutzen wir Satz 6.2 und Übung 6.3, um
P∗λ (T ∗ ≥ kn ) = P∗λ (T ∗ = ∞) + P∗λ (kn ≤ T < ∞)
= ζλ + O(e−kn Iλ )
kn
= ζλ + O( )
n
zu erhalten.
Für die untere Schranke benutzen wir Satz 7.2, so dass wir mit
λn = λ(1 −
kn
)
n
erhalten:
Pλ (|C(v)| ≥ kn ) ≥ Pn−kn , λ (T ≥ kn ) ≥ P∗λn (T ∗ ≥ kn ) + O(
n
kn
),
n
wobei nun T und T ∗ die gesamte Nachkommenschaft eines B(n − kn , nλ )- bzw. P oi(λn )Verzweigungsprozesses sind. Nach Übung 6.3 gilt für kn ≥ a log n und a > I1λ
P∗λn (T ∗ ≥ kn ) = ζλn + O(e−kn Iλn ) = ζλn + O(
kn
).
n
Nach dem Mittelwertsatz folgt
ηλn = ηλ + (λn − λ)
d
kn
ηλ |λ=λ∗n = ηλ + O( )
dλ
n
für ein λ∗n ∈ (λn , λ), wobei wir Korollar 6.6 mit λ > 1 und λn − λ = knn benutzen. Also
gilt auch
kn
ζλn = ζλ + O( ).
n
Wenn wir diese Abschätzungen zusammenfassen, bekommen wir die untere Abschätzung.
Zusammen mit der vorher gezeigten oberen Abschätzung ist dies der Beweis von Proposition 7.10.
2
86
Proposition 7.11 Sei λ > 1 und kn so, dass kn → ∞. Dann gibt es für jedes α < ζλ ein
J = J(α, λ) > 0, so dass
Pλ (kn ≤ |C(v)| ≤ αn) ≤ Ce−kn J .
Beweis: Wir beginnen mit der Schranke
Pλ (kn ≤ |C(v)| ≤ αn) =
αn
X
t=kn
Pλ (|C(v)| = t) ≤
αn
X
Pλ (St = 0).
t=kn
Nach Proposition 7.7 ist St ∼ B(n − 1, 1 − (1 − p)t ) + 1 − t. Daher gilt nun p =
Pλ (St = 0) = Pλ (B(n − 1, 1 − (1 − p)t ) = t − 1).
λ
n
(7.7)
Um den exponentiellen Abfall der Wahrscheinlichkeiten zu erklären, beachte man, dass
für p = nλ und t = αn gilt
1 − (1 − p)t = 1 − (1 −
λ αn
) = (1 − e−λα )(1 + o(1)).
n
Die Gleichung
1 − e−λα = α
wird nun eindeutig durch α = ζλ gelöst (dies ist eine Übung).
Für α < ζλ ist daher auch α < 1 − e−λα und die Wahrscheinlichkeit in (7.7) fällt exponentiell. Im Detail: Wir starten mit (7.7) und benutzen, dass
1 − p ≤ e−p ,
also 1 − (1 − p)t ≥ 1 − e−pt
gilt, um zu erhalten:
Pλ (St = 0) =
≤
≤
≤
Pλ (B(n − 1, 1 − (1 − p)t ) = t − 1)
Pλ (B(n − 1, 1 − (1 − p)t ) ≤ t − 1)
Pλ (B(n, (1 − (1 − p)t )) ≤ t)
Pλ (B(n, (1 − e−pt )) ≤ t).
Da die eindeutige Lösung in α für 1 − e−λα = α durch α = ζλ gegeben ist, prüft man
schnell nach (Übung), dass für α < ζλ und λ > 1 ein δ = δ(α, λ) > 0 existiert, so dass für
alle β ≤ α gilt:
1 − λβ ≤ e−λβ ≤ 1 − (1 + δ)β.
Sei nun X ∼ B(n, 1 − e−pt ) und t = βn, wobei wir β so wählen, dass
Dann folgt aus der vorhergehenden Ungleichungskette:
β(1 + δ)n ≤ Eλ [X] ≤ λβn.
Also:
Pλ (St ≤ 0) ≤ Pλ (X ≤ t) ≤ Pλ (X ≤ Eλ X − βδn).
87
kn
n
≤ β ≤ α gilt.
Für Binomial-verteilte Zufallsvariablen aber können wir exponentielle Schranken für die
Wahrscheinlichkeit der Abweichung vom Erwartungswert angeben (siehe Proposition 7.12
unten). Hiermit folgt
Pλ (St ≤ 0) ≤ e−βδ
Setzen wir J = J(α, λ) =
δ2
,
2λ
2 n/2λ
tδ 2
= e− 2λ .
so folgt
Pλ (kn ≤ |C(v)| ≤ αn) =
≤
≤
αn
X
t=kn
αn
X
Pλ (St = 0)
e−Jt
t=kn
−Jkn
e
.
1 − e−J
2
Proposition 7.12 Es seien X1 , . . . , Xn unabhängige Ber(pi )-verteilte Zufallsvariablen.
Dann gelten mit
n
n
X
X
X=
Xi und λ = EX =
pi
i=1
i=1
die folgenden Schranken:
−t2
P[X ≥ EX + t] ≤ exp
2(λ + t )
2 3
−t
P[X ≤ EX − t] ≤ exp
.
2λ
Beweis: Sei Y ∼ B(n, nλ ). Da “log” eine konkave Funktion ist, gilt für alle x1 , . . . , xn ∈ R
n
X
1
log(xi ) ≤ log
n
i=1
n
1X
xi
n i=1
!
(z. B. nach Jensen). Somit folgt für alle u ∈ R
EeuX =
n
Y
(1 + (eu − 1)pi )
i=1
P
n n
i=1
1
u
log(1+(e −1)pi )
n
= e
(eu −1)λ
≤ en log(1+ n )
(eu − 1)λ n
)
= (1 +
n
= EeuY .
88
Mithilfe der exponentiellen Markov-Ungleichung folgt für n ≥ 0
P(X ≥ EX + t) ≤ e−u(EX+t) EeuX
≤ e−u(EX+t) EeuY
= e−u(λ+t) (1 − p + peu )n ,
(7.8)
wobei wir wieder p = nλ und λ = EX gesetzt haben. Für t > n − λ ist die linke Seite von
(7.8) gleich 0 und die Behauptung ist trivialerweise wahr. Für λ + t < n nimmt die rechte
Seite ihr Minimum für das u an, das der Bedingung
eu =
(λ + t)(1 − p)
(n − λ − t) · p
genügt. Daraus erhält man für 0 ≤ t ≤ n − λ
λ+t n−λ−t
λ
n−λ
P(X ≥ λ + t) ≤
.
λ+t
n−λ−t
Dies ist die sogenannte Chernoff-Schranke. Für 0 ≤ t ≤ n−λ kann man sie folgendermaßen
umschreiben:
−t
t
)),
P(X ≥ λ + t) ≤ exp(−λϕ( ) − (n − λ)ϕ(
λ
n−λ
wobei
ϕ(x) = (1 + x) log(1 + x) − x (x ≥ −1)
ist. Ersetzt man X durch n − X, erhalten wir auch für 0 ≤ t ≤ n − λ
t
t
)).
P(X ≤ λ − t) ≤ exp(−λϕ( ) − (n − λ)ϕ(
λ
n−λ
Da ϕ(x) ≥ 0 für alle x gilt, können wir den zweiten Term des Exponenten vernachlässigen.
Weiter ist ϕ(0) = 0 und ϕ′ (x) = log(1 + x) ≤ x, so dass ϕ(x) ≥ x2 /2, was die zweite
Schranke beweist. Ähnlich rechnet man
ϕ(0) = ϕ′ (0) = 0
und für x ∈ [0, 1]
1
1
≥
ϕ (x) =
=
1+x
(1 − x3 )3
′′
so dass
ϕ(x) ≥
x2
2(1 + xs )
′′
,
x2
2(1 + x3 )
gilt, was die erste Ungleichung beweist.
2
Korollar 7.13 (Zu Proposition 7.11):
Sei kn = K log n und α < ζλ . Dann gibt es für hinreichend großes K mit Wahrscheinlichkeit wenigstens 1 − n−δ keine Zusammenhangskomponente der Größe zwischen kn und
αn.
89
Beweis: Die erwartete Anzahl von Komponenten mit einer Größe zwischen kn und αn
für α < ζλ ist
Eλ [Z≥kn − Z≥αn+1 ] = nPλ (kn ≤ |C(v)| ≤ αn) ≤ Cne−kn J ,
wobei wir Proposition 7.11 verwendet haben. Wenn kn = K log n ist und K hinreichend
groß, ist dies höchstens O(n−δ ). Mithilfe der Markov-Ungleichung folgt
Pλ (∃v : kn ≤ |C(v)| ≤ αn)
= Pλ (Z≥kn − Z≥αn+1 ≥ 1)
≤ Eλ [Z≥kn − Z≥αn+1 ] = O(n−δ ).
2
Um Satz 7.9 zu beweisen, fehlt uns noch eine Varianzabschätzung. Dazu sei
χ<k (λ) = Eλ (|C(v)|1l{|C(v)|<k} ).
Dann gilt
Proposition 7.14 Für alle n und jedes k ∈ {1, . . . , n} gilt
Vλ (Z≥k ) ≤ (λk + 1)nχ<k (λ).
Dies ist im superkritischen Fall viel besser als Proposition 7.8. Die Abschätzung aus
Proposition 7.8 ergäbe
Vλ (Z≥k ) ≤ nχ≥k (λ).
Wenn aber der Satz 7.9, den wir gerade beweisen sollen, stimmt, dann ist |C(1)| = Θ(n)
mit positiver Wahrscheinlichkeit. Daher ist
n · χ≥k (λ) = Θ(n2 ),
was eine triviale Schranke für VZ≥k ist. Die Schranke aus Proposition 7.14 ist dagegen
höchstens Θ(k 2 n), was für kleine k viel kleiner ist als Θ(n2 ). Wir werden wieder
k = kn = Θ(log n)
wählen.
Beweis: Sei
Z<k =
n
X
v=1
Da Z<k = n − Z≥k gilt, folgt
1l{|C(v)|<k} .
Vλ (Z≥k ) = Vλ (Z<k ).
Also genügt es zu zeigen, dass
V(Z<k ) ≤ (λk + 1)nχ<k (λ).
90
Hierzu berechnen wir
n
X
Vλ (Z<k ) =
[Pλ (|C(i)| < k, |C(j)| < k) − Pλ (|C(i)| < k)Pλ (|C(j)| < k)].
i,j=1
Wir unterteilen auf, je nachdem, ob i ↔ j oder nicht:
Vλ (Z<k ) ≤
n
X
[Pλ (|C(i)| < k, |C(j)| < k, i 6↔ j)
i,j=1
−Pλ (|C(i)| < k)P(|C(j)| < k)]
n
X
Pλ (|C(i)| < k, |C(j)| < k, i ↔ j).
+
i,j=1
Wenn i ↔ j gilt, dann ist C(i) = C(j), also auch |C(i)| = |C(j)|, also
n
X
=
i,j=1
n
X
i,j=1
=
n
X
i=1
=
n
X
i=1
Pλ (|C(i)| = |C(j)| < k, i ↔ j)
Eλ [1l{|C(i)|<k} 1l{i↔j} ]
Eλ [1l{|C(i)|<k}
n
X
j=1
1l{i↔j} ]
Eλ [|C(i)|1l{|C(i)|<k} ]
= nχ<k (λ).
Für die andere Summe schreiben wir für l < k
Pλ (|C(i)| = l, |C(j)| < k, i 6↔ j)
= Pλ (|C(i)| = l)Pλ (i 6↔ j |C(i)| = l)
×Pλ (|C(j)| < k |C(i)| = l, i 6↔ j).
Wir beschränken Pλ (i 6↔ j |C(i)| = l) ≤ 1 und erhalten:
Pλ (|C(i)| = l, |C(j)| < k, i 6↔ j) ≤ Pλ (|C(i)| = l) · P(|C(j)| < k |C(i)| = l, i 6↔ j).
Ist nun |C(i)| = l und i 6↔ j, so ist |C(j)| verteilt wie |C(1)| in einem G(n − l, p)-Graphen
mit p = nλ . Also
Pn,λ (|C(j)| < k |C(i)| = l, i 6↔ j) = Pn−l,λ(|C(1)| < k).
Daher gilt:
Pλ (|C(j)| < k |C(i)| = l, i ↔ j)
= Pn−k,λ(|C(1)| < k)
= Pn,λ(|C(1)| < k) + Pn−l,k (|C(1)| < k) − Pn,l (|C(1)| < k).
91
Wir konstruieren eine Kopplung zwischen G(n−l, p) und G(n, p), indem wir zu G(n−l, p)
die Knoten {n − l + 1, . . . , n} hinzufügen und indem wir die zusätzlichen Kanten mit
Wahrscheinlichkeit p unabhängig belegen. Mit dieser Kopplung sieht man, dass
Pn−l,λ(|C(1)| < k) − Pn,λ (|C(1)| ≤ k)
gleich der Wahrscheinlichkeit ist, dass |C(1)| < k in G(n − l, p) aber |C(1)| ≥ k in G(n, p)
ist. Wenn |C(1)| < k in G(n − l, p), aber |C(1) ≥ k in G(n, p) ist, muss wenigstens einer
der Knoten in {n − l + 1, . . . , n} mit einem der höchstens k Knoten in C(1) in G(n − l, p)
verbunden sein. Die Wahrscheinlichkeit hierfür ist höchstens lkp, so dass
Pλ (|C(j)| < k, i 6↔ j||C(i)| = l) − Pλ (|C(j)| < k) ≤
lkλ
.
n
Daher folgt
n
X
[Pλ (|C(i)| < k, |C(j)| < k, i 6↔ j) − Pλ (|C(i)| < k)Pλ (|C(j)| < k)
i,j=1
=
k−1
XX
Pλ (|C(i)| = l, |C(j)| < k, i 6↔ j) − Pλ (|C(i)| = l)Pλ (|C(j)| < l)
l=1 i,j
P(|C(i)| = l)(P(|(j)| ≤ k |C(i)| = l 6↔ j) − P(|C(j)| < k))
i,j
≤
≤
=
l=1
k−1 X
X
k−1 X
X
λkl
l=1 i,j
n
Pλ (|C(i)| = l)
λk X
Eλ [|C(i)|1l{|C(i)|<k} ]
n i,j
= nkλχ<k (λ),
was mit der obigen Abschätzung die Proposition beweist.
2
Wir sind nun in der Lage, Satz 7.9 zu beweisen.
Beweis von Satz 7.9: Sei ν ∈ ( 21 , 1) und α ∈ ( ζ2λ , ζλ ). Sei ferner kn = K log n für ein
hinreichend großes K. Sei
En := En(1) ∩ En(2)
mit
En(1) := {|Z≥kn − nζλ | ≤ nν }
und
En(2) := {∃v ∈ {1, . . . , n} : kn ≤ |C(v)| ≤ αn}.
Wir benötigen nun noch das folgende Lemma:
92
Lemma 7.15 En tritt mit großer Wahrscheinlichkeit auf, d. h.
Pλ (Enc ) ≤ cn−δ
für eine Konstante c < ∞ und ein δ > 0. Weiter gilt auf En
|Cmax | = Z≥kn .
(1)c
Beweis: Offenbar ist Enc = En
(2)c
∪ En . Nun ist nach Proposition 7.10
Eλ [Z≥kn ] = nPλ (|C(v)| ≥ kn ) = nζλ + O(kn ).
Somit folgt für hinreichend großes n, und da kn = O(log n) = o(nν ) gilt:
{|Z≥kn − Eλ [Z≥kn ]| ≤ nν /2} ⊆ {|Z≥kn − nζλ | ≤ nν }.
Mithilfe der Chebyshev-Ungleichung und Proposition 7.14 zusammen mit
χ≤kn (λ) ≤ kn
erhalten wir für großes n:
Pλ (|Z≥kn − E[Z≥kn ]| ≤ nν /2)
1 − 4n−2ν V(Z≥kn )
1 − 4n1−2ν (λkn2 + kn )
1 − n−δ
Pλ (|Z≥kn − nζλ | ≤ nν ) ≥
≥
≥
≥
(1)c
für δ < 2ν − 1, da kn = K log n gilt. Dies beschreibt die Wahrscheilichkeit von En .
Weiter ist nach Korollar 7.13
Pλ (∃v ∈ {1, . . . , n} : kn ≤ |C(v)| ≤ αn) ≤ n−δ .
(2)c
Also ist auch die Wahrscheinlichkeit von En
klein, zusammen also
Pλ (Enc ) = O(n−δ ).
Um die zweite Aussage herzuleiten, bemerken wir, dass
{|Z≥kn − ζλ n| ≤ nν } ⊆ {Z≥kn ≥ 1}.
Also ist auf En
|Cmax | ≤ Z≥kn ,
denn da es Cluster der Größe mindestens kn gibt, liegt jeder Punkt in Cmax in einem
Cluster der Größe ≥ kn . Ist diese Ungleichung strikt, d. h. gilt
|Cmax | < Z≥kn ,
dann gibt es mindestens zwei Zusammenhangskomponenten der Größe mindestens kn . Auf
En gibt es aber keine Zusammenhangskomponenten mit einer Größe zwischen kn und αn.
Also muss es zwei solche Cluster mit einer Größe von mindestens αn geben, also muss
Z≥kn ≥ αn
93
gelten. Ist nun 2α > ζλ und n hinreichend groß, so widerspricht dies
Z≥kn ≤ ζλ n + nν .
Also gilt die Behauptung.
2
Wir beenden nun den Beweis von Satz 7.9. Nach Lemma 7.15 gilt
Pλ (|Cmax | − ζλ n| ≤ nν ) ≥ Pλ (|Cmax| − ζλ n| ≤ nν ∩ En )
= Pλ (En ) ≥ 1 − O(n−δ ),
da nach Lemma 4.15 auf En |Cmax | = Z≥kn und |Z≥an − nζλ | ≤ nν gilt. Dies beweist Satz
7.9.
2
Eine interessante Frage, die beinahe auf der Hand liegt, ist die folgende: Wir betrachten
eine Zufallsgröße G(n, p) mit p = nλ und λ > 1. Dann wissen wir aus dem Inhalt dieses
Kapitels, dass G(n, p) mit großer Wahrscheinlichkeit eine riesige Komponente der Größe
ζλ n enthält. Wie sieht nun der Graph aus, wenn wir diese entfernen? Die Antwort ist,
dass wir dann in einen subkritischen Bereich geraten. Genauer:
Satz 7.16 (Diskretes Dualitätsprinzip)
Es sei λ > 1 und µ = µλ < 1 zu λ dual in dem Sinn, dass
µe−µ = λ e−λ
gilt. Die bedingte Verteilung des Graphen G(n, nλ ), bei dem man die riesige Komponente
µ
entfernt, ist nahe an der Verteilung eines G(m, m
)-Modells, wobei m := n − ⌈nζλ ⌉ die
asymptotische Anzahl an Knoten ist, die außerhalb der riesigen Komponente liegen.
Bemerkung 7.17 Mit der Formulierung “die Verteilung . . . liegt dicht bei der Verteilung
. . .” meinen wir das folgende: Wir schreiben P′λ für die Verteilung des G(n, nλ ), bei dem wir
die riesige Komponente entfernt haben. Sei E ein Ereignis, das über die Kantenvariablen
definiert ist. Dann gilt, falls limm→∞ Pm,µ (E) existiert:
lim P′n,λ (E) = lim Pm,µ (E).
n→∞
m→∞
Beweisskizze für Satz 7.16: Bemerke, dass alle Kanten im Komplement der riesigen
Komponente unabhängig sind. Wir berechnen nun die Wahrscheinlichkeit, dass so eine
Kante auftritt. Diese ist natürlich nach wie vor nλ . Dies schreiben wir für |Cmax | = n − m
als
λ m
λ
=
· .
n
m n
Nun ist m mit großer Wahrscheinlichkeit ungefähr ζλ n, also
λ
ληλ
µ
≈
=
n
m
m
(erinnere, dass ηλ = 1−ζλ ist). Andererseits auch n−m ≈ ηλ ·n, d. h. wir haben tatsächlich
µ
)-Graph.
2
einen G(m, m
94
8
Der kritische Erdös-Rényi-Graph
Wir haben in Kapitel 7 gesehen, das es einen Phasenübergang im Erdös-Rényi-Graphen
gibt: Während für p = nλ , λ < 1, die Größe der größten Zusammenhangskomponente bei
etwa
1
|Cmax | ∼
log n
=
Iλ
mit
Iλ = λ − 1 − log λ
liegt, ist sie für λ > 1 dicht bei
|Cmax | ∼
= ζλ n,
wobei ζλ die Überlebenswahrscheinlichkeit eines Poisson(λ)-Verzweigungsprozesses ist.
Die Frage, was bei p = n1 geschieht, liegt nahe. Die Antwort darauf gibt
Satz 8.1 Für p = n1 gilt: Es gibt eine Konstante b > 0, so dass für hinreichend großes n
und alle w > 1 gilt
1
b
P( n2/3 ≤ |Cmax | ≤ wn2/3 ) ≥ 1 − .
w
w
Der Beweis dieses Satzes wird uns in diesem Abschnitt beschäftigen. Der Beweis benutzt
Abschätzungen über die erwartete Clustergröße und die Tails der Verteilung der Clustergröße. Diese werden wir zunächst formulieren und dann später beweisen. Sei
P≥k (λ) := Pλ (|C(v)| ≥ k).
Proposition 8.2 Sei λ = 1. Für k ≤ rn2/3 gibt es Konstanten
0 < c1 = c1 (v) < c2 < ∞
mit
min{c1 (r) : r ≤ κ} > 0
für ein κ > 0 und ein von r unabhängiges c2 , so dass für alle hinreichend großen n gilt
c2
c1
√ ≤ P≥k (λ) ≤ √ .
k
k
Proposition 8.2 bedeutet, dass sich die Tails der kritischen Clustergrößen-Verteilung ähnlich verhalten wie die Tails der Gesamtgröße eines P oi(λ)-Galton-Watson-Baumes. Für
diesen hatten wir
r
1
2 1
∗
√ (1 + O( ))
P1 (T ≥ k) =
π k
k
hergeleitet. Natürlich kann Proposition 8.2 nicht wahr sein, wenn k beliebig von n abhängen
darf. Für k > n gilt ja z. B.
P≥k (1) = 0.
95
Tatsächlich ist die obere Schranke aber für alle k wahr. Für die untere Schranke hingegen gibt es eine Schwelle, wo diese zusammenbricht. Diese liegt bei rn2/3 . Als nächstes
betrachten wir
Eλ [|C(1)|] = χ(λ).
Proposition 8.3 Es gibt ein K > 0, so dass für alle λ ≤ 1 und alle n ∈ N gilt
χ(λ) ≤ Kn1/3 .
Dies ist auf intuitiver Ebene konsistent mit Satz 8.1. Tatsächlich sollte ja ein wesentlicher
Beitrag zu χ(1) vom größten Cluster kommen. Also
χ(1) ∼ E1 [|C(v)|1l{v∈Cmax } ]
= E1 [|Cmax |1l{v∈Cmax } ].
Wenn tatsächlich Satz 8.1 stimmt, dann ist |Cmax | = Θ(n2/3 ), dann ist
P1 (v ∈ Cmax ) ∼
n2/3
= n−1/3 .
n
Daher sollte man intuitiv erwarten, dass
χ(1) ∼ n1/3 .
Wir beweisen zunächt Satz 8.1 unter Voraussetzung von Proposition 8.2 und 8.3.
Beweis von Satz 8.1: Wir beginnen mit der oberen Schranke für |Cmax |. Wir erinnern,
dass
{|Cmax | ≥ k} = {Z≥k ≥ k}
Pn
gilt, wobei wieder Z≥k = v=1 1l{|C(v)|≥1} ist. Mithilfe der Markov-Ungleichung erhalten
wir
1 1
P1 [|Cmax | ≥ wn2/3 ] = P[Z≥wn2/3 ≥ wn2/3 ] ≤
E1 [Z≥wn2/3 ].
w n2/3
Nach Proposition 8.2 lässt sich dies nun abschätzen als
c2
E1 [Z≥wn2/3 ] = nP≥wn2/3 (1) ≤ n2/3 √ .
w
Also
P1 [|Cmax | ≥ wn2/3 ] ≤
c2
,
w 3/2
was für große w ≥ 1 sogar stärker ist als behauptet.
Für die untere Schranke bemerken wir zunächst, dass für w < b nichts zu zeigen ist. b > 0
werden wir groß wählen, so dass wir für w > b w > κ1 , für das κ > 0 aus Proposition 8.2,
annehmen können.
96
Die Chebyshev-Ungleichung, zusammen mit
{|Cmax | < k} = {Z≥k = 0}
ergibt
P1 (|Cmax | <
Nun ist
V(Z≥w−1n2/3 )
1 2/3
n ) = P1 (Z≥w−1n2/3 = 0) ≤
.
w
E[Z≥w−1 n2/3 ]2 )]
√
E1 [Z≥w−1 n2/3 ] = nPgew−1 n2/3 (1) ≥ c1 wn2/3 ,
wobei wir Proposition 8.2, w ≥ κ1 und c1 = minr≤κ c1 (r) > 0 benutzt haben. Nun haben
wir V(Z≥k ) schon (zweimal) in Kapitel 7 abgeschätzt. Wir benutzen Proposition 7.7 um
zu erhalten:
V(Z≥w−1n2/3 ) ≤ nχ≥w−1 n2/3 (1) = nE[|C(1)|1l{|C(1)|≥ 1 n2/3 } ].
w
Mithilfe von Proposition 8.3 erhalten wir also weiter:
V(Z≥w−1n2/3 ) ≤ nχ≥w−1 n2/3 (1) ≤ nχ(1) ≤ Kn4/3 .
Wenn wir diese Abschätzungen zusammenfügen, ergibt sich:
P1 (|Cmax | <
1 2/3
Kn4/3
K
n ) ≤ 2 4/3 = 2 .
w
c1 wn
c1 w
Also ergibt sich
1
P1 ( n2/3 ≤ |Cmax | ≤ wn2/3 )
w
= 1 − P1 (|Cmax | < w −1 n2/3 ) − P1 (|Cmax | > wn2/3 )
c2
b
K
≥ 1 − 2 − 3/2 ≥ 1 − ,
c1 w w
w
wenn wir b = Kc21 + c2 setzen.
2
Bleibt Proposition 8.2 und 8.3 zu beweisen.
Beweis von Proposition 8.2: Sei λ ≤ 1. Der Vergleich zwischen G(n, p) und einem
Binomialbaum ergibt wieder
P≥k (λ) ≤ Pn,p (T ≥ k),
wobei sich die rechte Seite wieder auf einen B(n, p)-Galton-Watson-Baum mit p = nλ
bezieht. Vergleichen wir diesen wieder mit einem P oi(λ)-Galton-Watson-Baum, so sehen
wir
P≥k (λ) ≤ P∗λ (T ∗ ≥ k) + ek (n)
mit
k
2X ∗ ∗
P (T ≥ s).
|ek (n)| ≤
n j=1 λ
97
Nun haben wir die Tails eines kritischen P oi(1)-GW-Baumes abgeschätzt:
C
P∗λ (T ∗ ≥ s) ≤ P∗1 (T ∗ ≥ s) ≤ √
s
für C > 0.
Dies ergibt eine Schranke für |ek (n)|:
Da auch P∗λ (T ∗ ≥ s) ≤
√
k
k
4C
2X C
√ ≤ 4C
≤√ .
|ek (n)| ≤
n s=1 s
n
k
C
√
,
k
erhalten wir
5C
P≥k (λ) ≤ √ .
k
Für λ = 1 zeigt dies die obere Schranke in Proposition 8.2. Für die untere Schranke
benutzen wir
P1 (|C(1)| ≥ k) ≥ Pn−p,k (T ≥ k),
wobei sich die rechte Seite wieder auf ein entsprechendes Binomial-Baum-Modell bezieht.
Wir wählen k ≤ rn2/3 und p = n1 . Vergleichen wir die rechte Seite wieder mit einem
Poisson-Galton-Watson-Prozess, so ergibt sich mit λn = 1 − rn−1/3 ≥ 1 − nk und obiger
Fehlerschranke
√
√
4C k
4C r
∗
∗
∗
∗
P1 (|C(1)| ≥ k) ≥ Pλn (T ≥ k) −
≥ Pλn (T ≥ k) − 2/3 .
n
n
Verwenden wir nun Satz 6.2, so erhalten wir
√
∞
X
4C r
∗
∗
Pλn (T = t) − 2/3
P(|C(1)| ≥ k) ≥
n
t=k
√
X (λn t)t−1
4c r
−λn t
e
− 2/3
=
t!
n
√
X
4C r
∗
∗
−Iλn t
− 2/3 ,
≥
P1 (T = k)e
n
t≥k
wobei
1
Iλn = λn − 1 − log λn = (λn − 1)2 + O(|λn − 1|)3 .
2
Übung 8.4 Man zeige die letzte Ungleichung, genauer:
(λt)t−1 −λt
1
e
= e−Iλ t P∗1 (T ∗ = t).
t!
λ
Also gilt für hinreichend großes n
Pλ (|C(1)| ≥ k) ≥
≥
∞
X
t=k
∞
X
t=k
P∗1 (T ∗
− 21 (λn −1)2 t(1+o(1))
= t)e
√
C − 1 (λn −1)t(1+o(1)) 4C r
e 2
− 2/3
t3
n
C1 (r)
≥ √ ,
k
98
√
4C r
− 2/3
n
da λn − 1 = −rn−1/3 , und wobei wir
√
3
c1 (r) = C(2−3/2 e−r − e r) > 0
für hinreichend kleines r gesetzt haben.
2
Der Beweis von Proposition 8.3 ist zu lang, um hier gezeigt zu werden.
99
9
Der Zentrale Grenzwertsatz für die riesige
Komponente
In diesem kurzen Abschnitt wollen wir eine sehr natürliche Frage beantworten, die sich
|
aus Satz 7.9 ergibtr. Dort haben wir festgestellt, dass |Cmax
gegen ζλ konvergiert im Sinne
n
des Gesetzes der großen Zahlen. Aus wahrscheinlichkeitstheoretischer Sicht ist die nächstliegende Frage nun die nach einem Zentralen Grenzwertsatz. Dieser wird nun bewiesen.
Satz 9.1 (CLT für die Größe des riesigen Komponente)
Sei λ > 1. Dann gilt
|Cmax | − ζλ n D
√
−→ Z,
n
wobei Z ∼ N (0, σλ2 )-verteilt ist und
σλ2 =
ζλ (1 − ζλ )
.
(1 − λ + λζλ )2
Der Beweis wird Satz 7.9 und eine Art Explorationsprozess für die Zusammenhangskomponenten verwenden.
Wir wählen zunächst k = kn , der wir später spezifizieren werden. Wir untersuchen mit
einem Explorationsprozess die Vereinigung der Zusammenhangskomponenten der Vertizes
{1, . . . , k}. Wenn k → ∞ geht, wird dies die größte Zusammenhangskomponente enthalten
und sie kann nicht größer sein als |Cmax | + kbn , wobei wir mit
bn ≤ K log n
die Größe der zweitgrößten Zusammenhangskomponente beschränken. Wenn also √
k =
o(nν ) für ein ν < 21 ist, dann hat diese Vereinigung eine Größe von |Cmax | + o( n).
Können wir also einen Zentralen Grenzwertsatz für die Größe dieser Vereinigung ableiten,
dann haben wir auch einen für |Cmax |.
Wir leiten nun eine Formel für die Größe der Vereinigung der Zusammenhangskomponenten von {1, . . . , k} her.
Sei S1 die Anzahl an Knoten, die aus {k+1, . . . , n} stammen und mit {1, . . . , k} verbunden
sind. Man überzeugt sich, dass
S1 ∼ B(n − k, 1 − (1 − p)k )
gilt. Für m ≥ 1 sei
Sm = Sm−1 + Xm−1 ,
wobei
Xm ∼ B(n − Sm−1 − (m + k − 1), p).
Einen ähnlichen Explorationsprozess haben wir schon kennengelernt. Dann gilt
100
(9.1)
Proposition 9.2 Für alle t ∈ {1, . . . , n} gilt
St + (t − 1) ∼ B(n − k, 1 − (1 − p)t+k−1 ).
Weiter gilt für alle l, m ∈ {1, . . . , n} mit l ≥ m bedingt auf Sm
Sl + (l − m) − Sm ∼ B(n − (m + k − 1) − Sm , (1 − (1 − p)l−m )).
Für k = 1 haben wir dies schon in einer früheren Proposition kennengelernt.
Beweis: Für t = 1 folgt alles aus der Formel für S1 . Für t ≥ 1 sei Nt die Anzahl der noch
nicht untersuchten Knoten, also
Nt = n − (t + k − 1) − St .
Es ist bequemer, die äquivalente Behauptung
Nt ∼ B(n − k, (1 − p)t+k−1 ) ∀ t
zu zeigen. Um dies einzusehen, bemerke, dass jeder der Knoten {k + 1, . . . , n} unabhängig
von allen anderen Knoten Wahrscheinlichkeit (1 − p)t+k−1 hat, in den ersten t Explorationen neutral zu bleiben. Formal: Bedingt unter St−1 gilt
Xt ∼ B(n − St−1 − (t + k − 2), p) = B(Nt−1,p )
nach (9.1). Behält man im Hinterkopf, dass
N1 ∼ B(Nm , (1 − p)l−m ),
so ergibt sich:
Nt =
=
=
=
=
n − (t + k − 1) − St
n − (t + k − 1) − St−1 − Xt + 1
n − (t + k − 2) − St−1 − B(Nt−1 , p)
Nt − 1 − B(Nt−1 , p)
B(Nt−1 , 1 − p)
und die Behaputung folgt induktiv mithilfe der folgenden Übung. Für l ≥ m impliziert
diese Rechnung auch:
Nl ∼ B(Nm , (1 − p)l−m ).
Setzt man Nm = n − (m + k − 1) − Sm ein, so folgt hieraus
n − (l + k − 1) − Sl ∼ B(n − (n + k − 1) − Sm , (1 − p)l−m )
= n − (m + k − 1) − Sm − B(n − (m + k − 1) − Sm , 1 − (1 − p)l−m ),
was wiederum äquivalent zu der Behauptung ist, dass für alle l ≥ m und bedingt unter
Sm
Sl + (l − m) − Sm ∼ B(n − (m + k − 1) − Sm , 1 − (1 − p)l−m )
gilt.
2
101
Übung 9.3 Ist N ∼ B(n, p)-verteilt und M bedingt auf N, M ∼ B(N, q)-verteilt, dann
ist M ∼ B(n, pq)-verteilt.
Eine Folge aus Proposition 9.2 ist, dass S⌊nt⌋ einem CLT genügt. Wir setzen S0 = k
und machen bei der Formulierung Gebrauch von der asymptotischen Approximation für
Mittelwert und Varianz von S⌊nt⌋
µt = 1 − t − eλt
und
vt = e−λt (1 − e−λt ).
√
Korollar 9.4 Sei k = kn = o( n). Dann konvergiert für jedes t ∈ [0, 1]
S⌊nt⌋ − nµt D
−→ N (0, 1).
√
nvt
Beweis: Die Behauptung folgt sofort aus einem CLT für die Binomialverteilung (X ∼
B(an , pn ) mit an pn (1 − pk ) → ∞), wenn wir zeigen können, dass
√
und
(9.2)
ES⌊nt⌋ = nµt + o n
VS⌊nt⌋ = nvt + o(n)
gilt, denn
S⌊nt⌋ − nµt
=
√
nvt
s
V(S⌊nt⌋ ) S⌊nt⌋ − E[S⌊nt⌋ ] E[S⌊nt⌋ ] − nµt
p
+ p
.
nvt
V(S⌊nt⌋ )
V(S⌊nt⌋ )
Nach (9.2) konvergiert der zweite Summand gegen 0 und der Varfaktor gegen 1. Nach
dem gewöhnlichen CLT konvergiert
S⌊nt⌋ − ES⌊nt⌋
p
V[S⌊nt⌋ ]
gegen die Standardnormalverteilung. Um (9.2) einzusehen, bemerken wir für den Erwartungswert, dass
√
λ
E[S⌊nt⌋ ] = (n − k)(1 − (1 − )⌊nt⌋+k−1 ) − (⌊nt⌋ − 1) = nµt + o( n)
n
und für die Varianz, dass
V[S⌊nt⌋ ] = (n − k)(1 −
λ
λ ⌊nt⌋+k−1
)
(1 − (1 − )⌊nt⌋+k−1 ) = nvt 0o(n)
n
n
√
gilt, so lange k = o( n) bzw. k = o(n) gilt.
Nun beweisen wir Satz 9.1.
102
2
Beweis von Satz 9.1: Sei |C≤k | die Größe der Vereinigung der Zusammenhangskomponenten der Knoten 1, . . . , k. Dann gilt
|C≤k | ∼ min{m : Sm = 0}.
(9.3)
Sei k = kn = log n. Dann folgt aus Satz 7.9, dass die Wahrscheinlichkeit, dass keiner der
ersten kn Knoten in der größten Zusammenhangskomponenten liegt, von oben abgeschätzt
werden kann durch
"
k #
n − |Cmax | n
Eλ
= o(1).
n
Somit gilt mit großer Wahrscheinlichkeit |C≤k | ≥ |Cmax |. Andererseits folgt aus Korollar
7.13 und Satz 7.9, das mit großer Wahrscheinlichkeit das zweitgrößte Cluster eine Größe
von höchstens k log n (für ein großes k > 0) hat, falls 2α > ζλ ist. Also gilt mit großer
Wahrscheinlichkeit
|C≤k | ≤ |Cmax | + (k − 1)K log n.
Also folgt ein CLT für |Cmax | aus einer solchen für |C≤k | mit k = log n. Diesen CLT für
|C≤k | beweisen wir durch obere und untere Schranken an
Pλ
|C≤k | − ζλ n
√
≤x .
n
Für die obere Schranke verwenden wir, dass (9.3) impliziert, dass für jedes l
Pλ (|C≤k | > l) = Pλ (∀ m ≤ l : Sm > 0)
gilt. Wendet man (9.4) auf
(9.4)
√
l = mx = ⌊nζλ + x n⌋
an, erhält man
|C≤k | − ζλ n
√
> x = Pλ (∀ m ≤ mX : Sm > 0) ≤ Pλ (Smx > 0).
Pλ
n
Nun verwenden wir (9.2) und µζλ = 0 und sehen (für die Ableitung µ′t von µt nach t):
√
√
E[Smx ] = nµζλ + nxµ′ζλ + o( n)
√
√
=
nx(λe−λζλ − 1) + o( n)
(bemerke, dass λe−λζλ − 1) < 0 für λ > 1 gilt.
Übung 9.5 Man zeige, dass für λ > 1
µ′ζλ = λe−λζλ − 1 < 0
und µζλ = 0 gilt.
103
Man berechnet mithilfe von (9.2) die Varianz von Smx als
V(Smx ) = nvζλ + o(n).
Somit ergibt sich
Pλ (Smx > 0) = Pλ
x(1 − λe−λζλ )
Smx − E(Smx )
p
>
√
vζλ
V(Smx
!
+ o(1).
(9.5)
Nach Korollar 9.4 konvergiert die rechte Seite gegen
x(1 − λe−λζλ )
P Z>
= P(Z ′ > x),
√
vζλ
v
λ
verteilt sind. Schließlich ist
wobei Z ∼ N (0, 1) und Z ′ ∼ N (0, (1−λeζ−λζ
λ )2
ζλ = λ − ηλ ,
also
1 − ζλ = e−λζλ ,
so dass
vζλ = e−λζλ (1 − e−λζλ ) = ζλ (1 − ζλ )
gilt. Also lässt sich die Varianz von Z ′ umschreiben zu
ζλ(1 − ζλ )
vζλ
=
.
2
−λζ
λ
)
(1 − λe
(1 − λ + λζλ )2
Dies ergibt die obere Schranke wegen (9.5). Für die untere Schranke benutzen wir wieder,
dass
Pλ (|C≤k | − ζλ > x) = Pλ (∀ m ≤ mx : Sm > 0)
√
gilt, wobei wir wieder mx = ⌊nζλ + x n⌋ gesetzt haben. Dann gilt für alle ε > 0
√
Pλ (∀ m ≤ mx : Sm > 0) ≥ Pλ (∀ m < mx : Sm > 0Smx > ε n)
√
√
= Pλ (Smx > ε n) − Px (Smx > ε n, ∃m < mx : Sm = 0).
Der erste Term kann ähnlich wie bei der oberen Schranke behandelt werden. Tatsächlich
erhält man exakt so wie dort für jedes ε > 0
√
x(1 − λe−λζλ ) + ε
Pλ (Smx > ε n) = P(Z >
+ o(1).
√
vζλ
Wieder gilt für ε → 0, dass die rechte Seite gegen P(Z ′ > x), für Z ′ ∼ N (0, σλ2 ). Somit
genügt es zu zeigen, dass
√
Pλ (Smx > ε n, ∃ m < mx : Sm = 0) = o(1).
Mithilfe der Bollschen Ungleichung folgt:
√
Pλ (Smx > ε n, ∃ m < mx : Sm = 0) ≤
m
x −1
X
√
Pλ (Sm = 0, Smx > ε n).
m=1
Für m ≤ αn mit α < ζλ kann man zeigen, dass, wenn k = K log n und K hinreichend
groß, gleichmäßig in m ≤ αn gilt
1
(9.6)
Pλ (Sm = 0) = o( ).
n
104
Übung 9.6 Man beweise (9.6).
Wir zeigen eine ähnliche Schranke für m > αn, wobei α < ζλ ist (und beliebig dicht bei
ζλ gewählt werden kann). Wir benutzen hierbei, dass für m dicht bei nζλ gilt EXm < 1,
so dass wir uns Sm in m nahezu als Irrfahrt mit negativer Drift
√vorstellen können. Daher
ist die Wahrscheinlichkeit dafür, dass Sm = 0, aber Smx > ε n, exponentiell klein. Im
einzelnen:
√
√
Pλ (Sm = 0, Smx > ε n) ≤ P(Smx > ε n|Sm = 0)
√
= Pλ (B(n − (m + k − 1), 1 − (q − p)mx −m ) > (mx − m) + ε n),
da nach Proposition 9.2 bedingt unter Sm = 0
Sl + (l − m) ∼ B(n − (m + k − 1), (1 − (1 − p)l−m ))
verteilt ist. Wähle
κ = ζλ − ε
für ein sehr kleines ε > 0. Unter Ausnutzung von
1 − (1 − a)b ≤ ab
für alle a, b mit 0 < a < 1, b ≥ 1, erhalten wir 1 − (1 − p)mx −m = (1 − (1 − nλ )mx −m ) ≤
λ(mx −m)
. Somit haben wir für
n
X ∼ B(n − (m + k − 1), (1 − (1 − p)mx −m )),
unter Ausnutzung von n − (m + k − 1) ≤ n − m ≤ n(1 − ζλ + ε) und p =
λ
n
EX ≤ (1 − ε)(mx − m).
Daher folgt
√
√
Pλ (Sm = 0, Smx > ε n) ≤ Pλ (X − EX ≥ ε((mx − m) + n)).
Benutzt man wieder Proposition 7.12 über die√Abschätzung der Tails der Binomialverteilung, erhält man dann für t = ε((mx − m) + n)
√
t2
Pλ (Sm = 0, Smx > ε n) ≤ exp −
2((1 − ε)(mx − m) + 3t )
!
t2
√
≤ exp −
.
2((mx − m) + 2ε3 n )
√
Daher folgt für mx − m ≥ ε n, da t ≥ ε(mx − m),
√
√
√
1
Pλ (Sm = 0, Smx > ε n) ≤ exp(−3ε n|8) = exp(−ε n/2) = o( ).
n
Dies beschließt den Beweis des Zentralen Grenzwertsatzes.
105
2
10
Inhomogene Zufallsgraphen
Der G(n, p)-Zufallsgraph ist gewissermaßen die homogenste Art, einen Zufallsgraphen zu
konstruieren. Wir wollen hier nun das zugrunde liegende Modell ein wenig ändern und
sehen, welche Auswirkungen das hat. Wir wollen zu diesem Zweck den Knoten Gewichte
zuweisen. Die Kanten werden dann, gegeben diese Gewicht, unabhängig gewählt. Die
Gewichte selbst können deterministisch sein oder selbst zufällig. Sei wi das Gewicht von
Knoten i. Die Wahrscheinlichkeit, eine Kante zwischen Knoten i und j zu legen, ist nun
gegeben durch
wi wj
(GRG)
pij = pij
=
,
ln + wi wj
wobei ln das Gesamtgewicht ist:
ln =
n
X
wi .
i=1
Das entstehende Modell nenne wir HRG (w) (generaoized random graph mit Gewicht
w). Hierbei nehmen wir an, dass wi > 0 gilt (alle i mit wi = 0 wären mit Wahrscheinlichkeit isoliert). Sind die (wi ) selbst Ergebnis von Zufallsvariablen (Wi ), so schreiben wir
GRG(W). Sind die (wi) i.i.d. mit Erwartungswert µ, so kann man auch ln durch n ersetzen
und kommt zu den Gewichten
1
Wi Wj
Wi Wj
µ
.
pij =
=
µn + wi wj
n + µ1 wi wj
Ein Spezialfall ist der GRG mit Gewichten
wi =
nλ
,
n−λ
womit man pij = nλ , also das G(n, p)-Modell mit p =
λ
n
erhält.
Wir beginnen damit, im Modell GRG(w) die Gradfolge zu studieren. Es sei Dk der Grad
von Knoten k. Dann gilt
(n)
Satz 10.1
a) Für feste Gewichte w = (wi ) gibt es eine Poisson-Zufallsvariable Zk
(n)
mit Zk ∼ P oi(wk ), so dass
(n)
n
(n)
X
(wj )2
(wk )2
P(Zk =
6 Dk ) ≤
(1 + 2
).
ln
ln
j=1
Insbesondere konvergiert Dk in dem Fall, dass
(n)
wk = lim wk
n→∞
existiert, gegen eine P oi(wk )-verteilte Zufallsvariable, falls
n
X
(n)
(wi )2 = o(ln2 ).
i=1
106
b) Gilt für die Kantenwahrscheinlichkeiten pij
lim pij = 0,
n→∞
so sind die Grade D1 , . . . , Dm asymptotisch unabhängig.
Um ein Korollar hieraus zu formulieren, benötigen wir noch eine Definition.
Definition 10.2 Eine Zufallsvariable X hat eine gemischte Poissonverteilung mit Mischungsverteilung F , falls
wk
P[X = k] = E[e−w ]
k!
und W eine Zufallsvariable mit Verteilungsfunktion F .
Korollar 10.3 Sind die Gewichte im GRG-Modell gegeben durch w = (wi )ni=1 und
i
wi = (1 − Fw )−1 ( )
n
für eine Verteilungsfunktion Fw mit endlichem Erwartungswert, dann gilt
a) Der Grad eines mit Gleichverteilung gezogenen Knotens konvergiert in Verteilung
gegen eine gemischte Poisson-Verteilung mit Mischungsverteilung Fw .
b) Die Grade mit Gleichverteilung auf {1, . . . , n} gezogener Knoten sind asymptotisch
unabhängig.
Beweis von Satz 10.1: Wir werden wieder die schon in Kapitel 5 vorgestellte Kopplung
von Poisson- und Binomial-verteilten (genauer Summe von Bernoulli-Zufallsvariablen)
Zufallsvariablen benutzen. Wir unterdrücken das (n) in der Schreibweise und schreiben
(n)
wi = wi . Es ist
n
X
Dk =
Xkj ,
j=1
wobei Xkj der Indikator für die kj-te Kante ist. Da Xkj ∼ Ber(pkj mit pkj =
es mithilfe der Kopplung eine P oi(λk )-Zufallsvariable Ŷk , wobei
λk =
X
j=k
wk wj
ln + wk wj
und eine Zufallsvariable D̂k mit
D
D̂k = Dk ,
107
wk wj
ln+wk wj
gibt
so dass
P(D̂k 6= Ŷk ) ≤
X
p2kj
=
X
j 6= k
≤
wk2
j6=k
wk2 wj2
(ln + wk wj )2
n
X
wj2
.
2
ln
j=1
Also genügt es, um die Behauptung zu beweisen, Ŷk an eine P oi(wk )-verteilte Zufallsvariable Ẑk so zu koppeln, dass
P(Ŷk 6= Ẑk ) ≤ wk2
n
X
wj2
ln2
j=1
gilt. Hierzu bemerken wir, dass
λk ≤
X wk wj
ln
j=k
k
wk X
wj = wk
≤
ln j=1
gilt. Daher ist εk := wk − λk ≥ 0. Sei
Vk ∼ P oi(εk )
unabhängig von Ŷk und schreiben
Ẑk = Ŷk + V̂k .
Dann ist
P(Ŷk 6= Ẑk ) = P(V̂k 6= 0) = P(V̂k ≥ 1) ≤ E[V̂k ] = εk .
Wir wollen daher εk beschränken:
wk wj
ln + wk wj
j6=k
n
X
1
wk2
1
=
wk wj
+
−
ln lnwk wj
ln + wk2
j=1
ε k = wk −
=
n
X
j=1
X
wj2 wk2
wk2
+
ln(ln + wk wj ) ln + wk2
n
wk2 X wj2wk2
+
≤
ln
ln2
j=1
= wk2
n
X
wj2
1+
ln2
j=1
!
.
Also
P(D̂k 6= Ẑk ) ≤ P(D̂k 6= Ŷk ) + P(Ŷk 6= Ẑk ) ≤
108
2wk2
n
X
wj2
wk2
+
ln2
ln
j=1
wie benötigt. Somit ist der erste Teil des Satzes bewiesen.
Um den zweiten Teil herzuleiten, genügt es zu zeigen, dass man (D1 , . . . , Dm ) an einen
unabhängigen Vektor (D̂1 , . . . , D̂m , so dass
P((D1 , . . . , Dm ) 6= (D̂1 , . . . , D̂m )) → 0.
(10.1)
Sei wieder Xij der Indikator dafür, ob die Kante (i, j) gesetzt ist. Die Zufallsvariablen
(Xij ) sind natürlich unabhängig mit Parameter (pij ). Seien (Xij′ ) unabhängige (Bernoulli-)
Variablen, die unabhängige Kopien von (Xij ) sein sollen. Sei
Di′
=
X
Xij′
+
j<i
Offenbar ist
n
X
Xij .
j=i+1
D
Di = Di′ .
Während Di und Dj unabhängig sind (beide enthalten Xij ), sind Di′ und Dj′ unabhängig
(denn das eine enthält Xij′ , das andere Xji = Xij ). Also ist
′
(D1′ , . . . , Dm
)
unabhängige Summe unabhänger Bernoulli-Variablen. Nun ist
(D1 , . . . , Dm ) 6= (D̂1 , . . . , D̂m )
dann, und nur dann, wenn es i, j ∈ {1, . . . , m} gibt mit
Xij 6= Xij′ ,
d. h. entweder ist Xij = 0 und Xij′ = 1 oder andersrum?? Also
P((D1 , . . . , Dm ) 6= (D̂1 , . . . , D̂m )) ≤ 2
m
X
P(xij = 1) = 2
i,j
m
X
pi,j .
i,j=1
Nach Voraussetzung ist pij −→ 0, so dass (9.1) gilt. Also folgt die zweite Behauptung
n→∞
und damit der ganze Satz.
2
Der Beweis von Korollar 10.3 gestaltet sich erstaunlich aufwändig.
Beweis von Korollar 10.3: Man überlegt sich zunächst, dass x 7→ (1 − Fw )−1 (x) nichtwachsend, so dass
n
n
1X
i
1X 2
wi =
(1 − Fw )−1 ( )2
n i=1
n i=1
n
n
≤ (1 − Fw )−1 (
109
i
11X
(1 − Fw )−1 ( ).
n n i=1
n
Nun behaupten wir, dass
1
(1 − Fw )−1 ( ) = o(n),
n
−1 1
da dies äquivalent ist zu (1 − Fw ) ( n ) ≤ an für jedes ε > 0 und n hinreichend groß. Dies
wiederum ist äquivalent zu
1
Fw (ε) ≥ 1 − ,
n
d. h.
1 − Fw (εn) = P(w1 > εn) ≤ Yn .
Nun gilt aber sogar
1
P(w1 > εn) = o( ).
n
Außerdem ist (Übung):
n
1X
i
(1 − Fw )−1 ( ) ≤ E[Fw−1 (U)] = EW,
n i=1
n
wobei W die Verteilungsfunktion F hat. Da EW < ∞ ist, folgt
n
1
1X 2
wi ≤ Fw−1 (1 − )EW = o(n),
n i=1
n
da ln = Θ(n). Mithilfe von Satz 10.1a) können wir daher schließen, dass der Grad jedes
(n)
(n)
(n)
Knotens, für den wk beschränkt, annähernd P oi(wk )-verteilt ist. Da wB(n) mit großer
Wahrscheinlichkeit beschränkt ist, können wir Satz 9.1a) anwenden.
Da eine gemischte Poisson-Zufallsvariable gegen eine Limes-gemischte Poisson-Zufallsvariable
konvergiert, wenn die Mischugnsverteilungen in Verteilung konvergieren, genügt es zu zeigen, dass das Gewicht eines nach der Gleichverteilung gezogenen Vertex eine Limesverteilung hat, die durch F gegeben ist.
Sei D der Grad eines nach der Gleichverteilung gezogenen Knoten. Nach Satz 10.1 gilt
für ein rein zufälliges B (n) in {1, . . . , n}
P(D = x) =
n
X
P(Di = x|B (n) = i)P(B (n) = i)
i=1
n
1X
P(Di = x|B (n) = i)
=
n i=1
n
1X
(n)
=
P(P oi(wi ) = x) + o(1).
n i=1
Sind nun die Gewichte so gegeben wie im Korolalr, so ergibt sich
n
1X
(n)
P(D = x) =
P(P oi(wi ) = x) + o(1)
n i=1
n
=
i
1X
P(P oi((1 − Fw )−1 ( )) = x) + o(1)
n i=1
n
= P(P oi((1 − Fw )−1 (Un )) = x) + o(1),
110
wobei Un eine diskrete Zufallsvariable mit Werten in {0, n1 , . . . , n−1
} ist. Daher folgt
n
−1 (U
P(D = x) = E[e−(1−Fw )
n)
((1 − Fw )−1 (Un ))
+ o(1).
x!
Nun konvergiert aber
D
Un −→ U,
wobei U die Gleichverteilung auf [0, 1] ist, und da für jedes x ≥ 0 die Funktion
y 7→ e−y
yx
x!
stetig und beschränkt ist, folgt
−1 (U
e−(1−Fw )
n)
(1 − Fw )−1 (Un )x
(1 − Fw )−1 (U)x
−1
→ e−(1−Fw ) (U )
.
x!
x!
Da all diese Größen Wahrscheinlichkeiten sind und somit zwischen 0 und 1 liegen, folgt
mit majorisierter Konvergenz
x
−1
∗
−w W
−(1−Fw )−1 (U ) (1 − Fw ) (U)
+ o(1) = E e
+ o(1),
P(D = x) = E e
x!
x!
wenn W die verteilungsfunktion Fw hat. Also hat D eine gemischte Poisson-Verteilung
mit Mischungsverteilung Fw .
Der Beweis von Teil b) ist ähnlich zum Beweis von Satz 10.1 b).
2
Satz 10.1 macht Aussagen über ein festes Element der Gradfolge. Ähliche Resultate lassen sich auch für die gesamte Gradfolge ableiten. Hierzu betrachten wir die empirische
Verteilungsfunktion
n
1X
(n)
Ln =
1l{Di =k} .
n i=1
Wir wollen den folgenden Satz zeigen:
Satz 10.4 Die Gewichte seien gegeben durch
i
wi = [1 − Fw ]−1 ( )
n
für eine Verteilungsfunktion Fw mit endlichem Erwartungswert. Dann gilt für jedes ε > 0
X
P(
Lnk − pk | ≥ ε) → 0,
k
wobei
pk = E[e−w
und W die Verteilungsfunktion Fw besitzt.
111
wk
]
k!
Beweis: Es ist
X
k
(n)
|Pk
− pk | = 2dT V (L(n) , p).
Man überlegt sich schnell,d ass für zwei Wahrscheinlichkeiten µ und ν auf N0 gilt
dT V (µ, ν) → 0 ⇔ max |µ(i) − ν(i)| → 0.
i
Dies impliziert insbesondere
X
X (n)
(n)
P(|Lk − pk | ≥ ε).
|Lk − pk | ≥ ε) ≤
P(
k
k
Nun ist
(n)
ELk = P(D1 = k).
Man kann nun zeigen, dass tatsächlich P(D1 = k) gegen pk konvergiert und zwar gleichmäßig
in l, dass also
ε
(n)
max |ELk − pk | ≤
k
2
für hinreichend großes n gilt (Übung). Also folgt für alle großen n
(n)
P(max |Lk
k
− pk | ≥ ε) ≤
∞
X
ε
(n)
P(Lk − ELnk ≥ ).
2
k=0
Mithilfe der Chebyshev-Ungleichung erhalten wir
4
ε
(n)
(n)
(n)
P(|Lk − ELk |E ) ≤ 2 VLk .
2
ve
(n)
Setzt man die Definition Lk ein, so sieht man, dass
(n)
E(Lk )2 =
1 X
1
n−1
P(D
=
D
=
k)
=
P(D
=
k)
+
P(D1 = D2 = k).
1
i
j
k 2 1≤i,j≤n
n
n
Somit erhalten wir
(n)
VLk ≤
1
P(D1 = k) + (P(D1 = D2 = k) − P2 (D1 = k)).
n
Nun sind die Grade asymptotische unabhängig, also folgt
P(D2 = k|D2 = k) − P(D1 = k) = o(1).
Somit ergibt sich
4 X
ε
(n)
(n)
(n)
V(Pk )
P(max |Lk − ELk | ≥ ) ≤ 2
k
2
ε k
4 X
1
≤ 2
P(D1 = k)( + P(D2 = k|D1 = k) − P(D1 = k)).
ε k
n
Dies konvergiert wegen dominanter Konvergenz gegen 0 (Übung).
112
2